You are on page 1of 142

Building Standards in Educational and Professional Testing

National Testing Service Pakistan Overseas Scholarship Scheme for PhD Studies
National Testing Service Pakistan
(Building Standards in Educational & Professional Testing)









































Email: correspondence@nts.org.pk
Logon to: www.nts.org.pk for more information


Building Standards in Educational and Professional Testing
National Testing Service Pakistan Overseas Scholarship Scheme for PhD Studies







PATTERNS AND
PRACTICE
Building Standards in Educational and Professional Testing
National Testing Service Pakistan Overseas Scholarship Scheme for PhD Studies

2 Quantitative Ability

The Quanti tati ve secti on measures your basi c mathemati cal ski l l s,
understandi ng of el ementary mathemati cal concepts, and the abi l i ty to reason
quanti tati vel y and sol ve probl ems i n a quanti tati ve setti ng. There i s a
bal ance of questi ons requi ri ng basi c knowl edge of ari thmeti c, al gebra,
geometry, and data anal ysi s. These are essenti al content areas usual l y
studi ed at the hi gh school l evel .

The questi ons i n the quanti tati ve secti on can al so be f rom

Di screte Quanti tati ve Questi on
Quanti tati ve Compari son Questi on
Data Interpretati on Questi on etc.

The di stri buti on i n thi s gui de i s onl y to f aci l i tate the candi dates. Thi s
di stri buti on i s not a part of test templ ate, so, a test may contai n al l the
questi ons of one f ormat or may have a random number of questi ons of
di f f erent f ormats.

Thi s chapter i s di vi ded i nto 4 maj or secti ons. The f i rst di scusses the syl l abus
/contents i n each secti on of the test respecti vel y and the remai ni ng three
secti ons address the questi on f ormat, gui de l i nes to attempt the questi ons i n
each f ormat and some exampl e questi ons.

2.1 General Mathematics Review

2. 1. 1 Arithmetic

The f ol l owi ng are some key poi nts, whi ch are phrased here to ref resh your
knowl edge of basi c ari thmeti c pri nci pl es.

Basic arithmetic

For any number a, exactl y one of the fol l owi ng i s true:
o a i s negati ve
o a i s zero
o a i s posi ti ve

The onl y number that i s equal to i ts opposi te i s 0 (e.g.
0 a a only if a = =
)


If 0 i s mul ti pl i ed to any other number, i t wi l l make i t zero (
0 0 a =
).

Product or quoti ent of two numbers of the same si gn are al ways posi ti ve
and of a di fferent si gn are al ways negati ve. E.g. i f a posi ti ve number i s
mul ti pl i ed to a negati ve number the resul t wi l l be negati ve and i f a
negati ve number i s di vi ded by another negati ve number the resul t wi l l be
posi ti ve.

See the fol l owi ng tabl es for al l combi nati ons.


Building Standards in Educational and Professional Testing
National Testing Service Pakistan Overseas Scholarship Scheme for PhD Studies
+ or + = +
+ or =
or + =
or = +
The sum of two posi ti ve numbers i s al ways posi ti ve.

The sum of two negati ve numbers i s al ways negati ve.

Subtracti ng a number from another i s the same as addi ng i ts opposi te

( ) a b a b = +

The reci procal of a number a i s
a
1


The product of a number and i ts reci procal i s al ways one

1
1
=
a
a


Di vi di ng by a number i s the same as mul ti pl yi ng by i ts reci procal

b
a b a
1
=


Every i nteger has a fi ni te set of factors (di vi sors) and an i nfi ni te set of
multipliers.

If a and b are two i ntegers,the f ol l owi ng four terms are synonyms
o a i s a divisor of b
o a i s a factor of b
o b i s a divisible b y a
o b i s a multiple of a

They al l mean that when a i s di vi ded by b there i s no remainder.

Posi ti ve i ntegers, other than 1, have at l east two posi ti ve factors.

Posi ti ve i ntegers, other than 1, whi ch have exactl y two factors, are known
as prime numbers.

Every i nteger greater than 1 that i s not a pri me can be wri tten as a product
of pri mes.
To fi nd the pri me factori zati on of an i nteger, fi nd any two factors of that
number, i f both are pri mes, you are done; i f not, conti nue factori zati on
unti l each factor i s a pri me.
E.g. to fi nd the pri me factori zati on of 48, two factors are 8 and 6. Both of
them are not pri me numbers, so conti nue to factor them.
Factors of 8 are 4 and 2, and of 4 are 2 and 2 (2 2 2).
Factors of 6 are 3 and 2 (3 2).
So the number 48 can be wri tten as 2 2 2 2 3.

The Least Common Mul ti pl e (LCM) of two i ntegers a and b i s the smal l est
i nteger whi ch i s di vi si bl e by both a and b, e.g. the LCM of 6 and 9 i s 18.

Building Standards in Educational and Professional Testing
National Testing Service Pakistan Overseas Scholarship Scheme for PhD Studies
The Greatest Common Di vi sor (GCD) of two i ntegers a and b i s the l argest
i nteger whi ch di vi des both a and b, e.g. the GCD of 6 and 9 i s 3.

The product of GCD and LCM of two i ntegers i s equal to the products of
numbers i tsel f. E.g. 6 9 = 54
3 18 = 54 (where 3 i s GCD and 18 i s LCM of 6 and 9).

Even numbers are al l the mul ti pl es of 2 e.g. ( , 4, 2, 0, 2, 4, )

Odd numbers are al l i ntegers not di vi si bl e by 2 ( , 5, 3, 1, 1, 3, 5, )

If two i ntegers are both even or both odd, thei r sum and di fference are
even.

If one i nteger i s even and the other i s odd, thei r sum and di fference are
odd.

The product of two i ntegers i s even unl ess both of them are odd.

When an equati on i nvol ves more than one operati on, i t i s i mportant to
carry them out i n the correct order. The correct order i s Parentheses,
Exponents, Mul ti pl i cati on and Di vi si on, Addi ti on and Subtracti on, or just
the fi rst l etters PEMDAS to remember the proper order.

Exponents and Roots

Repeated addi ti on of the same number i s i ndi cated by mul ti pl i cati on:
17 + 17 + 17 + 17 + 17 = 5 17

Repeated mul ti pl i cati on of the same number i s i ndi cated by an exponent:
17 17 17 17 17 = 17
5


In the expressi on 17
5
, 17 i s cal l ed base and 5 i s the exponent.

For any number b: b
1
= b and b
n
= b b b, where b i s used n ti mes
as factor.

For any numbers b and c and posi ti ve i ntegers m and n:
o
n m n m
b b b
+
=
o
n m
n
m
b
b
b

=
o
mn n m
b b = ) (
o
m m m
bc c b ) ( =

If a i s negati ve, a
n
i s posi ti ve i f n i s even, and negati ve i f n i s odd.

There are two numbers that sati sfy the equati on
9
2
= x
:
3 = x
and
3 = x
.
The posi ti ve one, 3, i s cal l ed the (pri nci pal ) square root of 9 and i s denoted
by symbol
9
. Cl earl y, each perfect square has a square root:
0 0 =
,
3 9 =
,
6 36 =
,
13 169 =
,
25 225 =
etc.

Building Standards in Educational and Professional Testing
National Testing Service Pakistan Overseas Scholarship Scheme for PhD Studies
For any posi ti ve number a there i s a posi ti ve number b that sati sfi es the
equati on
b a =
.
For any posi ti ve i nteger,
a a a a = =
2
) (
.

For any posi ti ve numbers a and b:
o
b a ab =


and
b
a
b
a
=


o
7 4 3 16 9 16 9 25 5 = + = + + = =
+ +
as
b a b a


Al though i t i s al ways true that
a a =
2
) (
,
a a =
2
i s true onl y i f a i s
posi ti ve as
5 5 25 ) 5 (
2
= =

For any number a,
2
n
n
a a =
.


For any number a, b and c:


ac ab c b a + = + ) (

ac ab c b a = ) (


and i f
0 a



a
c
a
b
a
c b
+ =
+ ) (

a
c
a
b
a
c b
=
) (


Inequalities

For any number a and b, exactl y one of the fol l owi ng i s true:

b a >
or
b a =
or
b a <
.

For any number a and b,
b a >
means that
b a
i s posi ti ve.

For any number a and b,
b a <
means that
b a
i s negati ve.

For any number a and b,
b a =
means that
b a
i s zero.

The symbol

means greater than or equal to and the symbol

means l ess
than or equal to. E.g. the statement
5 x
means that x can be 5 or any
number greater than 5.
The statement
5 2 < < x
i s an abbrevi ati on of
x < 2
and
5 < x
.

Addi ng or subtracti ng a number to an i nequal i ty preserves i t.

If
b a <
, then
c b c a + < +
and
c b c a <
.

Building Standards in Educational and Professional Testing
National Testing Service Pakistan Overseas Scholarship Scheme for PhD Studies
e.g.
10 6 10 5 10 6 10 5 6 5 < + < + < and



Addi ng i nequal i ti es i n same di recti on preserves i t:
If
d c and b a < <
, then
d b c a + < +
.

Mul ti pl yi ng or di vi di ng an i nequal i ty by a posi ti ve number preserves i t. If
b a <
and
c
i s a posi ti ve number, then
c b c a <
and
c
b
c
a
<
.

Mul ti pl yi ng or di vi di ng an i nequal i ty by a negati ve number reverses i t. If
b a <
and
c
i s a negati ve number, then
c b c a >
and
c
b
c
a
>
.

If si des of an i nequal i ty are both posi ti ve and both negati ve, taki ng the
reci procal reverses the i nequal i ty.

If
0 1 x < <
and
a
i s posi ti ve, then
xa a <
.

If
0 1 x < <
and
m and n
are i ntegers wi th
m n >
, then
m n
x x x < <
.


If
0 1 x < <
, then
x x >
.


If
0 1 x < <
, then
1
x
x
>
and
1
1
x
>


Properties of Zero

0 i s the onl y number that i s nei ther negati ve nor posi ti ve.

0 i s smal l er than every posi ti ve number and greater than every negati ve
number.

0 i s an even i nteger.

0 i s a mul ti pl e of every i nteger.

For every number
a
:
0 0 a a and a a + = =
.

For every number
a
:
0 0 a =
.

For every posi ti ve i nteger
n
:
0 0
n
=
.

For every number
a
(i ncl udi ng 0):
0
0
a
a and

are undefi ned symbol s.

For every number a (other than 0):
0
0 0 a
a
= =
.
Building Standards in Educational and Professional Testing
National Testing Service Pakistan Overseas Scholarship Scheme for PhD Studies
0 i s the onl y number that i s equal to i ts opposi te:
0 0 =
.

If the product of two or more numbers i s 0, at l east one of them i s 0.

Properti es of One
For any number
a
:
1 a a =
and
1
a
a =
.
For any number
n
:
1 1
n
=
.

1 i s the di vi sor of every i nteger.

1 i s the smal l est posi ti ve i nteger.

1 i s an odd i nteger.

1 i s not a pri me.

Fractions and Decimals

When a whol e i s di vi ded i nto n equal parts, each part i s cal l ed one nth of
the whol e, wri tten

1
n
. For exampl e, i f a pi zza i s cut (di vi ded) i nto 8 equal
sl i ces, each sl i ce i s one ei ghth (
1
8
) of the pi zza; a day i s di vi ded i nto 24
equal hours, so an hour i s one twenty-fourth
1
( )
24

of a day and an i nch i s
one twel fth (
1
12
) of a foot. If one works for 8 hours a day, he works ei ght
twenty-fourth (
8
24
) of a day. If a hockey sti ck i s 40 i nches l ong, i t
measures forty twel fths
40
( )
12
of a foot.
The numbers such as
1
8
,
1
24
,
8
24
and
40
12
, i n whi ch one i nteger i s wri tten
over the second i nteger, are cal l ed fractions. The center l i ne i s cal l ed the
fracti on bar. The number above the bar i s cal l ed the numerator, and the
number bel ow the bar i s cal l ed denominator.

The denomi nator of a fracti on can never be 0.
A fracti on, such as
1
24
, i n whi ch the denomi nator i s greater than
numerator, i s known as a proper fraction. Its val ue i s l ess than one.
A fracti on, such as
40
12
, i n whi ch the denomi nator i s l ess than numerator, i s
known as an improper fraction. Its val ue i s greater than one.
Building Standards in Educational and Professional Testing
National Testing Service Pakistan Overseas Scholarship Scheme for PhD Studies
A fracti on, such as,
12
12
i n whi ch the denomi nator i s equal to the numerator,
i s al so known as an improper fraction. But, Its val ue i s one.
Every fracti on can be expressed i n deci mal form (or as a whol e number) by
di vi di ng the number by the denomi nator.
3 3 8 48 100
0.3, 0.75, 1, 3, 12.5
10 4 8 16 8
= = = = =

Unl i ke the exampl es above, when most fracti ons are converted to deci mal s,
the di vi si on does not termi nate, after 2 or 3 or 4 deci mal pl aces; rather i t
goes on forever wi th some set of di gi ts repeati ng i t.
2 3 5 17
0.66666..., 0.272727..., 0.416666..., 1.133333...
3 11 12 15
= = = =

Building Standards in Educational and Professional Testing
National Testing Service Pakistan Overseas Scholarship Scheme for PhD Studies

To compare two deci mal s, fol l ow these rul es:

o Whi chever number has the greater number to the l eft of the deci mal
poi nt i s greater: si nce 11 > 9, 11.0001 > 9.8965 and si nce 1 > 0,
1.234 > .8. (Recal l that i f a deci mal i s wri tten wi thout a number on
l eft of deci mal poi nt, you may assume that a 0 i s there, so, .8 =
0.8).
o If the numbers to the l eft of the deci mal poi nt are equal , proceed as
fol l ows:

If the numbers do not have the same number of di gi ts to the ri ght
of the deci mal poi nt, add zeroes to the end of the shorter one to
make them equal i n l ength.
Now compare the numbers i gnori ng the deci mal poi nt.
For exampl e, to compare 1.83 and 1.823, add a 0 to the end of
1.83 formi ng 1.830. Now compare them, thi nki ng of them as whol e
numbers wi thout deci mal poi nt: si nce 1830 > 1823, then 1.830
>1.823.

There are two ways to compare fracti ons:
o Convert them to deci mal s by di vi di ng, and use the method al ready
descri bed to compare these deci mal s. For exampl e to compare
2
5

and
1
4
, convert them to deci mal s.
2
0.4
5
=
and
1
0.25
4
=
. Now, as 0.4
> 0.25,
2
5
>
1
4
.
o Cross mul ti pl y the fracti ons. For exampl e to compare
2
5
and
1
4
,
cross mul ti pl y:
2
5

1
4

Si nce

2 4 1 5 >
,

then

2
5
>
1
4
.
Whi l e compari ng the fracti ons, i f they have same the denomi nators, the
fracti on wi th the l arger numerator i s greater. For exampl e
3 2
5 5
>
.

If the fractions have the same numerator, the fracti on with the
smaller denominator is greater. For example
3 3
5 10
>
.
Two fracti ons are cal l ed equivalent fractions i f both of them have same
deci mal val ue.
For exampl e,
1 5
2 10
=
as both of these are equal to 0.5.
Another way to check the equi val ence of two fracti ons i s to cross-mul ti pl y.
If both of the products are same, the fracti ons are equi val ent. For Exampl e,
to compare
2
5
wi th
6
15
, cross-mul ti pl y. Si nce
2 15 6 5 =
, both of the
fracti ons are equi val ent.
Every fracti on can be reduced to l owest terms by di vi di ng the numerator
and denomi nator by thei r greatest common di vi sor (GCD). If the GCD i s 1,
Building Standards in Educational and Professional Testing
National Testing Service Pakistan Overseas Scholarship Scheme for PhD Studies
the fracti on i s al ready i n l owest terms. For exampl e to reduce
10
15
, di vi de
both numerator and denomi nator by 5 (whi ch i s GCD of 10 and 15). Thi s
wi l l reduce the fracti on to
2
3
.
To mul ti pl y two fracti ons, mul ti pl y thei r numerators and mul ti pl y thei r
denomi nators. For exampl e
3 4 3 4 12
5 7 5 7 35

= =

.
To mul ti pl y a number to a fracti on, wri te that number as a fracti on whose
denomi nator i s 1. For exampl e
3 3 7 3 7 21
7
5 5 1 5 1 5

= = =

.
When a probl em requi res you to fi nd the fracti on of a number, mul ti pl y that
fracti on wi th the number. For exampl e, to fi nd two fi fth (
2
5
) of 200,
mul ti pl y:
2 2 200 400
200
5 5 1
= =
80
80
5
=
/
.
The reci procal of a fracti on
a
b
i s another fracti on
b
a
si nce
1
a b
b a
=

To di vi de one fracti on by the other fracti on, mul ti pl y the reci procal of
di vi sor wi th the di vi dend. For exampl e,
22 11 22
7 7
=
2
7
7 11

2
2
1
= =
.
To add or subtract the fracti ons wi th same denomi nator, add or subtract
numerators and keep the denomi nator. For
exampl e
4 1 5 4 1 3
9 9 9 9 9 9
and + = =
.

Percents

The word percent means hundredth. We use the symbol % to express the
word percent. For exampl e 15 percent means 15 hundredths and can be
wri tten wi th a % symbol , as a fracti on, or as a deci mal .
20
20% 0.20
100
= =
.
To convert a deci mal to a percent, move the deci mal poi nt two pl aces to the
ri ght, addi ng 0s i s necessary, and add the percent symbol (%).
For exampl e, 0.375 = 37.5% 0.3 = 30% 1.25 = 125% 10=1000%

To convert a fracti on to a percent, fi rst convert that fracti on to deci mal ,
than use the method stated above to convert i t to a percent.

To convert a percent to a deci mal , move the deci mal poi nt two pl aces to the
l eft and remove the % symbol . Add 0s i f necessary.
For exampl e, 25% = 0.25 1% =0.01 100% = 1

You shoul d be fami l i ar wi th the fol l owi ng basi c conversi ons:
1 5
0.50 50%
2 10
= = =

1 2
0.20 20%
5 10
= = =

Building Standards in Educational and Professional Testing
National Testing Service Pakistan Overseas Scholarship Scheme for PhD Studies
1
0.25 25%
4
= =

3
0.75 75%
4
= =


For any posi ti ve i nteger
a
:
% a
of 100 i s
a
.

For any posi ti ve numbers
a
and
b
:
% % a of b b of a =


The percent change i n the quanti ty i s
100%
actual change
original amount

. For exampl e:
If the pri ce of a l amp goes from Rs.80 to Rs.100, the actual i ncrease i s
Rs.20, and the percent i ncrease i s
20 1
100% 100% 25%
80 4
= =
.
If
a b <
, the percent i ncrease i n goi ng from
a
to
b
i s al ways greater than
percent decrease i n goi ng from
b
to
a
.

To i ncrease a number by
% k
, mul ti pl y i t by
1 % k +
, and to decrease a
number by % k , mul ti pl y i t by 1 % k . For exampl e, the val ue of an
i nvestment of Rs. 20,000 after 25% i ncrease i s
20, 000 (1 25%) 20, 000 (1.25) 25, 000 + = = .

If a number i s the resul t of i ncreasi ng another number by
% k
, to fi nd the
ori gi nal number di vi de i t by
1 % k +
, and i f a number i s the resul t of
decreasi ng another number by % k , to fi nd the ori gi nal number, di vi de i t by
1 % k
.
For exampl e, The government announced an 20%i ncrease i n sal ari es. If
after the i ncrement, The sal ary of a parti cul ar empl oyee i s Rs. 18, 000,
what was the ori gi nal sal ary?
Ori gi nal sal ary (i n Rs.) =
18, 000 18, 000
15, 000
1 1 20% 1.20
current salary
percent increase
= = =
+ +


Ratios and Proportions

A rati o i s a fracti on that compares two quanti ti es that are measured i n the
same uni ts. The fi rst quanti ty i s the numerator and the second quanti ty i s
denomi nator. For exampl e, i f there are 16 boys and 4 gi rl s, we say that the
rati o of the number of boys to the number of gi rl s on the team i s 16 to 4,
or
16
4
. Thi s i s often wri tten as 16:4. Si nce a rati o i s j ust a fracti on, i t can
be reduced or converted to a deci mal or a percent. The Fol l owi ng are
di fferent ways to express the same rati o:

16 4 to
,
16: 4
,
16
4
,
4
1
,
0.25
,
25%

If a set of objects i s di vi ded i nto two groups i n the rati on
: a b
, then the
fi rst group contai ns
a
a b +
of the total objects and si mi l arl y the second group
Building Standards in Educational and Professional Testing
National Testing Service Pakistan Overseas Scholarship Scheme for PhD Studies
contai ns
b
a b +
of the total number of obj ects. Thi s rul e appl i es to extended
rati os, as wel l . If a set i s di vi ded i nto three groups i n the rati o
: : a b c
, then
the fi rst group contai ns
a
a b c + +
of the total objects, and so on.

A proporti on i s an equati on that states that two rati os are equi val ent. Si nce
rati os are just fracti ons, any equati on such as
4 10
6 15
=
i n whi ch each si de i s
a si ngl e fracti on i s proporti on. Thi s proporti on states that 4 rel ates to 6 i n
same rati o as 10 rel ates to 15.
For each proporti on of the form
a c
b d
=
,
ad bc =
. Thi s property can be used
to sol ve proporti ons for unknowns (vari abl es). For exampl e: If 3 oranges
cost Rs.5, how many oranges can you buy for Rs.100. To sol ve thi s
probl em we have to set up a proporti on. If the number of oranges for
Rs.100 i s
x
, then:
3 3 100
3 100 5 60
5 100 5
x
x x x

= = = =


Averages

The average of a set of n numbers i s the sum of those numbers di vi ded by
n.

sum of n numbers
average
n
=

or si mpl y

Sum
A
n
=

the techni cal name for these ki nd of averages i s Ari thmeti c Mean.

If you know the average of n numbers, mul ti pl y that average wi th n to get
the sum of numbers.

If al l the numbers i n a set are the same, then that number i s the average.


Assume that the average of a set of numbers i s A. If a new number x i s
added to that set, the new average wi l l be;

o Greater if x is greater than the existing average
o Smaller if x is small er than the existing average
o Unchanged if x is equal to the existing average

Ari thmeti c sequence i s an ordered set of numbers, such that, the di fference
between two consecuti ve numbers i s the same.

If there i s an ari thmeti c sequence of n terms, then the average cal cul ati on
can be made si mpl e by usi ng these rul es.
o The average of the terms i n that sequence wi l l be the mi ddl e term,
i f n i s odd.
o If n i s even, the average wi l l be the average of two mi ddl e terms.

2.1.2 Algebra

Building Standards in Educational and Professional Testing
National Testing Service Pakistan Overseas Scholarship Scheme for PhD Studies
Polynomials


A monomi al i s any number or vari abl e or product of numbers and vari abl es.
For exampl e
3 2 3 2
3, 4, , , 3 , 2 , 5 , 1.5 , x y x xyz x xy a b
are al l monomi al s.

The number that appears i n front of a vari abl e i n a monomi al i s cal l ed the
coeffi ci ent. The coef fi ci ent of
3
5x
i s 5. If there i s no number, the
coeffi ci ent i s ei ther 1 or 1, because
x
means
1x
and
x
means
1x
.

A pol ynomi al i s a monomi al or the sum of two or more monomi al s. Each
monomi al that makes up the pol ynomi al i s cal l ed a term of that pol ynomi al .

If a pol ynomi al has onl y one term i t i s a si mpl e monomi al , i f i t has two
terms, i t i s known as bi nomi al and i f i t has three terms, i t i s cal l ed
tri nomi al .

Two terms are cal l ed l i ke terms i f they di ffer onl y i n thei r coeffi ci ents.
3
5x

and
3
2x
are l i ke terms, whereas,
3
5x
and
2
5x
are not.

If l i ke terms are i nvol ved i n a pol ynomi al , they can be combi ned, by addi ng
thei r coeffi ci ents, to make that pol ynomi al si mpl er. The pol ynomi al
2 2
3 4 5 2 7 x x x x + +
i s equi val ent to the pol ynomi al
2
9 7 x x +
.
Al l l aws of ari thmeti c are al so appl i cabl e to pol ynomi al s. Most i mportant of
them i s PEMDAS.

Pol ynomi al s can be added, subtracted, mul ti pl i ed or di vi ded.

To add two pol ynomi al s, put a pl us si gn between them, erase the
parentheses, and combi ne l i ke terms.


Example:
What i s the sum of
2
5 10 7 x x +
and
2
3 4 2 x x +
?

Solution:
2 2
2 2
2
(5 10 7) (3 4 2)
5 10 7 3 4 2
8 6 5
x x x x
x x x x
x x
+ + +
= + + +
= +


To subtract two pol ynomi al s, reverse the si gns of subtrahend, and add two
pol ynomi al s as done before.

Example:
Subtract
2
3 4 2 x x +
from
2
5 10 7 x x +


Building Standards in Educational and Professional Testing
National Testing Service Pakistan Overseas Scholarship Scheme for PhD Studies
Solution:
2 2
2 2
2 2
2
(5 10 7) (3 4 2)
(5 10 7) ( 3 4 2)
5 10 7 3 4 2
2 14 9
x x x x
x x x x
x x x x
x x
+ +
= + + +
= + +
= +


To mul ti pl y monomi al s, fi rst mul ti pl y thei r coeffi ci ents, and then mul ti pl y
thei r vari abl es by addi ng the exponents.

Example:
What i s the product of
2
3x yz
from
2 2
2x y
?

Solution:
2 2 2
2 2 2
4 3
(3 )( 2 )
(3 2)( )( )( )
6
x yz x y
x x y y z
x y z

=
=


To mul ti pl y a monomi al by a pol ynomi al , just mul ti pl y each term of the
pol ynomi al by the monomi al .






Example:
What i s the product of
3x
from
2 2
3 6 2 x xy +
?

Solution:
2 2
2 2
3 2 2
(3 )(3 6 2)
(3 3 ) (3 6 ) (3 2)
9 18 6
x x xy
x x x xy x
x x y x
+
= +
= +


To mul ti pl y two bi nomi al s, mul ti pl y each term of fi rst bi nomi al by each term
of second bi nomi al , then add the resul ts.

Example:
What i s the product of
3x y +
from
2 2
3 6 x xy
?

Solution:
2 2
2 2 2 2
2 2 2 2 3
2 2 2 2 3
(3 )(3 6 )
(3 3 ) (3 ( 6 )) ( 3 ) ( ( 6 ))
(9 ) ( 18 ) (3 ) ( 6 )
9 18 3 6
x y x xy
x x x xy y x y xy
x x y x y xy
x x y x y xy
+
= + + +
= + + +
= +


The three most i mportant bi nomi al products are:
Building Standards in Educational and Professional Testing
National Testing Service Pakistan Overseas Scholarship Scheme for PhD Studies
o
2 2 2 2
( )( ) x y x y x xy xy y x y + = + + = +

o
2 2 2 2
( )( ) 2 x y x y x xy xy y x xy y + + = + + + = + +

o
2 2 2 2
( )( ) 2 x y x y x xy xy y x xy y = + = +

Memori zi ng these can save a l ot of cal cul ati on ti me duri ng the test.

To di vi de a pol ynomi al by a monomi al , di vi de each term of the pol ynomi al
by the monomi al .

Example:
What i s the quoti ent i f
2 3
32 12 x y xy z +
i s di vi ded by
8xy
?

Solution:
2 3 2 3
2
32 12 32 12 3
4 (by reducing the terms)
8 8 8 2
x y xy z x y xy z
x y z
xy xy xy
+
= + = +

Building Standards in Educational and Professional Testing
National Testing Service Pakistan Overseas Scholarship Scheme for PhD Studies
Sol vi ng Equati ons and Inequal i ti es

The basi c pri nci pl e i n sol vi ng equati ons and i nequal i ti es i s that you can
mani pul ate them i n any way as l ong as you do the same thing to both
sides. For exampl e you may add a number to both si des, or you may
di vi de or mul ti pl y both si des wi th same number etc.
By usi ng the fol l owi ng si x-step method, you can sol ve most of the
equati ons and i nequal i ti es. The method i s expl ai ned wi th the hel p of an
exampl e.
Example:
i f
1
3( 2) 2( 1) 1
2
x x x + = + +
, what i s the val ue of
x
?

Solution:
Step What to do Example
1 Get ri d of fracti ons and
deci mal s by mul ti pl yi ng
both si des by the LCD.
Mul ti pl y each si de by 2 to get:
6( 2) 4( 1) 2 x x x + = + +

2 Get ri d of al l parentheses
by sol vi ng them.
6 12 4 4 2 x x x + = + +

3 Combi ne l i ke terms on
each si de.
7 12 4 6 x x = +

4 By addi ng and subtracti ng
get al l the vari abl es on
one si de (mostl y l eft).
Subtract
4x
from each si de to get:
3 12 6 x =

5 By addi ng or subtracti ng
get al l pl ai n numbers on
the other si de.
Add 12 to each si de to get:
3 18 x =

6 Di vi de both si des by the
coeffi ci ent of the vari abl e.
(If you are deal i ng wi th
an i nequal i ty and you
di vi de wi th a negati ve
number, remember to
reverse the i nequal i ty.)
Di vi de both si des by 3 to get:
6 x =


When you have to sol ve one vari abl e and the equati on/i nequal i ty i nvol ve
more than one vari abl e, treat al l other vari abl es as pl ai n numbers and
appl y the si x-step method.
Example:
i f
3 a b c =
, what i s the val ue of
b
i n terms of
a
and
c
?
Solution:
Step What to do Example
1 There are no fracti ons and
deci mal s.

2 There are no parentheses.
3 There are no l i ke terms.
4 By addi ng and subtracti ng get
al l the vari abl es on one si de.
Remember there i s onl y one
vari abl e
b
, whi ch i s on one
si de onl y.
5 By addi ng or subtracti ng get
al l pl ai n numbers on the
other si de.
Remember we are
consi deri ng
a
and
c
as
pl ai n number. Add
c
to
each si de to get:
3 a c b + =

Building Standards in Educational and Professional Testing
National Testing Service Pakistan Overseas Scholarship Scheme for PhD Studies
6 Di vi de both si des by the
coeffi ci ent of the vari abl e.
Di vi de both si des by 3 to
get:
3
a c
b
+
=

It i s not necessary to fol l ow these steps i n the order speci fi ed. Some ti mes
i t makes the probl em much easi er, i f you change the order of these steps.

Example:
If
4 11 x =
, what i s the val ue of
x
-8?

Solution:
Goi ng i mmedi atel y to step 5, add 4 to each si de to get:
15 x =
. Now
subtract 8 from both si des to get:
8 7 x =
.

Doi ng the same thi ng on each si de of an equati on does not mean doi ng the
same thi ng to each term of the equati on. Thi s i s very i mportant i f you are
doi ng di vi si ons, or deal i ng wi th exponents and roots.

Example:
If
0 a >
and
2 2 2
a b c + =
, what i s the val ue of
a
i n terms of
b
and
c
.

Solution:
2 2 2 2 2 2
a b c a c b + = =
. Now you can t take a square root of each term
to get
a c b =
. You must take the square root of each si de:
2 2 2 2 2
a c b a c b = =


Another type of equati on i s that i n whi ch the vari abl e appears i n exponent.
These equati ons are basi cal l y sol ved by i ncepti on.
Exampl e:
If
3
2 32
x+
=
, what i s the val ue of
2
3
x+
?
Sol uti on:
3 3 5
2 32 2 2 3 5 2
x x
x x
+ +
= = + = =
.
Now as
2 x =
, you can get
2 4
2 2 4 3 3 81
x
x x
+
= + = = =


A system of equati ons i s a set of two or more equati ons havi ng two or more
vari abl es. To sol ve such equati ons, you must fi nd the val ue of each
vari abl e that wi l l make each equati on true.

To sol ve a system of equati ons, add or subtract them to get a thi rd
equati on. If there are more than two equati ons you can just add them.

Example:
If
10 x y + =
and
10 x y =
what i s the val ue of
y
?


Solution:
Building Standards in Educational and Professional Testing
National Testing Service Pakistan Overseas Scholarship Scheme for PhD Studies
Add two equati ons:
10
2
2 12 6
x y
x y
x x
+ =
=
= =

Now repl aci ng
x
wi th 6 i n the fi rst equati on:
6 10 4 y y + = =


If you know the val ue of one vari abl e i n a system of two equati ons, you can
use thi s val ue to get the val ue of the other vari abl e. As i t i s done i n the
previ ous questi on.


Word probl ems
To sol ve word probl ems, fi rst transl ate the probl em from Engl i sh to
Al gebra. Whi l e transl ati ng, use vari abl es to represent unknowns. Once
transl ated, i t i s easy to sol ve them usi ng the techni ques you have l earned
i n previ ous secti ons.
Fol l owi ng Engl i sh to Al gebra di cti onary wi l l be hel pful i n transl ati ng word
probl ems to al gebrai c expressi ons.

English words
Mathematical
meaning
Symbol
Is, was, wi l l be, had, has, wi l l
have, i s equal to, i s the same as
Equal s =

Pl us, more than, sum, i ncreased
by, added to, exceeds, recei ved,
got, ol der than, farther than,
greater than
Addi ti on +

Mi nus, fewer, l ess than,
di fference, decreased by,
subtracted from, younger than,
gave, l ost
Subtracti on

Ti mes, of, product, mul ti pl i ed by Mul ti pl i cati on

Di vi ded by, quoti ent, per, for Di vi si on
or
a
b


More than, greater than Inequal i ty >

At l east Inequal i ty

Fewer than, l ess than Inequal i ty <

At most Inequal i ty

What, how many, etc. Unknown quanti ty
x

(Some vari abl e)

Exampl es:
o The sum of 5 and some number is 13. 5 13 x + =
o Javed was two years younger than Saleem. 2 J S =
o Bilal has at most Rs.10,000. 10000 B
o The product of 2 and a number exceeds that
number by 5 (is 5 more than that number). 2 5 N N = +

In word probl ems, you must be sure about what you are answeri ng. Do not
answer the wrong questi on.

In probl ems i nvol vi ng ages, remember that years ago means you need to
subtract, and years from now means you need to add.

Building Standards in Educational and Professional Testing
National Testing Service Pakistan Overseas Scholarship Scheme for PhD Studies
Di stance probl ems al l depend on three vari ati ons of the same formul a:
o
distance speed time =

o
distance
speed
time
=

o
distance
time
speed
=


Example:
How much l onger, i n seconds, i s requi red to dri ve 1 mi l e at 40 mi l es per
hour than at 60 mi l es per hour?

Solution:
The ti me to dri ve at 40 mi l es per hour can be cal cul ated as
1 time
=
1
40
hours =
1
40
2
60
3
mi nutes =
3
2
60
30
seconds =
90
seconds

The ti me to dri ve at 60 mi l es per hour can be cal cul ated as
2 time
=
1
60
hours =
1
60
60
mi nutes =
1 60
seconds =
60
seconds

1 2 90 60 30 difference time time = = = seconds.

2. 1. 3 Geometry

Li nes and Angles

An angle i s formed at the i ntersecti on of two l i ne segments, rays or l i nes.
The poi nt of i ntersecti on i s cal l ed the vertex. Angl es are measured i n
degrees.

Angl es are cl assi fi ed accordi ng to thei r degree measures.
An acute angl e measures l ess than
90

A right angl e measures
90

An obtuse angl e measures more than
90
but l ess than
180

A strai ght angl e measures
180


If two or more angl es combi ne together to form a strai ght angl e, the sum
of thei r measures i s
180
.
180 a b c d + + + =


Building Standards in Educational and Professional Testing
National Testing Service Pakistan Overseas Scholarship Scheme for PhD Studies
The sum of al l the measures of al l the angl es around a poi nt i s
360

360 a b c d e + + + + =


When two l i nes i ntersect, four angl es are formed, two angl es i n each pai r of
opposi te angl es are cal l ed vertical angles. Verti cal angl es, formed by the
i ntersecti on of two l i nes, have equal measures.
a c =
and
b d =


If one of the angl es formed by the i ntersecti on of two l i nes i s a ri ght angl e,
then al l four angl es are ri ght angl es. Such l i nes are cal l ed perpendicular
l i nes

90 a b c = = =


In the fi gure bel ow a l i ne l di vi des the angl e i n two equal parts. Thi s l i ne i s
sai d to bisect the angl e. The other l i ne k bi sects another l i ne i nto two
equal parts. Thi s l i ne i s sai d to bi sect a l i ne.

Two l i nes are sai d to be parallel, i f they never i ntersect each other.
However, i f a thi rd l i ne, cal l ed a transversal, i ntersects a pai r of paral l el
l i nes, ei ght angl es are formed. And the rel ati onshi p among theses angl es i s
shown i n the fol l owi ng di agram.

- Al l four acute angl es are equal
a c e g = = =

- Al l four obtuse angl es are equal
b d f h = = =

- The sum of any pai r of acute and
obtuse angl e i s
180
, e.g.
180 , 180 , 180 a d d e b g + = + = + =
etc.

Tri angl es
Building Standards in Educational and Professional Testing
National Testing Service Pakistan Overseas Scholarship Scheme for PhD Studies
In any tri angl e, the sum of the measures of the three angl es i s
180
.
180 x y z + + =


In any tri angl e:
o The l ongest si de of tri angl e i s opposi te the l argest angl e.
o The shortest si de i s opposi te the smal l est angl e.
o Si des wi th the same l ength are opposi te the angl es wi th the same
measure.

Tri angl es are cl assi fi ed i nto three di fferent ki nds wi th respect to the
l engths of si des.
o Scalene: i n whi ch al l three si des are of di fferent l engths.
o Isosceles: i n whi ch two of the si des of tri angl e are equal i n
l ength, the thi rd i s di fferent.
o Equilateral: i n whi ch al l three si des are equal i n l ength.

Tri angl es are al so cl assi fi ed wi th respect to the angl es.
o Acute triangle: i n whi ch al l three angl es are acute.
o Obtuse triangle: i n whi ch one angl e i s obtuse and two are acute.
o Right triangle: Thi s has one ri ght and two acute angl es.

In a ri ght tri angl e, the opposi te to the ri ght angl e i s known as hypotenuse
and i s the l ongest si de. The other two si des are cal l ed legs.

In any ri ght tri angl e, the sum of the measures of the two acute angl es i s
90
.


By Pythagorean Theorem, the sum of squares of the l engths of l egs of a
ri ght tri angl e i s al ways equal to the square of l ength of hypotenuse.

2 2 2
a b c + =

In any tri angl e, the sum of any two si des i s al ways greater than the thi rd
one. And the di fference of any two si des i s al ways l ess than the thi rd one.

a b c + >
and
a b c <


The perimeter of a tri angl e i s cal cul ated by addi ng the l engths of al l the
si des of that tri angl e.
Building Standards in Educational and Professional Testing
National Testing Service Pakistan Overseas Scholarship Scheme for PhD Studies


perimeter a b c = + +

The area of a tri angl e i s cal cul ated by the formul a:
1
2
area bh =
where
b
i s
the base of the tri angl e and
h
i s the height of the tri angl e.
o Any si de of tri angl e can be taken as the base.
o Hei ght i s the al ti tude (perpendi cul ar) drawn to the base from i ts
opposi te vertex.
o In a ri ght tri angl e any l eg coul d be taken as the base, the other
wi l l be the al ti tude.




Quadrilateral and other Polygons

A polygon i s a cl osed geometri c fi gure, made up of l i ne segments. The
l i ne segments are cal l ed sides and the end poi nts of l i nes are cal l ed
vertices (pl ural of vertex). Li nes, i nsi de the pol ygon, drawn from one
vertex to the other, are cal l ed diagonals.



The sum of the measures of the
n
angl es i n a pol ygon wi th
n
si des i s
al ways
( 2) 180 n
.

In any quadri l ateral , the sum of the measures of the four angl es i s
360
.

A regular polygon i s a pol ygon i n whi ch al l of the si des are of the same
l ength. In any regul ar pol ygon, the measure of each i nteri or angl e i s
( 2) 180 n
n

and the measure of each exteri or angl e i s
360
n

.
Building Standards in Educational and Professional Testing
National Testing Service Pakistan Overseas Scholarship Scheme for PhD Studies
A parallelogram i s a speci al quadri l ateral , i n whi ch both pai rs of opposi te
si des are paral l el . The Fol l owi ng are some properti es of paral l el ogram.

o Lengths of opposi te si des are equal .
AB CD =
and
AD BC =

o Measures of opposi te angl es are equal .
a c =
and
b d =

o Consecuti ve angl es add up to
180
.
180 a b + =
,
180 b c + =
etc.
o The two di agonal s bi sect each other.
AE EC =
and
BE ED =

o A di agonal di vi des the paral l el ogram i nto two tri angl es that are
congruent.

A rectangle i s a paral l el ogram i n whi ch al l four angl es are ri ght angl es. It
has al l the properti es of a paral l el ogram. In addi ti on i t has the fol l owi ng
properti es:
o The measure of each angl e i n a rectangl e i s
90
.
o The di agonal s of a rectangl e are equal i n l ength.

A square i s a rectangl e that has the fol l owi ng addi ti onal properti es:
o A square has al l i ts si des equal i n l ength.
o In a square, di agonal s are perpendi cul ar to each other.





To cal cul ate the area, the fol l owi ng formul as are requi red:
o For a paral l el ogram,
Area bh =
, where
b
i s the base and
h
i s the
hei ght.
o For a rectangl e,
Area lw =
, where
l
i s the l ength and
w
i s the
wi dth.
o For a square,
2
Area s =
, where
s
i s the si de of the square.

Peri meter for any pol ygon i s the sum of l engths, of al l i ts si des.
Building Standards in Educational and Professional Testing
National Testing Service Pakistan Overseas Scholarship Scheme for PhD Studies
Circles

A circle consi sts of al l the poi nts that are the same di stance from one fi xed
poi nt cal l ed the center. That di stance i s cal l ed the radius of a ci rcl e. The
word radi us i s al so used to represent any of the l i ne segments joi ni ng the
center and a poi nt on the ci rcl e. The pl ural of radi us i s radii.

Any tri angl e, such as
CED
i n the fi gure, formed by connecti ng the end
poi nts of two radi i , i s an i soscel es.

A l i ne segment, such as
ED
i n the di agram above, both of whose end
poi nts are on a ci rcl e i s cal l ed a chord.

A chord that passes through the center of the ci rcl e i s cal l ed the diameter
of the ci rcl e. The l ength of the di ameter i s al ways doubl e the radi us of the
ci rcl e. The di ameter i s the l ongest cord that can be drawn i n a ci rcl e.

The total l ength around a ci rcl e i s known as the circumference of the
ci rcl e.

The rati o of the ci rcumference to the di ameter i s al ways the same for any
ci rcl e. Thi s rati o i s denoted by the symbol

(pronounced as pi ).

2
C
C d C r
d
= = =
where C i s the ci rcumference, d i s the di ameter
and r i s the radi us of the ci rcl e.
Val ue of

i s approxi matel y
3.14


An arc consi sts of two poi nts i n a ci rcl e and al l the poi nts between them.
E.g. PQi s an arc i n the di agram.

An angl e whose vertex i s at the center of the ci rcl e i s cal l ed the central
angle.
PCQ
i n the di agram above i s a central angle.

The degree measure of a compl ete ci rcl e i s
360
.

The degree measure of an arc i s the measure of the central angl e that
i ntercepts i t. E.g. the degree measure of

PQ i s equal to the measure of


PCQ i n the di agram above.
Building Standards in Educational and Professional Testing
National Testing Service Pakistan Overseas Scholarship Scheme for PhD Studies
If
x
i s the degree measure of an arc, i ts l ength can be cal cul ated as
360
x
C
,
where C i s the ci rcumference.
The area of a ci rcl e can be cal cul ated as
2
r
.
The area of a sector formed by the arc and two radii can be
calculated as
A
x
360
, where A is the area of a circle.

2.2 Discrete Quantitative Questions

These are standard multiple-choice questions. Most of such questions require you to do some
computations and you have to choose exactly one of the available choices based upon those
computations. This section will teach you the basic tactics to attempt such questions.

2. 2. 1 Question format
Each question will consist of a question statement and the choices labeled from A to E. The
number of choices may vary from 2 to 5, but exactly one choice will be correct for each
question.

2. 2. 2 How to attempt?

Following are some tactics, which will lead you to the correct answer.

Whenever you know how to answer a questi on di rectl y, just do i t. The
tacti cs shoul d be used onl y when you do not know the exact sol uti on, and
you just want to el i mi nate the choi ces.

Remember that no probl em requi res l engthy or di ffi cul t computati ons. If
you fi nd yoursel f doi ng a l ot of compl ex ari thmeti c, thi nk agai n. You may
be goi ng i n the wrong di recti on.

Whenever there i s a questi on wi th some unknowns (vari abl es), repl ace
them wi th the appropri ate numeri c val ues for ease of cal cul ati on.

When you need to repl ace vari abl es wi th val ues, choose easy-to-use
numbers, e.g. the number 100 i s appropri ate i n most percent-rel ated
probl ems and the LCD (l east common denomi nator) i s best sui ted i n
questi ons that i nvol ve fracti ons.

Appl y back-sol vi ng whenever you know what to do to answer the questi on
but you want to avoi d doi ng al gebra. To understand thi s tacti c read the
fol l owi ng exampl e:

On Monday, a storeowner recei ved a shi pment of books. On Tuesday, she
sol d hal f of them. On Wednesday after two more were sol d, she had
exactl y 2/5 of the books l eft. How many were i n the shi pment?

(A) 10 (B) 20 (C) 30 (D) 40 (E) 50

now by thi s tacti c:

Assume that (A) i s the correct answer, i f so; she must have 3 books on
Wednesday. But 2/5 of 10 are 4, so, (A) i s i ncorrect;

Assume that (B) i s the correct answer, i f so; she must have 8 books on
Wednesday. 2/5 of 20 are 8, so, (B) i s the correct choi ce, and as there may
Building Standards in Educational and Professional Testing
National Testing Service Pakistan Overseas Scholarship Scheme for PhD Studies
be onl y one correct choi ce, there i s no need to check for remai ni ng choi ces.

Thi s tacti c i s very hel pful when a normal al gebrai c sol uti on for the probl em
i nvol ves compl ex or l engthy cal cul ati ons.
If you are not sure how to answer the questi on, do not l eave i t
unanswered. Try to el i mi nate absurd choi ces and guess from the remai ni ng
ones. Most of the ti mes four of the choi ces are absurd and your answer i s
no l onger a guess.

Many thi ngs may hel p you to real i ze that a parti cul ar choi ce i s absurd.
Some of them are l i sted bel ow.

o The answer must be posi ti ve but some of the choi ces are negati ve
so el i mi nate al l the negati ve ones.
o The answer must be even but some of the choi ces are odd so
el i mi nate al l the odd choi ces.
o The answer must be l ess then 100, but some of the choi ces are
greater than 100 (or any other val ue) so el i mi nate al l choi ces that
are out of range.
o The answer must be a whol e number, but some of the choi ces are
fracti ons so el i mi nate al l fracti ons.
o These are some exampl es. There may be numerous si tuati ons
where you can appl y thi s tacti c and fi nd the correct answer even i f
you do not know the ri ght way to sol ve the probl em.

Example questions with solutions

The fol l owi ng are some exampl es, whi ch wi l l hel p you to master these types of
questi ons.

Example
If 25% of 220 equal s 5.5% of X, what i s X?
(A) 10 (B) 55 (C) 100 (D) 110 (E) 1000

Solution:
Si nce 5.5% of X equal s 25% of 220, X i s much greater than 220. So, choi ces
A, B, C, and D are i mmedi atel y el i mi nated because these are not l arger than
220. And the correct answer i s choi ce E.

(Note: An important point here is that, even if you know how to solve a problem, if
you immediately see that four of the five choices are absurd, just pick the remaining
choice and move on.)

Example
Sci ence students choose exactl y one of three fi el ds (i .e. medi cal sci ences,
engi neeri ng sci ences and computer sci ences). If, i n a col l ege, three-fi fths of
the students choose medi cal sci ences, one-forth of the remai ni ng students take
computer sci ences, what percent of the students take engi neeri ng sci ences?

(A) 10 (B) 15 (C) 20 (D) 25 (E) 30

Solution:
The l east common denomi nator of 3/5 and 1/4 i s 20, so assume that there are
20 students i n that col l ege. Then the number of students choosi ng medi cal
sci ences i s 12 (3/4 of 20). Of the remai ni ng 8 students, 2 (1/4 of 8) choose
computer sci ences. The remai ni ng 6 choose engi neeri ng sci ences. As 6 i s
30% of 20, the answer i s E.
Building Standards in Educational and Professional Testing
National Testing Service Pakistan Overseas Scholarship Scheme for PhD Studies

Example
If a school cafeteri a needs C cans of soup each week for each student and
there are S students, for how many weeks wi l l X cans of soup l ast?

(A) CX/S (B) XS/C (C) S/CX (D) X/CS (E) CSX

Solution:
Repl ace C, S and X wi th three easy to use numbers. Let C=2, S=5 and X=20.
Now each student wi l l need 2 cans per week and there are 5 students, so 10
cans are needed per week and 20 cans wi l l l ast for 2 weeks. Now put these
val ues i n choi ces to fi nd the correct one.
The choi ces A, B, C, D and E become 8, 50, 1/8, 2 and 200 respecti vel y. So
the choi ce D represents the correct answer.

2.3 Quantitative Comparison Questions

Some of the questi ons i n the Quanti tati ve secti on of the test may be
quanti tati ve compari son questi ons. The Fol l owi ng text wi l l expl ai n you the
format and techni ques u need to attempt the questi ons of thi s format.

2.3.1 Question format

Such questi ons consi st of two quanti ti es, one i n col umn A and the other i n
col umn B. You have to compare the two quanti ti es. The i nformati on
concerni ng one or both quanti ti es i s presented before them. Onl y the fol l owi ng
four choi ces wi l l be gi ven:

A. The quanti ty i n col umn A i s greater
B. The quanti ty i n col umn B i s greater
C. The two quanti ti es i n both col umns are equal
D. The rel ati onshi p cannot be determi ned from the i nformati on gi ven

And as it is clear from the choices, only one will be correct at one time. Your job is
to choose one of them after careful comparison. The following text explains some
simple tactics to attempt such questions.

2.3.2 How to attempt

Whenever you encounter a quanti tati ve compari son questi on, the fol l owi ng
gui del i nes wi l l hel p you to fi nd the correct answer qui ckl y.

If the questi on i nvol ves some vari abl es, repl ace them wi th appropri ate
numbers. Here are some gui del i nes i n choosi ng an appropri ate number:
o The very best numbers to use are 1, 0 and 1.
o Often fracti ons between 0 and 1 are useful (e.g. 1/2, 3/4 etc.).
o Occasi onal l y, l arge numbers such as 10 or 100 can be used.
o If there i s more than one vari abl e, i t i s permi ssi bl e to repl ace each
wi th the same number.
o Do not i mpose any un-speci fi ed condi ti ons on numbers. Choose
them randoml y.

El i mi nate the choi ces and choose from the remai ni ng ones. For exampl e If
you found the quanti ti es ever equal , the correct choi ce coul d never be A or
B, so, el i mi nate A and B.
A quanti tati ve compari son questi on can be treated as an equati on or
i nequal i ty. Ei ther:
Column A < Column B, or
Column A = Column B, or
Building Standards in Educational and Professional Testing
National Testing Service Pakistan Overseas Scholarship Scheme for PhD Studies
Column A > Column B
So, you can perform si mi l ar operati on on both col umns to si mpl i fy the
probl em just as i n equati ons (or i nequal i ti es).

Example:
m > 0 and m 1




In thi s exampl e di vi de both the quanti ti es by m2. Thi s wi l l change
col umn A to 1 and col umn B to m. Now the compari son i s very si mpl e,
as we know that m i s greater than 0 and cannot be 1. So the
rel ati onshi p i s not determi nabl e usi ng the current i nformati on. m can
be both greater than 1 or between 0 and l ess than 1.
2.3.3 Example questions with Answers and
Explanations
Example 1:
A student earned a 75 on each of her fi rst three
math tests and an 80 on her f ourth and fi fth
tests.
A B
Average after 4 tests Average after 5 tests
A. The quanti ty i n col umn A i s greater
B. The quanti ty i n col umn B i s greater
C. The two quanti ti es i n both col umns are equal
D. The rel ati onshi p cannot be determi ned from the i nformati on gi ven
Remember you want to know whi ch average i s hi gher, not what the averages
are. After 4 tests, the average i s cl earl y l ess than 80, so an 80 on the fi fth
test had to rai se the average. So the answer i s choi ce (B).


Example 2:
A B
The ti me i t takes to
dri ve 40 mi l es at 35
mph
The ti me i t takes to
dri ve 35 mi l es at 40
mph
A. The quanti ty i n col umn A i s greater
B. The quanti ty i n col umn B i s greater
C. The two quanti ti es i n both col umns are equal
D. The rel ati onshi p cannot be determi ned from the i nformati on gi ven

Once agai n there i s no need for cal cul ati on, as the speed i n col umn B i s hi gher
than that i n col umn A. It i s obvi ous that i t wi l l take l ess ti me to travel shorter
di stance at a greater speed. So the val ue i n col umn A i s l arger. The answer i s
opti on (A).

Example 3:
A B
m
2
m
3

Building Standards in Educational and Professional Testing
National Testing Service Pakistan Overseas Scholarship Scheme for PhD Studies
20
2

5
5


A. The quanti ty i n col umn A i s greater
B. The quanti ty i n col umn B i s greater
C. The two quanti ti es i n both col umns are equal
D. The rel ati onshi p cannot be determi ned from the i nformati on gi ven
Square each col umn:
2
20 20
5
2 4

= =



and
2
5 25
5
5 5

= =


. So both col umns are
equal and the answer i s choi ce (C).
Example 4:
A B
13y

15y

To sol ve thi s questi on, subtract
13y
from both col umns to get
13 13 0 y y =
for
col umn A and
15 13 2 y y y =
for col umn B. As there are no restri cti ons,
2y
can
be greater than, l ess than or equal to 0. So the correct choi ce i s (D).

2.4 Data Interpretation Questions

These questi ons are based on the i nformati on that i s presented i n the form of
a graph, chart or tabl e. Most of the data i s presented graphi cal l y. The most
common types of graphs are l i ne graphs, bar graphs and ci rcl e graphs. The
objecti ve of such questi ons i s to test your abi l i ty to understand and anal yze
stati sti cal data.

2.4.1 Question Format
Data i nterpretati on questi ons al ways appear i n sets, you are presented wi th
some data i n any format (chart, graph or tabl e), and you wi l l then be asked
wi th some questi ons about that data.

The fol l owi ng exampl e expl ai ns the format of such questi ons.

Example:

Question 1:
What i s the average sal e, i n mi l l i on Rs., for the peri od 1994-2000?
(A) 5.5 (B) 6.0 (C) 7.0 (D) 8.0
(E) 8.5

Question 2:

For which year, the percentage increase in sales from the previous year is the greatest.
(A) 1995 (B) 1996 (C) 1999 (D) 2000
(E) 2001

2.4.2 How to attempt
Building Standards in Educational and Professional Testing
National Testing Service Pakistan Overseas Scholarship Scheme for PhD Studies
Do not try to answer such questi ons i mmedi atel y, fi rst of al l read the
presented data careful l y. You must be very cl ear about the data and i ts
meani ngs even before readi ng the fi rst questi on.

Do not confuse numbers wi th percents. Thi s confusi on i s most l i kel y to
occur when data i s presented i n pi e graphs. For exampl e i n the
fol l owi ng graph

0
2
4
6
8
10
12
1994 1995 1996 1997 1998 1999 2000 2001
Years
S
a
l
e
s

i
n

m
i
l
l
i
o
n

R
s
.



Now i t woul d be a great mi stake here to thi nk that sal es of TVs & VCRs i s
15% more than the sal es of Computers i n 2001 by XYZ Corporati on. To
know thi s you have to cal cul ate i t as
15
100 60%
25
=
.
Try to avoi d un-necessary cal cul ati ons. Most of the questi ons coul d
easi l y be sol ved by observati on and esti mati on. Use esti mati on to
el i mi nate the choi ces, i f you are not abl e to fi nd the correct answer
wi thout cal cul ati on. For exampl e to sol ve Questi on 1 presented i n the
exampl e at the start of thi s secti on, i f you are not sure of the correct
answer, you can then try to cut down the number of possi bl e choi ces by
observati on. You are bei ng asked to tel l the percentage i ncrease.
Where as, i n year 2000, the sal e i s decreasi ng i nstead of i ncreasi ng, so,
you can i mmedi atel y el i mi nate choi ce (D) i n that questi on.
Building Standards in Educational and Professional Testing
National Testing Service Pakistan Overseas Scholarship Scheme for PhD Studies
Your answers must be based upon the i nformati on presented i n the
gi ven charts and graphs. If your knowl edge contradi cts any of the data
presented, i gnore what you know and sti ck to the presented data. The
presented data shoul d be the onl y base for your cal cul ati ons and
esti mati ons.
Al ways use the proper uni ts, there may be some questi ons that ask you
to compare di fferent data i tems possi bl y from di fferent data sets. Be
careful about the uni ts used to represent the data.
Because graphs and charts present data i n a form that enabl es you to
readi l y see the rel ati onshi ps among val ues and to make qui ck
compari sons, you shoul d al ways try to vi sual i ze you answer i n the same
format as the ori gi nal data was presented.
Be sure that your answer i s reasonabl e. For exampl e, the profi t coul d
never i ncrease the actual sal es, or the expenses coul d never be negati ve
etc. Whi l e answeri ng the questi on, fi rst of al l el i mi nate such un-
reasonabl e choi ces, and then choose from the remai ni ng ones.

2.5 Practice exercise
1 What i s the average of posi ti ve i ntegers from 1 to 100 i ncl usi ve?

(A) 49
(B) 49.5
(C) 50
(D) 50.5
(E) 51

2 If
6 x y + =
,
7 y z + =
, and
9 x z + =
, what i s the average of
x
,
y
and
z
?
(A)
11
3
(B)
11
2
(C)
22
3

(D)
11
(E)
22


3 In the di agram bel ow, l i nes l and m are not paral l el .



If A represents the average measure of al l the ei ght angl es, what i s the val ue
of A?
(A)
45 A =

(B)
45 90 A < <

(C)
90 A =

(D)
90 180 A < <

(E)
180 A =


4 Asl am has 4 ti mes as many books as Sal man and 5 ti mes as many as Javed.
If Javed has more than 40 books, what i s the l east number of books that
Asl am coul d have?

(A) 200 (B) 205 (C) 210 (D) 220 (E) 24
Building Standards in Educational and Professional Testing
National Testing Service Pakistan Overseas Scholarship Scheme for PhD Studies
5 Asl am i s now 3 ti mes as ol d as Javed, but 5 years ago, he was 5 ti mes as
Javed was. How ol d i s Asl am now?

(A) 10 (B) 12 (C) 24 (D) 30 (E) 36
6 If
% x
of
y
i s 10, what i s y?
(A)
x
10

(B)
x
100

(C)
x
1000

(D)
100
x

(E)
10
x


Answer Key

1 D
2 A
3 A
4 D
5 D
6 C

Building Standards in Educational and Professional Testing
National Testing Service Pakistan Overseas Scholarship Scheme for PhD Studies
3 Analytical Ability

3.1.1 Question format

Each anal yti cal reasoni ng questi on i s a l ogi cal puzzl e, based on a gi ven set of
condi ti ons. Li ke mathemati cal questi ons, these questi ons have exactl y one
correct answer, whi ch i s what you need to sel ect.

Anal yti cal reasoni ng questi ons are presented i n groups of four or fi ve
questi ons. Each group i s based on a short passage fol l owed by a set of
condi ti ons. Occasi onal l y, there are graphs and tabl es i nstead of a passage.
To understand the general format of the questi on, consi der the fol l owi ng
exampl e.

Question 1-4:
As part of thei r sports physi cal , seven col l ege athl etes F, G, H, I, J, K and L
are bei ng wei ghed. In announci ng the resul ts of the physi cal exams, the
coach has gi ven the fol l owi ng i nformati on.
i . None of the athl etes i s exactl y the same wei ght as another athl ete.
i i . K i s heavi er than L, but l i ghter than H.
i i i . I i s heavi er than J
i v. Both F and G are heavi er than H.
1 Each of the fol l owi ng coul d be true EXCEPT
A. F i s the heavi est.
B. G i s the heavi est.
C. I i s the heavi est.
D. More than three athl etes are heavi er than K.
E. More than three athl etes are l i ghter than K.
2 Whi ch of the fol l owi ng, i f true, woul d be suffi ci ent to determi ne whi ch
athl ete i s the l i ghtest?
A. I i s the heavi est
B. I i s l i ghter than K
C. K i s heavi er than J
D. J i s heavi er than K
E. Exactl y fi ve students are l i ghter than F.

3 If J i s heavi er than F, how many di fferent ranki ngs by wei ght, of the athl etes
are possi bl e?
A. 1 B. 2 C. 3 D. 4 E. 5
4 If H i s heavi er than I, whi ch of the fol l owi ng CANNOT be true?
A. I s wei ght i s equal to the average of F s wei ght and G s wei ght.
B. I s wei ght i s equal to the average of K s wei ght and L s wei ght
C. J s wei ght i s equal to the average of K s wei ght and L s wei ght
D. J i s the second l i ghtest.
Answers:

1. E 2. D 3. C 4. A
3.1.2 How to attempt
Si mpl i fy the i nformati on by usi ng abbrevi ati ons and symbol s. The fi rst
step i s to stri p away al l of the excess verbi age from the gi ven passage
and then to abbrevi ate the remai ni ng key words wi th si ngl e l etters. For
exampl e, i n the questi on statement fi ve musi ci ans a bassi st, a
drummer, a gui tari st, a pi ani st, and a trumpeter are performi ng i n a
tal ent show, you shoul d i mmedi atel y abbrevi ate them B, D, G, P and T.
Building Standards in Educational and Professional Testing
National Testing Service Pakistan Overseas Scholarship Scheme for PhD Studies
You can use abbrevi ated l etters to represent a whol e sentence al so. You
shoul d use symbol s to represent condi ti ons. You may devel op your own
symbol i c conventi ons for thi s. The obj ecti ve i s to convert the probl em
i nto notati ons, so that, i t i s easi l y understandabl e. The fol l owi ng i s a
basi c set of symbol s, whi ch are most commonl y used.
A represents the statement Akbar i s goi ng.
B represents the statement Babur i s goi ng.

Symbo
l
Meaning Examples
~ Not ~A Akbar i s not goi ng. Or you can say,
i t i s not the case that Akbar i s
goi ng.


And A

B Akbar and Babur are goi ng.


A

~B Akbar i s goi ng and Babur i s not


goi ng.


Or A

B Akbar or Babur i s goi ng.


A

~B Akbar i s goi ng or Babur i s not goi ng.


If, then
A

B
If Akbar i s goi ng then Babur i s goi ng.
(A

B)

S
If Akbar and Babur are goi ng, then
Sal eem i s goi ng.


If and onl y i f
A

B
Babur i s goi ng, i f and onl y i f Akbar i s
goi ng.
Before l earni ng the tacti cs to attempt an anal yti cal reasoni ng questi on,
you must be fami l i ar wi th some basi c l ogi c facts, whi ch are expl ai ned i n
the fol l owi ng text. Consi der A and B are two statements.
o A i s true means ~A i s fal se.
o ~A i s true means A i s fal se.
o (A

B) i s true means both A and B are true.


o (A

B) i s true means ei ther A or B or both are fal se.


o (A

B) i s true means ei ther A or B or both are true.


o (A

B) i s fal se means both A and B are fal se.


o ~(A

B) i s equi val ent to (~A

~B).
o ~(A

B) i s equi val ent to (~A



~B).
o If (A

B) i s true then
If A i s true B i s al so true.
If A i s fal se B may be true or fal se.
o If (A

B) i s fal se then A i s true and B i s fal se.


o (A

B) i s equi val ent to (~B

~A)
o (A

B) i s true means:
If A i s true B i s true.
If A i s fal se B i s fal se.
o (A

B) i s fal se means:
If A i s true B i s fal se.
If A i s fal se B i s true.
o (A

B) i s equi val ent to [(A

B)

(B

A)].

You must be fami l i ar wi th the most common types of anal yti cal
reasoni ng questi ons. The fol l owi ng four types occur more frequentl y
than the others, and when you see them, you shoul d i mmedi atel y know
what you need to do to answer them.
o Which of the following could be true? If onl y one of the
answer choi ces coul d be true, then each of the other four choi ces
must be fal se; that i s, each one must vi ol ate at l east one of the
gi ven condi ti ons.

Building Standards in Educational and Professional Testing
National Testing Service Pakistan Overseas Scholarship Scheme for PhD Studies
o Which of the following must be true? Si nce onl y one of the
answer choi ces must be true, then for each of the choi ces, ei ther
i t i s fal se or i t i s possi bl y (but not defi ni tel y) true. You have to
choose onl y that choi ce whi ch i s defi ni tel y true.

o Which of the following cannot be true? Si nce onl y one of the
answer choi ces cannot be true, then each of the other choi ces
coul d be true. The correct answer i s the onl y choi ce, whi ch
vi ol ates at l east one of the gi ven condi ti ons or i s otherwi se
i nconsi stent wi th what you know must be true.
o How many possibilities are there? Thi s questi on asks, How
many di fferent ways are there to sati sfy al l of the gi ven
condi ti ons? Here, you must systemati cal l y count or l i st al l of the
possi bi l i ti es that do not vi ol ate any of the condi ti ons.

Identi fy the key words that serve to l i mi t the si tuati on. Certai n words
are cri ti cal to your understandi ng of the si tuati on. Be sure to
i ncorporate your symbol s. Some frequentl y used key words are l i sted
bel ow:

After Al l Al ways At l east At most
Before But Can be Cannot be Consecuti ve
Di fferent Di rectl y Each
No fewer
than
No more than
Onl y Possi bl e Enti re Every Exactl y
Except Fewer Fi rst If If and onl y i f
Immedi atel y Impossi bl e Last Least Most
Must be Same Some The Least The Most
Unl ess Smal l est Greatest None

Note that certai n key words have onl y one functi on, to rul e out a
potenti al ambi gui ty.

El i mi nati ng the choi ces i s al ways a good strategy. Whi l e el i mi nati ng the
choi ces, fi rst of al l , el i mi nate those whi ch are rul ed out by i ndi vi dual
condi ti ons: Then work through the remai ni ng choi ces.

Study condi ti ons, not merel y for what they state but al so for what they
i mpl y. Certai n anal yti cal reasoni ng questi ons resembl e the i nference
questi ons you fi nd i n the readi ng comprehensi on secti on. To answer
them correctl y, you must understand not onl y what the condi ti ons state
expl i ci tl y, but al so what they i mpl y.

Often the key to answeri ng anal yti cal reasoni ng questi ons i s to organi ze
the gi ven i nformati on i n a l i st or tabl e.
On some anal yti cal reasoni ng questi ons, an excel l ent way to deal wi th
the i nformati on i s to draw a si mpl e di agram, pi cture, or map. Thi s i s
parti cul arl y hel pful when you are deal i ng wi th the physi cal or temporal
order of thi ngs. It i s much easi er to tel l whether person A can be
seated opposi te person B i f you have sketched a di agram of the tabl e; i t
i s easi er to know whether person C i s ol der or younger than person D i f
you have entered al l of the gi ven i nformati on on a ti me l i ne; and i t i s
easi er to determi ne whether town E i s east of town W i f you have drawn
a si mpl e map.
Building Standards in Educational and Professional Testing
National Testing Service Pakistan Overseas Scholarship Scheme for PhD Studies

3.1.3 Example questions with Answers and
Explanations
Questions 1-5:
Si x actors ---- Bob, Carol , Dave Ed, Frank, and Grace audi ti on for a part i n an
off-Broadway pl ay. The audi ti ons wi l l take pl ace over four consecuti ve days,
starti ng on a Thursday. Each actor wi l l have one audi ti on; the days on whi ch
the di fferent actors wi l l audi ti on must conform to the fol l owi ng condi ti ons.

i . At l east one audi ti on wi l l take pl ace each day.
i i . No more than two audi ti ons wi l l take pl ace on any day.
i i i . No more than three audi ti ons wi l l take pl ace on any two
consecuti ve days.
i v. Bob s audi ti on must take pl ace on Saturday.
v. Carol s audi ti on must take pl ace on the same day as another
audi ti on.
vi . Frank s audi ti ons must take pl ace on the day bef ore Grace s
audi ti on.
vi i . Dave s audi ti on must take pl ace on a day after Ed s audi ti on.

1 If onl y one audi ti on takes pl ace on Thursday whi ch actor coul d have that
audi ti on?
(A) Bob (B) Carol (C) Dave (D) Frank (E) Grace

2 If Bob s and Frank s audi ti ons are on the same day, whi ch of the fol l owi ng
must be true
(A) Dave s audi ti on wi l l take pl ace on Thursday
(B) Dave s audi ti on wi l l take pl ace on Fri day
(C) Grace s audi ti on wi l l take pl ace on Thursday
(D) Carol s audi ti on wi l l take pl ace on Sunday
(E) Ed s audi ti on wi l l take pl ace on Sunday

3 If the di rector deci des to hol d two audi ti ons on Thursday and two on
Sunday, how many actors woul d be el i gi bl e to audi ti on on Fri day?
(A) 1 (B) 2 (C) 3 (D) 4 (E) 5

4 If Ed and Grace have thei r audi ti ons on the same day whi ch of the fol l owi ng
must be true?
(A) Ed s audi ti on wi l l take pl ace on Thursday.
(B) Frank s audi ti on wi l l take pl ace on Fri day.
(C) Carol s audi ti on wi l l take pl ace on Saturday.
(D) Grace s audi ti on wi l l take pl ace on Saturday.
(E) Carol s audi ti on wi l l take pl ace on Sunday.

5 If Ed s audi ti on i s on Saturday, whi ch of the fol l owi ng actors cannot
audi ti on on the same day as any other actor?
(A) Bob
(B) Carol
(C) Ed
(D) Frank
(E) Gr
ace
Questions 6-10:
Duri ng the fi rst hal f of the year, from January through June, the chai rperson of
the mathemati cs department wi l l be on sabbati cal . The dean of the col l ege has
asked each of the si x professors i n the department --- Arkes, Borofsky, Chang,
Denture, Hobbes, and Lee--- to serve as acti ng chai rperson duri ng one of
Building Standards in Educational and Professional Testing
National Testing Service Pakistan Overseas Scholarship Scheme for PhD Studies
those months. The mathemati ci ans can deci de the order i n whi ch they wi l l
serve, subject onl y to the fol l owi ng cri teri a establ i shed by the dean.
i . Chang wi l l serve as chai rperson i n February.
i i . Arkes wi l l serve as chai rperson before Hobbes does.
i i i . Borofsky and Dexter wi l l serve as chai rpersons i n consecuti ve
months.

6 Whi ch of the fol l owi ng professors coul d serve as chai rperson i n January?

(A) Borodfsky (B) Chang (C) Dexter (D) Hobbes (E) Lee

7 In how many ways can the schedul e be made up i f Lee has to serve as
chai rperson i n May?
(A) 1
(B) 2
(C) 3
(D) 4
(E) 6

8 If Lee serves i n Apri l , al l of the fol l owi ng coul d be true EXCEPT
(A) Arkes serves i n January
(B) Hobbes serves i n march
(C) Borofsky serves i n may
(D) Borofsky serves i n June
(E) Hobbes serves i n June
Building Standards in Educational and Professional Testing
National Testing Service Pakistan Overseas Scholarship Scheme for PhD Studies
9 If Borofsky serves i n May, what i s the l atest month i n whi ch Arkes coul d
serve?
(A) January
(B) February
(C) March
(D) Apri l
(E) June

10 Whi ch of the fol l owi ng CANNOT be true?
(A) Arkes and Lee serve i n consecuti ve months.
(B) Lee and Hobbes serve i n consecuti ve months.
(C) Hobbes and Dexter serve i n consecuti ve months.
(D) Arkes and Chang serve i n consecuti ve months.
(E) Borofsky and Chang serve i n consecuti ve months.

Solutions 1-5:
Fi rst express each of the condi ti ons symbol i cal l y:
B, C, D, E, F, and G: 1 audi ti on each
Days: Thu, Fri , Sat, Sun
Each day: 1 or 2 audi ti ons
2 consecuti ve days: 2 or 3 audi ti ons
B=Sat Cx F<G E<D
1 A vi ol ates the condi ti on that Bob s audi ti on wi l l take pl ace on Saturday
(B=Sat). B vi ol ates the condi ti on that Carol s audi ti on cannot be the onl y
audi ti on on a parti cul ar day (Cx). Choi ces C and E are i mpossi bl e. Si nce
Dave s audi ti on must take pl ace on a day af ter Ed s audi ti on (E<D) and Grace s
audi ti on must take pl ace on a day after Frank s audi ti on (F<G) nei ther can
take pl ace on Thursday. Onl y choi ce D does not vi ol ate any of the gi ven
condi ti ons, so thi s i s the correct answer.

2 The condi ti on that Bob s and Frank s audi ti ons are on the same day compl etel y
determi nes the schedul e. They must take pl ace on Saturday (B=Sat). To
avoi d havi ng more than three audi ti ons on two consecuti ve days, there can be
onl y one audi ti on on Fri day and one on Sunday, whi ch means there wi l l be two
on Thursday. Si nce Frank must have to precede Grace (F<G), Grace s audi ti on
wi l l take pl ace on Sunday. Si nce Ed must precede Dave, Ed s audi ti on wi l l take
pl ace on Thursday and Dave s audi ti on on Fri day. Fi nal l y, Carol s audi ti on wi l l
be the second audi ti on on Thursday. The fi nal schedul e i s C and E on
Thursday, D on Fri day, B and F on Saturday and G on Sunday. Onl y choi ce B
i s consi stent wi th thi s schedul e, so B i s the correct choi ce.
3 Si nce onl y one audi ti on can take pl ace on Fri day, i t cannot be Carol s (Cx);
and, of course, i t cannot be Bob s (B = Sat). Any of the other four actors
coul d audi ti on on Fri day as i ndi cated i n the fol l owi ng schedul es:
E/F on Thu, D on Fri , B on Sat, C/G on Sun
C/F on Thu, E on Fri , B on Sat, D/G on Sun
C/E on Thu, E on Fri , B on Sat, D/G on Sun
E/F on Thu, G on Fri , B on Sat, C/D on Sun
So the correct choi ce i s D.

4 The onl y schedul e that ful fi l s the condi ti ons i s F on Thu, E/G on Fri , B on Sat,
and C/D on Sun. Onl y choi ce E i s consi stent wi th thi s schedul e.

5 Si nce Ed and Bob s audi ti ons are both taki ng pl ace on Saturday, el i mi nate
choi ces A and C. Si nce Carol e must audi ti on on the same day as another
actor, el i mi nate B. Fi nal l y, si nce Dave s audi ti on must take pl ace on Sunday
(E < D), Frank s audi ti on must take pl ace on Thursday and Grace s audi ti on on
Fri day (F < G). El i mi nate choi ce D. The compl ete schedul e i s: C/F on Thu, G
on Fri , B/F on Sat, and D on Sun.
Solutions 6-10:
Let A, B, C, D, H, L represents professor names.
C=February, A<H, B<<D and D<<B

Building Standards in Educational and Professional Testing
National Testing Service Pakistan Overseas Scholarship Scheme for PhD Studies
6 Onl y choi ce E i s there, whi ch does not vi ol ate any of the condi ti ons, so i s the
correct choi ce.

7 Wi th C servi ng i n February and L i n May, the onl y consecuti ve months
avai l abl e for B and D are March and Apri l . Then si nce A must serve i s before H
i n June. There are two possi bl e schedul es, dependi ng on the order of B and D,
so the correct choi ce i s B.

8 If L serves i n Apri l , the consecuti ve months avai l abl e for B and D are May and
June; so choi ces C and D coul d be true. Si nce A must serve before H, choi ces
A and B must be true, onl y choi ce E cannot be true.

9 Si nce A must serve before H does, A cannot serve i n June. Can A serve i n
Apri l ? No, because then, D woul d serve i n June (B<<D or D<<B), and agai n A
woul d not precede H. The l atest that A coul d serve i n March, whi ch coul d
occur i n the fi nal order: L, C, A, D, B and H.

10 The onl y professors that can serve i n January are A and L, so, one of them
must serve i n January, and nei ther serves i n February. So choi ce A cannot be
true.

3.2 Logical Reasoning

Each l ogi cal reasoni ng questi on requi res you to anal yze an argument presented i n
a short passage. Often you are asked ei ther to fi nd a concl usi on that i s a l ogi cal
consequence of the passage, or to choose a statement that, i f true, strengthen or
weakens the argument.

3.2.1 Question format

Logi cal reasoni ng questi ons are based upon a passage cal l ed argument. You have
to anal yze the argument presented i n the passage. The passage i s fol l owed by a
questi on. Occasi onal l y, there i s more than one questi on rel ated be the same
passage. No matter what the number i s, the questi ons al ways ai m at your abi l i ty
to understand the concl usi on reached by the author of the passage, and to gi ve
argument and contra arguments. Logi cal reasoni ng questi ons are a l ot l i ke readi ng
comprehensi on questi ons i n a verbal secti on.

For each l ogi cal reasoni ng questi on, the argument i s fol l owed by a mul ti choi ce
questi on. The choi ces are si mpl e statements. Mostl y the questi on statement
begi ns wi th the phrase whi ch of the fol l owi ng statements. Here are a few
exampl es:
Whi ch of the fol l owi ng statements i s an assumpti on on whi ch the concl usi on
of thi s argument i s based?

Whi ch of the fol l owi ng statements i denti fi es a fl aw i n the reasoni ng of thi s
argument?

Whi ch of the fol l owi ng statements can be most reasonabl y i nferred, from
the statements i n the gi ven passage?

Whi ch of the fol l owi ng statements, i f true, woul d most seri ousl y, weaken
the argument offered?

Whi ch of the fol l owi ng statements, i f true, woul d strengthen the concl usi on
i n the precedi ng argument?

Whi ch of the fol l owi ng statements woul d be the most i mportant to know to
eval uate the argument gi ven i n the precedi ng paragraph? Every l ogi cal
reasoni ng questi on does not fi t thi s mol d, but you shoul d try.


Building Standards in Educational and Professional Testing
National Testing Service Pakistan Overseas Scholarship Scheme for PhD Studies
3.2.2 How to attempt

Whi l e attempti ng l ogi cal reasoni ng questi ons, you shoul d read the questi on
statement before readi ng the argument. Then you shoul d anal yze the
argument presented i n the passage. You must know what aspect of the
argument you are to concentrate on, and f ocus on i t. By thi s, you wi l l not
be unnecessari l y wasti ng your ti me.
You must be abl e to spot the questi on type by readi ng the questi on
statement. If you do thi s, you wi l l be better abl e to approach the
argument i n hand. The fol l owi ng si x categori es are those whi ch most
commonl y occur:

1 Assumption: Questi ons that test your abi l i ty to recogni ze the premi ses on
whi ch an argument i s based, often take the fol l owi ng forms:
o The concl usi on above depends on whi ch of the fol l owi ng
assumpti ons?
o The author of the passage above makes whi ch of the fol l owi ng
assumpti ons?
o In the passage above, the author assumes whi ch of the fol l owi ng
statement to be true?

2 Inference: Questi ons, whi ch test your abi l i ty to go beyond the author s
expl i ci t statements and see what these statements i mpl y, may be worded l i ke
these.
o It can be i nferred from the passage above that the author
bel i eves that
o Whi ch of the fol l owi ng i s i mpl i ed by the passage above?
o From the i nformati on above, whi ch of the fol l owi ng i s the most
reasonabl e i nference?

3 Conclusion: Questi ons that test your abi l i ty to determi ne what cl ai m can
l ogi cal l y be made on the basi s of evi dence i n the passage above?
o If the statements above are true, whi ch of the fol l owi ng i n a
concl usi on that can be properl y drawn?
o The statements i n the passage, i f true, best supports whi ch of the
fol l owi ng concl usi ons?

4 Central Point: Questi ons that test your abi l i ty to understand the
thrust of an argument.
o The statement si ted above conveys whi ch of the fol l owi ng
proposi ti ons?
o The author of the passage above argues that
o Whi ch of the fol l owi ng expresses the poi nt the author of the
passage above makes?

5 Support: Questi ons that test your abi l i ty to recogni ze whether an
asserti on supports or undermi nes an argument.
o Whi ch of the fol l owi ng, i f true, best supports the author s
concl usi on?
o Whi ch of the fol l owi ng, i f true, most weakens the author s
concl usi on?

6 Argument Evaluation: Questions that test your ability to judge
an argument.
o Whi ch of the f ol l owi ng i denti fi es a f l aw i n the speaker s
reasoni ng?
o Whi ch of the fol l owi ng woul d be most i mportant to know when
eval uati ng the accuracy of the argument above?

Building Standards in Educational and Professional Testing
National Testing Service Pakistan Overseas Scholarship Scheme for PhD Studies
Do not try to ski m the passage, read each argument careful l y. It i s not
enough to have a general i dea about the argument; you must be abl e to
anal yze i t very careful l y.

You must fi nd the concl usi on of the argument, whi ch the author cl ai ms to
have reached. That most common si tuati ons are as fol l ows:
o The concl usi on i s the l ast sentence of the passage, often starti ng
by words such as so, therefore, thus, hence, consequentl y etc.
o The concl usi on i s the fi rst sentence of the passage fol l owed by
the supporti ng evi dence.
o Occasi onal l y, the concl usi on i s not present i n the passage; i n thi s
case, the questi on asks you to i denti fy the concl usi on.

Pay parti cul ar attenti on to si gnal words such as accordi ngl y, for thi s
reason, hence, al though, but, except, i n contrast, neverthel ess, unl i ke etc.

El i mi nati ng the choi ces i s al ways the best strategy i f you do not know what
the correct answer i s. Thi s process wi l l el i mi nate some obvi ous wrong
choi ces. And you wi l l be abl e to make an educated guess from the
remai ni ng ones.

Every argument i s based upon certai n assumpti ons made by the author. If
an argument s basi c premi ses are sound, the argument i s strengthened. If
an argument s basi c premi ses are fl awed, the argument i s weakened. In
support questi ons, where you have to deci de about weakeni ng or
strengtheni ng the questi on, pi npoi nt what the argument assumes. Then
compare that assumpti on wi th the answer choi ces. If the questi on asks you
to fi nd the choi ce, whi ch most strengthens the argument, l ook for the
choi ce that i s most i n keepi ng wi th the argument s basi c assumpti on. If
the questi on asks you to choose the choi ce that most weakens the
argument, l ook for the answer that casts the most doubt on that
assumpti on.

Some l ogi cal reasoni ng questi ons are essenti al l y mi ni anal yti cal reasoni ng
questi ons, so, be fami l i ar wi th al l of the i mportant l ogi cal facts and appl y
whenever needed. Exampl e questi ons wi th Answers and Expl anati ons

3.2.3 Example Questions with Answers and Explanations

Questions 1-2:
The mi crowave oven has become a standard appl i ance i n many ki tchens, mai nl y
because i t offers a fast way of cooki ng food. Yet, some homeowners bel i eve that
the ovens are sti l l not compl etel y safe. Mi crowaves, therefore, shoul d not be a
standard appl i ance unti l they have been careful l y researched and tested.

1 Whi ch of the fol l owi ng, i f true, woul d most weaken the concl usi on of the
passage above?

(A) Homeowners, often purchase i tems despi te knowi ng they may be unsafe.
(B) Those homeowners i n doubt about mi crowave safety ought not to purchase
mi crowaves.
(C) Research and testi ng of home appl i ances sel dom reveal s safety hazards.
(D) Mi crowaves are not as dangerous as steam i rons, whi ch are used i n al most
every home.
(E) Homeowners often purchase i tems that they do not need.

2 Whi ch one of the fol l owi ng, i f true, woul d most strengthen the concl usi on of
the passage above?

(A) Homeowners often doubt the adverti sed safety of al l new appl i ances.
(B) Speed of food preparati on i s not the onl y concern of today s homeowner.
Building Standards in Educational and Professional Testing
National Testing Service Pakistan Overseas Scholarship Scheme for PhD Studies
(C) Modern homeowners have more free ti me than ever before.
(D) Food preparati on has become al most a sci ence, wi th more compl i cated and
i nvol ved reci pes.
(E) Many mi crowave ovens have been found to l eak radi oacti ve el ements.

3 Years ago, a pol l concl uded that there are more tel evi si ons than there are
bathtubs i n Ameri can homes. No doubt that fact remai ns today, especi al l y i n
l i ght of the growi ng popul ari ty of home computers. Now, i n addi ti on to owni ng
tel evi si ons for entertai nment, more and more fami l i es are purchasi ng TV
moni tors for use wi th a personal computer. We can safel y guess that there are
sti l l many more peopl e stari ng at a pi cture tube than si ngi ng i n the shower.
Whi ch of the fol l owi ng statements can be i nferred from thi s passage?
(A) Personal computers probabl y cost l ess than i nstal l i ng a shower or bathtub.
(B) Peopl e can wash themsel ves wi thout a tub or shower, but they cannot
watch tel evi si on unl ess they own a tel evi si on set.
(C) TV moni tors wi l l work wi th personal computers i n pl ace of regul ar
computer moni tors.
(D) As many computers are sol d today as tel evi si on sets a few years ago.
(E) More tel evi si on moni tors are now used wi th personal computers than are
used to watch commerci al tel evi si on broadcasts.

4 Some sci enti sts have proposed that, over two hundred mi l l i on years ago, one
gi ant l and mass, rather than vari ous conti nents and i sl ands, covers one thi rd
of the earth. Long before there was any human l i fe, and over vast peri ods of
ti me, i sl ands and conti nents dri fted apart. Austral i a was the fi rst to separate,
whi l e South Ameri ca and Afri ca were l ate i n spl i tti ng apart. Some i sl ands, of
course, were formed by vol canoes and were never part of the great l and mass.

Al l the fol l owi ng woul d support the author s cl ai m EXCEPT

(A) Many of the pl ants of the South Ameri can rai n forests are markedl y si mi l ar
to those of Afri can rai n forests.
(B) Austral i a has more ani mal s that are not found i n any other conti nent than
have several of the much l arger conti nents.
(C) Vol cani c i sl ands l i ke Hawai i have ecosystems very di fferent from those of
conti nental l ands wi th the same average temperature.
(D) The pl ants of si mi l ar condi ti ons i n South Ameri ca have l ess i n common wi th
those of Austral i a than wi th those of Asi a, Afri ca or Europe.
(E) The pri mi ti ve l anguages of Austral i a are unl i ke those of Afri ca, whi ch
resembl es those of South Ameri ca.
5 Every Saturday, Ami r has pi zza for l unch and then goes to the movi es.
If the statement above i s true, whi ch of the fol l owi ng statements must al so be
true?
1 If i t i s not Saturday, than Ami r i s not havi ng pi zza for l unch and i s not
goi ng to the movi es.
2 If Ami r has pi zza for l unch and then goes to the movi es, i t i s Saturday.
3 If Ami r has pi zza for l unch, but does not go to the movi es, i t i s not a
Saturday.

(A) 1 onl y
(B) 2 onl y
(C) 3 onl y
(D) 1 and 2 onl y
(E) 2 and 3 onl y

6 Anti freeze l owers the mel ti ng poi nt of any l i qui d to whi ch i t i s added so that
the l i qui d wi l l not freeze i n col d weather. It i s commonl y used to mai ntai n the
cool i ng system i n automobi l e radi ators. Of course, the weather may become
so col d that even anti freeze i s not effecti ve, but such a severe cl i mati c
condi ti on rarel y occurs i n wel l -travel ed pl aces.
Building Standards in Educational and Professional Testing
National Testing Service Pakistan Overseas Scholarship Scheme for PhD Studies

Whi ch of the fol l owi ng can be deduced from the passage?
(A) Wel l -travel ed pl aces have means of transportati on other than automobi l es.
(B) Anti freeze does not l ower the mel ti ng poi nt of certai n l i qui ds i n extreme
condi ti ons.
(C) Severe cl i mati c condi ti ons rarel y occur.
(D) It i s not often that many travel ers who use anti freeze have thei r cool i ng
system freeze.
(E) Anti freeze rai ses the mel ti ng poi nt of some l i qui ds.
Solutions:
1 The concl usi on of the passage i s that, because of safety concerns, more
research and testi ng ought to be done before mi crowaves become standard
househol d appl i ances. If, however, research and testi ng are i neffecti ve means
of di scerni ng safety probl ems (as choi ce C says), then research and testi ng
woul d be i rrel evant. Thi s cri ti ci sm seri ousl y weakens the concl usi on. So
choi ce C i s the correct answer.

2 If many mi crowave ovens have been found to l eak radi oacti ve el ements (as
choi ce E says), then the concl usi on that mi crowaves shoul d not be standard
appl i ances unti l they are more careful l y researched and tested i s further
strengthened because more safety concerns need to be addressed. So, choi ce
E i s the correct answer.

3 Though Choi ces A and B may wel l be true, they cannot be i nferred from the
i nformati on i n the passage. But choi ce C can be i nferred si nce, more and
more fami l i es are purchasi ng TV moni tors for use wi th a personal computer.
TV moni tors must work wi th these computers, otherwi se, peopl e woul d not buy
them for that purpose. Choi ces D and E may or may not true, but they are not
i nferences from the passage, si mpl y addi ti onal i nformati on. So, the correct
choi ce i s C.

4 If Austral i a was the fi rst conti nent to separate, i t woul d fol l ow that i ts fl ora
and fauna woul d devel op i n i sol ati on over a l onger peri od of ti me. Si mi l arl y,
we may expect the pl ants and ani mal of South Ameri ca and Afri ca that
separated l ater, to be more al i ke. Choi ces A, B, and D support these i deas.
The separatel y devel oped i sl ands are di fferent at i s al so i n accord wi th the
passage. However the l anguages of al l the conti nents woul d have devel oped i n
i sol ati on, si nce man di d not evol ve unti l after the break-up of the l andmass,
and i t i s surpri si ng that Afri can and South Ameri can l anguages are si mi l ar.
Human l i keness or di fferences are i rrel evant to the cl ai ms of the passage. So
choi ce E i s the correct answer.

5 Thi s l ogi cal reasoni ng questi on i s very easy as soon as you express the gi ven
statement symbol i cal l y. If i t i s Saturday, then Ami r has Pi zza and goes to
Movi es transl ates as
) ( M P S
. Thi s i s equi val ent to
S M P ~ ) ( ~
,
whi ch i s equi val ent to
S M P ~ ) ~ (~
. So i f ei ther P or M i s fal se, then S i s
fal se. Therefore, 3 i s true, nei ther 1 nor 2 are true. So, the correct choi ce i s
C.

6 Choi ce D i s the correct answer. Si nce severe cl i mati c condi ti ons rarel y occur
i n wel l -travel ed pl aces, i t i s not necessari l y true that It i s not often that
many travel ers who use anti freeze have thei r cool i ng systems freeze. Choi ce
A menti ons other means of transportati on, whi ch i s not addressed i n the
passage. Choi ce B refers to certai n l i qui ds.
Building Standards in Educational and Professional Testing
National Testing Service Pakistan Overseas Scholarship Scheme for PhD Studies

4 Verbal Ability
The purpose of the Verbal Test i s to eval uate and anal yze your Engl i sh
comprehensi on and understandi ng towards the l anguage. The questi ons wi l l be
basi cal l y asked to judge the sentence compl eti on, anal ogy and cri ti cal readi ng
ski l l s. The questi ons of di fferent types i .e. about sentence compl eti on and
anal ogy testi ng wi l l be asked randoml y. The questi ons about the cri ti cal readi ng
however wi l l be asked separatel y.

4.1 About the Verbal Questions
As al ready di scussed, thi s secti on wi l l consi st of the fol l owi ng types of questi ons
i .e. sentence compl eti on, anal ogy and the cri ti cal readi ng. The detai l about each
secti on i s as bel ow;

4.1.1 Sentence Completion
The questi ons that come under thi s category are provi ded wi th vari ous choi ces.
You are asked to compl ete the sentences by fi l l i ng i n the bl anks wi th the most
sui tabl e choi ce.
The questi ons for sentence compl eti on can be rel ated to any of the other areas of
study i .e. sci ence, l i terature, hi story, geography etc but the subject matter woul d
not hi nder your l anguage abi l i ti es. You are asked to compl ete the sentence wi th
the use of correct grammar or vocabul ary.

These questi ons try to determi ne your abi l i ty to recogni ze the correct sentence
structure, ri ght grammar and how you make the correct choi ce of vocabul ary.

Techniques for Sentence Completion

For the sentence compl eti on a few choi ces are gi ven that coul d be sel ected for
compl eti ng the sentences. Onl y one choi ce i s correct out of the several choi ces.
You have to compl ete the sentence by sel ecti ng the correct choi ce accordi ng to
the grammar or vocabul ary. For maki ng the ri ght choi ce you can benefi t from the
fol l owi ng techni ques;

After you read the i ncompl ete sentence do not l ook at the choi ces. Try to
thi nk about the correct answer yoursel f. If you thi nk that you have
compl eted the sentence and found the correct choi ce you can consul t your
l i st of choi ces. If the answer you thought matches one of the choi ces
menti oned i n the l i st that i s most probabl y the ri ght choi ce to be marked.
If i t does not match wi th the choi ce you can l ook for a synonym
repl acement. Thi s tacti c i s very hel pful i n fi ndi ng the ri ght answer, i t
prevents you from confusi ng yoursel f wi th the wrong choi ces.

Do not sel ect the choi ce hasti l y. Even i f you are sati sfi ed wi th your choi ce
try to substi tute i t wi th the other choi ces so that you are more sati sfi ed
wi th your deci si on. Someti mes the other choi ce fi ts more appropri atel y to
the sentence.

When you are asked to compl ete a sentence, whi ch has two spaces to be
fi l l ed i n, try to put the fi rst word of every choi ce i n the fi rst bl ank. Note
down the choi ce that you fi nd best. Now for the second bl ank try every
second choi ce of al l choi ces. Note the choi ce that you thi nk i s most
appropri ate. Check i f the two sel ected choi ces are matchi ng one of the
gi ven pai r of choi ces. If i t does then sel ect i t as your correct choi ce, i f not
then consi der thi s pai r as a wrong choi ce and try wi th the other choi ces.

If you fi nd di ffi cul ty i n maki ng sense out of certai n words and you are not
very fami l i ar wi th them you can try to make a guess wi th reference to the
Building Standards in Educational and Professional Testing
National Testing Service Pakistan Overseas Scholarship Scheme for PhD Studies
context of the sentence. Try to break the word i nto vari ous parts and
anal yze i ts meani ng e.g. i f you do not know the meani ng of the word
ci vi l i zati on break i t i nto two i .e. ci vi l i ze and ati on now you may know
the meani ng of ci vi l i ze and through the term ati on you can make out that
the word i s a noun of ci vi l i ze. If you fi nd the word unfami l i ar wi th prefi xes
and suffi xes di vi de the word i nto i ts parts e.g. prerecordi ng. Thi s word
consi sts of both prefi x and suffi x. You can break the word l i ke pre-record-
i ng. Here you know that pre means before, record means to store and -i ng
i s a term of conti nuous tense. So you can fi nd thi s break up of words qui te
hel pful i n maki ng out the ri ght sense. If none of the techni que works try
maki ng a guess wi th reference to the context.

When l ong and compl ex sentences confuse you then try to break that
sentence i nto smal l er more sentences by rephrasi ng i t. After you di vi de i t
compare wi th the ori gi nal sentence to avoi d any mi si nterpretati on. If you
are sati sfi ed read the smal l er sentences to get the i dea more cl earl y.

Example Questions

1. Mul tan ___________ a very hot cl i mate.
A. Has
B. Have
C. Has been
D. Wi th
2. One of the l east effecti ve ways of sorti ng i nformati on i s l earni ng
_________ i t.
A. Repeat
B. Repeati ng
C. To repeat
D. how repeat

3. Sal man fi ni shed__________ two of hi s publ i shed composi ti ons before
hi s twel fth bi rthday.
A. Wri tten
B. Wri ti ng
C. To wri te
D. Wrote

4. Sofi a __________ col l ect stamps, but now she has other i nterests.
A. Used to
B. Was used to
C. Used to be
D. Usi ng to

5. After passi ng through a great trauma of her husband s death, she
__________ hard to achi eve mental rel axati on.
A. Struggl ed
B. Struggl i ng
C. Struggl e
D. To struggl e

6. In partnershi p wi th Paki stan, South Korea ____________on Motor way.
A. Hel ped worked
B. Hel pi ng work
C. Hel ped worki ng
D. To hel p worki ng

7. We wi l l wai t i f you __________ go.
A. Wanted to
B. Want
C. Want to
Building Standards in Educational and Professional Testing
National Testing Service Pakistan Overseas Scholarship Scheme for PhD Studies
D. Wanti ng to

8. If I had more ti me I ____________ checked my paper.
A. Woul d have
B. Woul d
C. Woul d had
D. Wi l l have

9. I thought that he___________ comi ng today.
A. Has been
B. Is
C. Was
D. Has

10. That professor enjoys teachi ng and ___________.
A. Wri ti ng
B. Wri tten
C. To wri te
D. Wri te

11. Just __________the f i l es on my tabl e.
A. Let
B. Leaves
C. Stay
D. Leave

12. Thank you for __________ me your book.
A. Borrowi ng
B. Lendi ng
C. Borrowed
D. Had l ent

13. ____________ di scovery of i nsul i n, i t was not possi bl e to treat
di abetes.
A. Pri or
B. Before to the
C. Pri or to the
D. To pri or the

14. Di stri bute the handouts ___________ the candi dates.
A. Between
B. Among
C. To
D. In

15. Onl y _________ were present at the semi nar.
A. a few peopl e
B. a l i ttl e peopl e
C. a few peopl es
D. the l i ttl e peopl e

Answer Key

1. A
2. C
3. B
4. A
5. A
6. C
7. C
8. A
9. C
10. A
11. D
12. B
13. C
14. B
15. A
Building Standards in Educational and Professional Testing
National Testing Service Pakistan Overseas Scholarship Scheme for PhD Studies

4.1.2 Analogy Questions

Anal ogy means si mi l ari ty i n exampl es or descri bi ng l i keness between two or more
words. These questi ons ask the reader to anal yze the rel ati onshi p between two
words i n a pai r and l ook for another si mi l ar or equi val ent pai r of words. You are
provi ded wi th fi ve other pai rs of words. You are expected to match the ori gi nal
pai r, whi ch i s gi ven i n the questi on wi th one of the pai rs i n the gi ven choi ces on
the bases of si mi l ar rel ati onshi ps between them. Thi s exerci se or such questi ons
try to determi ne your basi c understandi ng towards vocabul ary and your abi l i ty to
recogni ze the rel ati onshi p between words. Some questi ons may al so ask you to
sel ect a sui tabl e antonym for a gi ven word.

Techniques for Analogy Questions
For the anal ogy questi ons you can fol l ow the gui del i nes menti oned bel ow;

Do not read the choi ces before you have anal yzed the rel ati onshi p between
the pai r of words, yoursel f. Try to understand the words more
appropri atel y and thi nk on whi ch basi s the rel ati onshi p between the words
i s formed. After you reach a concl usi on read the gi ven choi ces afterwards
to get a proper match wi th another pai r havi ng the same rel ati onshi p.

When you f i nd yoursel f stuck wi th a word of di ffi cul t vocabul ary, do not f eel
confused. Try to understand i ts meani ng reference to the context or i f i t i s
somewhat fami l i ar try to remember where and when you heard the word
before. It can be a great hel p.

Someti mes you fi nd that there i s more than one pai r that fi ts wel l to the
questi on and i s appropri ate for the choi ce, gi ve the ori gi nal pai r a l i ttl e
more thought so that you can further study the rel ati onshi p between the
words and narrow i t down to a more di sti nct one. After you have been
successful i n fi ndi ng a cl oser rel ati onshi p you can now scruti ni ze the two
other pai rs that confused you earl i er. Repeati ng the same procedure wi th
these words woul d prove useful .
Do not get caught up by the tri cks of the test makers. Someti mes the
questi ons are provi ded wi th very tri cky and dodgi ng choi ces that mi sgui de
greatl y. Try to thi nk of every choi ce more speci fi cal l y and narrowl y.

If you are fami l i ar wi th the parts of speech and thei r nature, i t can be
benefi ci al i n maki ng a more sensi bl e choi ce. Remember i f the words i n the
ori gi nal pai r are a noun and an adjecti ve, the correct choi ce you make
shoul d al so contai n the words i n the same grammati cal order. Otherwi se,
your choi ce i s wrong. So, i f you are conf used wi th two pai rs and cannot
choose the correct choi ce you can easi l y l ook at thei r grammati cal order
and gi ve preference to the one, whi ch matches the ori gi nal one.

Excl ude the choi ce from your consi derati on that you thi nk i s
i ncorrect, e.g. the choi ces that do not have the same grammati cal
uni t as of the ori gi nal pai r cannot match the ori gi nal pai r i n anyway.
Spend more ti me on consi deri ng the more possi bl e choi ces.

You shoul d know about the vari ous ki nds of anal ogi es that are more
frequentl y asked. Some of the common anal ogy types are as
fol l ows;

i . Synonyms
Some words are l i nked together i n a pai r whi ch means the same or
has a si mi l ar di cti onary defi ni ti on.e.g Pretty- Beauti ful

Building Standards in Educational and Professional Testing
National Testing Service Pakistan Overseas Scholarship Scheme for PhD Studies
i i . Describing Qualities
Some pai rs have some words i n whi ch one word descri bes the other
word. Heavy- Rai n

i i i . Class and Member
Some pai rs have words whi ch are based on cl ass and member basi s
e.g. El ectroni cs-Radi o

i v. Antonyms
Some pai rs consi st of the words that are opposi te to each other
e.g.
Love- Hate

v. Describing Intensity
Some pai rs consi st of the words i n whi ch one descri bes the
i ntensi ty of the other e.g.
Smi l e-Laughter

vi . Function
In some pai rs a word descri bes the functi on of the other word e.g.
Pen-Wri te

vi i . Manners
Some words i n a speech descri be the manners and behavi or e.g.
Pol i te-Speech

vi i i . Worker-Workplace
Some pai rs i n a word descri be the professi on and i ts workpl ace e.g.
Doctor-Cl i ni c

Example Questions

1. HEIGHT: MOUNTAIN

(A) Depth : Trench
(B) Shade : Tree
(C) Wei ght : Age
(D) Speed : Hi ghway
(E) Mi neral : Mi ne

2. OBLIVIOUS : AWARENESS

(A) Comatose : Consci ousness
(B) Serene : Composure
(C) Erudite : Knowl edge
(D) Adroi t : Ski l l
(F) Invi gorate : Energy

3. BELLWETHER : BAROMETER

(A) Prosel yte : Spark pl ug
(B) Panhandl er : Ki l l
(C) Embezzl er : Abduct
(D) Cynosure : Magnet
(F) Morass : Catal yst
Building Standards in Educational and Professional Testing
National Testing Service Pakistan Overseas Scholarship Scheme for PhD Studies

4. ACT : ACTION

(A) Therapy : Thermometer
(B) Obl i vi on : Obvi ous
(C) Li turgy : Li terature
(D) Image : Imagi ne
(E) Bowl : Bowdl eri ze

5. BIBULOUS : DRINK

(A) Rapaci ous : Cl othi ng
(B) Gl uttonous : Food
(C) Al trui sti c : Money
(D) Vegetari an : Meat
(E) Controversy : Reconci l e

6. SONG : RECITAL

(A) Author : Bi bl i ography
(B) Epi sode : Seri es
(C) Coach : Team
(D) Dancer : Agi l e
(E) Poetry : Prose

7. HOUSE : BIG

(A) Home : Li ve
(B) School : Dai l y
(C) Water : Col d
(D) Cl othes : Socks



8. ANIMAL : MONKEY

(A) Zebra : Gi raffe
(B) Stati onery: Penci l
(C) Book : Cap
(D) Tree : Wood

9. HEAVEY : LIGHT

(A) Fat : Thi n
(B) Stupi d : Idi ot
(C) Rough : Surface
(D) Beauti ful : Di ary

Choose the lettered word or phrase that is most nearly opposite in
meaning to the word in capital letters.

10. DETER

(A) Twi st
(B) Inti mi date
Building Standards in Educational and Professional Testing
National Testing Service Pakistan Overseas Scholarship Scheme for PhD Studies
(C) Encourage
(D) Straighten
(E) Suppl y

11. INDIGENOUS

(A) Modern
(B) Al i en
(C) Ornamental
(D) Natural
(E) Conceal

12. THERAPEUTIC

(A) Curati ve
(B) Injuri ous
(C) Medi ci nal
(D) Practi cal
(E) Sl ander

13. QUIZZICAL

(A) Amused
(B) Unequi vocal
(C) Contorted
(D) Di ssi mi l ar
(E) Lax

14. ANCILLARY

(A) Dependent
(B) Concomi tant
(C) Appendi x
(D) Pri mary
(E) Buffet

15. VIRTUOSO

(A) Wi cked
(B) Dazzl i ng
(C) Medi ocre
(D) Honorabl e
(E) Bawdy


Answer Key

1. A 4. D 7. C 10. C 13. B
2. A 5. B 8. B 11. B 14. D
3. A 6. B 9. A 12. B 15. E

4.1.3 Critical Reading Questions

Building Standards in Educational and Professional Testing
National Testing Service Pakistan Overseas Scholarship Scheme for PhD Studies
Questi ons rel ated to cri ti cal readi ng try to judge your readi ng ski l l s and how you
understand and i nterpret what you read. The paper i ncl udes a few passages that
ask answeri ng questi ons rel ated to the passage.


Techniques for Critical Reading Exercises

There are a few techni ques rel ated to the Cri ti cal Readi ng Questi ons that
prove to be a good gui del i ne for sol vi ng such questi ons.

Do not read the questi ons before readi ng the whol e passage. Try to
ski m through the whol e passage and then read the questi ons to l ook
for a more speci fi c answer. Read the passage qui ckl y wi th
understandi ng but do not pani c. Try to anal yze what the whol e
passage i s about and what the author real l y i ntends to convey.
Whi l e readi ng mark the l i nes where you thi nk the passage carri es the
most i mportant poi nts. These strategi es woul d defi ni tel y hel p you
fi nd the answers.

When you fi nd yoursel f stuck wi th a questi on, do not waste your ti me
on i t and go ahead for the next questi ons. Someti mes, answeri ng
other questi ons guide you about the earl i er questi on. But, i f you sti l l
do not fi nd the answer mark i t for doi ng i n the end more cal ml y,
havi ng enough ti me to thi nk.

Try to fami l i ari ze yoursel f wi th the types of cri ti cal readi ng
questi ons. Once you know the nature of such questi ons, you wi l l be
abl e to fi nd the answers more qui ckl y even when you are readi ng the
passage. The exampl es of some commonl y asked questi ons are as
fol l ows:

o Central Idea
Mostl y, questi ons are asked to expl ai n the central i dea or mai n
theme of the whol e passage, whi ch anal yzes how you ski m
through i t. Someti mes, the openi ng and cl osi ng l i nes can gi ve you
a better cl ue about answeri ng such questi ons properl y.

o Specific Details
Someti mes to anal yze your scanni ng abi l i ti es you are asked to
answer some speci fi c detai l s about the passage. Such questi ons
are about when , where , whi ch and who . You can get the
answers of thi s ki nd of questi ons from the area of the passage
whi ch you marked i n the fi rst readi ng, where you thi nk the most
i mportant and i nformati onal remarks of the author l i es.

o Making Inferences
Most of the questi ons ask you to i nfer from the passages, maki ng
your opi ni on about what i s sai d i n the paragraph, impl yi ng
meani ng and maki ng your own poi nt of vi ew. These questi ons try
to assess your judgment; you must be cl ear i n your mi nd about
what the author i s referri ng to and then make your own opi ni on
accordi ng to your understandi ng and comprehensi on. Read and
thi nk about al l the choi ces and anal yze each of i t l ogi cal l y
accordi ng to your comprehensi on rather than the author s poi nt of
vi ew.
Building Standards in Educational and Professional Testing
National Testing Service Pakistan Overseas Scholarship Scheme for PhD Studies

o Meaning in Context
Some sel ected words from the passage are poi nted out to expl ai n
them wi th reference to the context to check your readi ng
comprehensi on. Someti mes the word that descri bes somethi ng i n
a di cti onary portrays i t the other way when i t appears i n the
context. The test tri es to judge your abi l i ty to make sense of the
word i n the context.

o Authors Approach
Some questi ons ask you to expl ai n the mood i n whi ch the author
i s wri ti ng whether i t i s sarcasti c, humorous, wi tty, sad etc. When
you are asked questi ons l i ke these you can l ook for certai n
expressi ons, words, phrases or excl amati ons, whi ch descri be the
tone, mood or styl e of the author. The feel i ngs of the wri ter are
mostl y exhi bi ted through choi ce of words. Whi l e answeri ng these
questi ons read the message careful l y observi ng parti cul arl y the
use of words.

o Title Selection
Some passages ask for sel ecti ng a ti tl e that best sui ts the
passage. Remember that the chosen ti tl e shoul d not be narrowl y
or broadl y sel ected. Try to avoi d choosi ng those ti tl es that
descri bes onl y one or two paragraphs but the one, whi ch i s
appl i cabl e to the whol e passage and portrays i t best.
Example Questions

Passage I:
We are profoundl y i gnorant about the ori gi ns of l anguage and have to
content oursel ves wi th more or l ess pl ausi bl e specul ati ons. We do not
even know for certai n when l anguage arose, but i t seems l i kel y that i t goes
back to the earl i est hi story of man, perhaps hal f a mi l l i on years. We have
no di rect evi dence, but i t seems probabl e that speech arose at the same
ti me as tool maki ng and the earl i est forms of speci fi cal l y human
cooperati on. In the great Ice Ages of the Pl ei stocene peri od, our earl i est
human ancestors establ i shed the Ol d Stone Age cul ture; they made fl i nt
tool s and l ater tool s of bone, i vory, and antl er; they made fi re and cooked
thei r food; they hunted bi g game, often by methods that cal l ed for
consi derabl e cooperati on and coordi nati on. As thei r materi al cul ture
gradual l y i mproved, they became arti sts and made carvi ngs and engravi ngs
on bones and pebbl es, and wonderful pai nti ngs of ani mal s on the wal l s of
caves. It i s di ffi cul t to bel i eve that the makers of these Pal eol i thi c cul tures
l acked the power of speech. It i s a l ong step Admi ttedl y, from the earl i est
fl i nt weapons to the spl endi d art of the l ate Ol d Stone Age: the fi rst crude
fl i nts date back perhaps to 500,000 B.C., whi l e the fi nest achi evements of
Ol d Stone Age man are l ater than 100,000 B.C.; and, i n thi s peri od, we can
envi sage a correspondi ng devel opment of l anguage, from the most
pri mi ti ve and l i mi ted l anguage of the earl i est human groups to a ful l y
devel oped l anguage i n the fl oweri ng ti me of Ol d Stone Age cul ture.

How di d l anguage ari se i n the fi rst pl ace? There are many theori es about
thi s, based on vari ous types of i ndi rect evi dence, such as the l anguage of
chi l dren, the l anguage of pri mi ti ve soci eti es, the ki nds of changes that
have taken pl ace i n l anguages i n the course of recorded hi story, the
Building Standards in Educational and Professional Testing
National Testing Service Pakistan Overseas Scholarship Scheme for PhD Studies
behavi or of hi gher ani mal s l i ke chi mpanzees, and the behavi or of peopl e
sufferi ng from speech defects. These types of evi dence may provi de us
wi th useful poi nters, but they al l suffer from l i mi tati ons, and must be
treated wi th cauti on. When we consi der the l anguage of chi l dren, we have
to remember that thei r si tuati ons are qui te di fferent from that of our
earl i est human ancestors, because the chi l d i s growi ng up i n an
envi ronment where there i s al ready a ful l y devel oped l anguage, and i s
surrounded by adul ts who use that l anguage and are teachi ng i t to hi m.
For exampl e, i t has been shown that the earl i est words used by chi l dren
are mai nl y the names of thi ngs and peopl e (Dol l , Spoon, Mummy):
but, thi s does not prove that the earl i est words of pri mi ti ve man were al so
the names of thi ngs and peopl e. When the chi l d l earns the name of an
object, he may then use i t to express hi s wi shes or demands: Dol l !: often
means Gi ve me my dol l ! Or I ve dropped my dol l : pi ck i t up for me!;
the chi l d i s usi ng l anguage to get thi ngs done, and i t i s al most an acci dent
of adul t teachi ng that the words used to formul ate the chi l d s demands are
mai nl y nouns, i nstead of words l i ke Bri ng! Pick up!; and so on.

1 The main idea of this excerpt is
A. to provi de evi dence of the ori gi n of l anguage.
B. to present the need for l anguage.
C. to di scuss how earl y man communi cated.
D. to present the cul ture of earl y man.
E. to narrate the story of Engl i sh.
F.
2 Theories of the origin of language include all of the following
EXCEPT
A. Changes occurri ng through the years.
B. The need to communi cate.
C. Language of chi l dren.
D. The fi rst man s extensi ve vocabul ary.
E. Communi cati on among pri mi ti ve men.

3 The purpose of the discussion of the word, Doll, is intended to
A. Trace the evol uti on of a noun.
B. Support the fact that nami ng thi ngs i s most i mportant.
C. Indi cate how adul ts teach l anguage to chi l dren.
D. Show the evol uti on of many meani ngs for one word.
E. Evi nce man s mul ti pl e uses of si ngl e words

4 The implication of the author regarding the early elements of
language is that
A. There were speci fi c real steps fol l owed to devel op our l anguage.
B. Care must be exerci sed when exhumi ng what we consi der the roots
of l anguage.
C. We owe a debt of grati tude to the chi mpanzee contri buti on.
D. Adul ts created l anguage i n order to i nstruct thei r chi l dren.
E. Language was ful l y devel oped by pri mi ti ve man.

5 If we accept that primitive man existed for a very long period of
time without language, then we may assume that
A. Language i s not necessary to man s existence.
B. Language devel oped wi th the devel opi ng cul ture of pri mi ti ves.
C. Pri mi ti ves exi sted i n total i sol ati on from one another.
Building Standards in Educational and Professional Testing
National Testing Service Pakistan Overseas Scholarship Scheme for PhD Studies
D. Chi l dren brought about a need for l anguage.
E. Manki nd was not i ntended to communi cate.

6 After a reading of this article, one might infer that
A. Soci ety creates probl ems wi th l anguage.
B. Language i s for adul ts to i nstruct chi l dren.
C. Soci ety uses l anguage to i mprove i tsel f.
D. Wi th the evol uti on of l anguage came wi sdom.
E. Language bri ngs power.

Answer Key

1. A 2. D 3.C 4. B 5. B 6. E

Building Standards in Educational and Professional Testing
National Testing Service Pakistan Overseas Scholarship Scheme for PhD Studies
COMMON QUERIES


Q. How to take NTS Online Test?

You will go through the following screen shots to learn how to take NTS Online test.

Step: 1


This is the first screen named as Candidate Login Screen. You will enter your Candidate ID and
Password provided to you by NTS



Building Standards in Educational and Professional Testing
National Testing Service Pakistan Overseas Scholarship Scheme for PhD Studies
Step: 2
You will see Test Instructions Screen after you successfully login. Please read the instructions
carefully to avoid any confusion during the test. After reading the instructions, press Start Test
button on center bottom of the page.


Building Standards in Educational and Professional Testing
National Testing Service Pakistan Overseas Scholarship Scheme for PhD Studies
Step: 3
After you click on the Start Test button your test starts and a page appears which shows your first
question of the test. Each question has various choices, if you know the answer you can select the
correct choice for your answer and press Next Question button. You can also add this question
in the Pass Box to answer it at some another stage. You will also find some additional information
about your test on this page. This information includes:

Total Number of Questions in the Test
Total Number of Questions Answered by you
Total Number of Questions in the Pass box to be attempted later



Building Standards in Educational and Professional Testing
National Testing Service Pakistan Overseas Scholarship Scheme for PhD Studies
Step: 4
If you place a question in Pass Box you will notice that the Questions in Pass Box field will
increase by one.




Building Standards in Educational and Professional Testing
National Testing Service Pakistan Overseas Scholarship Scheme for PhD Studies
Step: 5
If you want to answer the questions in Pass Box, simply click on Questions in Pass Box link at
the top. It will take you on the following screen. Now click Answer this Question button for the
question you want to answer.




Building Standards in Educational and Professional Testing
National Testing Service Pakistan Overseas Scholarship Scheme for PhD Studies
Step: 6
You have selected this question from Pass Box now select its answer and proceed for next
question.


Building Standards in Educational and Professional Testing
National Testing Service Pakistan Overseas Scholarship Scheme for PhD Studies
Step: 7
You have attempted all questions and now this last screen will show your detailed result.


Wish You Good Luck with your Test.
National Testing Service



Q. How is NTS Online Test different from paper-based Test?

For the Paper-based Test,

A pencil, eraser and a sharpener is required to attempt the paper-based test.
You can make the changes in the answer that you have already marked.
The announcement of results is delayed than the online tests.
You have to fill in a separate answer sheet to mark your answers.

Where as for the Online Test,

No stationery items are required to attempt online test.
If you want to attempt a question at the end, you can place it in the Pass Box for
attempting later.
You can not reattempt a question that you have already answered.
As soon as you finish the test, the result is displayed on the screen.
In case of a system failure during the test, you will have to log-in again and the
test will start from the same question where you had left. No information will be
lost while the system was unavailable.

Building Standards in Educational and Professional Testing
National Testing Service Pakistan Overseas Scholarship Scheme for PhD Studies
Q. How Can I Ask for Result Reporting?

Your result is reported to you right after you finish your test. You are given the
certificate fifteen days after the conduct of the test of the last batch. If you still do not
get the result you can Contact Us. You can also visit the website of NTS to check your
result. The final result is sent to you by email.

Q. Is There Any Negative Marking While Scoring?

There is no negative marking for wrong answers. However the negative marking may be
activated if it is required by the allied institute or organization.


Q. What are the Rules and Regulations that apply to me in the Test Center
when taking the Online General Test?

Rules and Regulations:
If you do not appear with the Identity Card (NIC) on the Test Center, you will not be
allowed to take the test.
The test will be given on the day and at the scheduled time. You are asked to observe
punctuality. Arriving late at the center may disqualify you from taking the test.
You are not allowed to bring any testing aids inside the test center. Nothing, except
the original Identity Card is required to be taken along.
You are not allowed to smoke, eat or drink inside the test center.
No discussion or any form of communication with the fellow candidates is allowed
during the testing session.

You will also not be allowed to leave the test center without the permission of the
supervisor.
Test centers do not have large waiting areas. Friends or relatives who accompany you
to the test center will not be permitted to wait in the test center or contact you while
you are taking the test.
You will be required to sign the attendance sheet before and after the test session and
any time you leave or enter the premises where the test is being conducted.
If you need to leave your seat at any time during the test (which shall only be allowed
in case of serious illness), raise your hand and ask the invigilator.
Repeated unscheduled breaks will be documented and reported to NTS. NTS reserves
the right to videotape all or any of Testing Sessions and use it to determine any
misconduct, etc.
If at any time during the test you believe that you have a problem with your test, or
need the Invigilation Staff for any reason, raise your hand to notify the Invigilation Staff.
Building Standards in Educational and Professional Testing
National Testing Service Pakistan Overseas Scholarship Scheme for PhD Studies
NOTE: The rest of the queries regarding the test format, contents and other
procedures have almost the same answers as of the paper based tests, given
above.
Building Standards in Educational and Professional Testing
National Testing Service Pakistan Overseas Scholarship Scheme for PhD Studies









DRILL TESTS
General
















Note: The sample papers do not include quantitatively the same number
of questions as there would be in the actual papers. They are merely
meant to provide conceptual guidance to the users or prospective
candidates.
Building Standards in Educational and Professional Testing
National Testing Service Pakistan Overseas Scholarship Scheme for PhD Studies









Drill Test I
Building Standards in Educational and Professional Testing
National Testing Service Pakistan Overseas Scholarship Scheme for PhD Studies


1. The number of degrees through which the hour hand of a clock moves in 2 hours and
12 minutes is

A. 66
B. 72
C. 732
D. 723
E. None of these



2. A cylindrical container has a diameter of 14 inches and a height of 6 inches. Since
one gallon equals 231 cubic inches, the capacity of the tank is approximately

A. 2-2/7 gallons
B. 4 gallons
C. 1-1/7 gallons
D. 2-2/7 gallons
E. None of these



3. A train running between two towns arrives at its destination 10 minutes late when it
goes 40 miles per hour and 16 minutes late when it goes 30 miles per hour. The
distance between the two towns is

A. 720 miles
B. 12 miles
C. 8-6/7 miles
D. 12-7/7 miles
E. None of these


4. If the base of a rectangle is increased by 30% and the altitude is decreased by 20%
the area is increased by

A. 25%
B. 10%
C. 5%
D. 1%
E. 4%


5. If the sum of the edges of a cube is 48 inches, the volume of the cube is

A. 512 inches
B. 96 cubic inches
C. 64 cubic inches

I
Quantitative
Section
No. Of
Questions
10
Choose the correct answer for each
question and shade the
corresponding CIRCLE on the
answer sheet
A B C D E
A B C D E
A B C D E
A B C D E
Building Standards in Educational and Professional Testing
National Testing Service Pakistan Overseas Scholarship Scheme for PhD Studies
D. 698 cubic inches
E. None of these




6. A certain triangle has sides, which are, respectively, 6 inches, 8 inches, and 10
inches long. A rectangle equal in area to that of the triangle has a width of 3 inches. The
Perimeter of the rectangle, expressed in inches, is

A. 11
B. 16
C. 22
D. 23
E. 24



7. The cube of 1/3 is

A. 3/9
B. 3/27
C. 1/81
D. 1/27
E. 1/9




8. In general, the sum of the squares of two numbers is greater than twice the product
of the numbers. The pair of numbers for which this generalization is not valid is

A. 8,9
B. 9,9
C. 9,10
D. 9,8
E. 8,10



9. A piece of wire 132 inches long is bent successively in the shape of an equilateral
triangle, a square, a regular hexagon, a circle. The plane surface of the largest area is
included when the wire is bent into the shape of a

A. Circle
B. Square
C. Hexagon
D. Triangle
E. Line



10. If pencils are bought at 35 cents per dozen and sold at 3 for 10 cents the total
profit on 5 1/2 dozen is

A B C D E
A B C D E
A B C D E
A B C D E
A B C D E
Building Standards in Educational and Professional Testing
National Testing Service Pakistan Overseas Scholarship Scheme for PhD Studies
A. 25 cents
B. 35 cents
C. 27 1/2 cents
D. 28 1/2 cents
E. 31 1/2 cents

0


A B C D E
Building Standards in Educational and Professional Testing
National Testing Service Pakistan Overseas Scholarship Scheme for PhD Studies



For question 1 to 4

Three adultsR, S, and Vwill be traveling in a van with five childrenF, H, J, L, and M.
The van has a driver's seat and one passenger seat in the front, and two benches behind
the front seats, one bench behind the first. Each bench has room for exactly three
people. Everyone must sit in a seat or on a bench, and seating is subject to the
following restrictions: An adult must sit on each bench. Either R or S must sit in the
driver's seat. J must sit immediately beside M.

1. Which of the following can sit in the front passenger seat?

A: J
B: L
C: R
D: S
E: V




2. Which of the following groups of three can sit together on a bench?

A: F, J, and M
B: F, J, and V
C: F, S, and V
D: H, L, and S
E: L, M, and R



3. If F sits immediately beside V, which of the following CANNOT be true?

A: J sits immediately beside S.
B: L sits immediately beside V.
C: H sits in the front passenger seat.
D: F sits on the same bench as H.
E: H sits on the same bench as R.



4. If S sits on a bench that is behind where J is sitting, which of the following must be
true?

A: H sits in a seat or on a bench that is in front of where M is sitting.
B: L sits in a seat or on a bench that is in front of where F is sitting.
C: F sits on the same bench as H.
D: L sits on the same bench as S
II
ANALYTICAL
Section
No. Of
Questions
20
Choose the correct answer for
each question and shade the
corresponding CIRCLE in the
answer sheet
A B C D E
A B C D E
A B C D E
Building Standards in Educational and Professional Testing
National Testing Service Pakistan Overseas Scholarship Scheme for PhD Studies
E: M sits on the same bench as V.



For question 5 to 9
The principal of a school is forming a committee. There are to be five members: three
teachers, chosen from Mr. J, Ms. K, Ms. L, Mr. M, and Mr. N; and two students, chosen
from O, P, Q, and R. The composition of the committee must conform to the following
conditions: Ms. J will serve only if R is also on the committee. Ms. L will not serve
unless Ms. K and O also serve. Neither Mr. M nor Mr. N will serve without the other. If
P serves, neither Q nor R can serve.

5. Which of the following is an acceptable committee?

A: J, L, M, N, O
B: K, L, N, O, P
C: K, M, N, O, R
D: L, M, N, O, R
E: M, N, O, P, Q



6. How many different committees could include Mr. J and Q?

A: 1
B: 2
C: 3
D: 4
E: 5



7. If Q and R are both on the committee, who else must be on the committee?

A: J
B: K
C: L
D: M
E: O



8. If M is not on the committee, each of the following must be on the committee EXCEPT

A: J
B: L
C: O
D: Q
E: R


9. In how many different ways can the principal select an acceptable committee?

A: Fewer than 3
B: 3
A B C D E
A B C D E
A B C D E
A B C D E
A B C D E
Building Standards in Educational and Professional Testing
National Testing Service Pakistan Overseas Scholarship Scheme for PhD Studies
C: 5
D: 7
E: More than 7



For question 10 to 13

A contractor will build five houses in a certain town on a street that currently has no
houses on it. The contractor will select from seven different models of housesT, U, V,
W, X, Y, and Z. The town's planning board has placed the following restrictions on the
contractor: No model can, be selected for more than one house. Either model W must
be selected or model Z must be selected, but both cannot be selected. If model Y is
selected, then model V must also be selected. If model U is selected, then model W
cannot be selected.


10. If model U is one of the models selected for the street, then which of the
following models must also be selected?

A: T
B: W
C: X
D: Y
E: Z




11. If T, U, and X are three of the models selected for the street, then which of the
following must be the other two models selected?

A: V and W
B: V and Y
C: V and Z
D: W and Y
E: Y and Z



A B C D E
A B C D E
A B C D E
Building Standards in Educational and Professional Testing
National Testing Service Pakistan Overseas Scholarship Scheme for PhD Studies

12. Which of the following is an acceptable combination of models that can be
selected for the street?

A: T, U, V, X, Y
B: T, U, X, Y, Z
C: T, V, X, Y, Z
D: U, V, W. X, Y
E: V, W, X, Y, Z



13. If model Z is one model not selected for the street, then the other model NOT
selected must be which of the following?

A: T
B: U
C: V
D: W
E: X




For question 14 to 16

Seven childrenF, J, K, M, R, S, and Tare eligible to enter a spelling contest. From
these seven, two teams must be formed, a red team and a green team, each team
consisting of exactly three of the children. No child can be selected for more than one
team. Team selection is subject to the following restrictions: If M is on the red team, K
must be selected for the green team. If F is on the red team, R, if selected, must be on
the green team. R cannot be on the same team as S. J cannot be on the same team as
K.

14. Which of the following can be the three members of the Red team?

A: F, J, and K
B: F, R, and T
C: J, K, and T
D: K, M, and R
E: M, R, and T



15. If M and F are both on the red team, the green team can consist of which of the
following?

A: J, K, and R
B: J, S, and T
C: K, R, and S
D: K, R, and T
E: R, S, and T



A B C D E
A B C D E
A B C D E
A B C D E
Building Standards in Educational and Professional Testing
National Testing Service Pakistan Overseas Scholarship Scheme for PhD Studies
16. If M is on the red team, which of the following, if selected, must also be on the
red team?

A: F
B: J
C: R
D: S
E: T



For question 17 to 20

A mail carrier must deliver mail by making a stop at each of six buildings: K, L, M, O, P,
and S. Mail to be delivered are of two types, ordinary mail and priority mail. The
delivery of both types of mail is subject to the following conditions: Regardless of the
type of mail to be delivered, mail to P and mail to S must be delivered before mail to M
is delivered. Regardless of the type of mail to be delivered, mail to L and mail to K must
be delivered before mail to S is delivered. Mail to buildings receiving some priority mail
must be delivered, as far as the above conditions permit, before mail to buildings
receiving only ordinary mail.

17. If K is the only building receiving priority mail, which of the following lists the
buildings in an order, from first through sixth, in which they can receive their mail?

A: L, K, P, S, O, M
B: L, K, S, P, M, O
C: K, L, P, M, S, O
D: K, P, L, S, O, M
E: O, K, L, P, S, M



18. If L, M, and S are each receiving priority mail, which of the following lists the
buildings in an order, from first to sixth, in which they must receive their mail?

A: K, L, P, S, O, M
B: L, K, O, P, S, M
C: L, K, S, P, M, O
D: M, L, S, P, K, O
E: S, L, M, P, K, O



A B C D E
A B C D E
A B C D E
Building Standards in Educational and Professional Testing
National Testing Service Pakistan Overseas Scholarship Scheme for PhD Studies

19. If the sequence of buildings to which mail is delivered is O, P, L, K, S, M and if S
is receiving priority mail, which of the following is a complete and accurate list of
buildings that must also be receiving priority mail?

A: O, L
B: O, P
C: P, L
D: P, M
E: O, P, L, K



20. If only one building is to receive priority mail, and, as a result, O can be no
earlier than fourth in the order of buildings, which of the following must be the building
receiving priority mail that day?

A: K
B: L
C: M
D: P
E: S


A B C D E
A B C D E
Building Standards in Educational and Professional Testing
National Testing Service Pakistan Overseas Scholarship Scheme for PhD Studies



Each sentence below has one or two blanks, each blank indicating that
something has been omitted. Beneath the sentence are five lettered words or
sets of words. Choose the word or set of words that, when inserted in the
sentence, best fits the meaning of the sentence as a whole.

1. Florence Nightingale was ___ in the development of modern medicine, ___ such
practices as sanitization of hospital wards and isolation of actively infected
patients.

A. a collaboratorrejecting
B. a maverickprotesting
C. an innovatorinitiating
D. a pioneercriticizing
E. an individualiststandardizing




2. As a journalist who works to overturn erroneous convictions, Griffin Nicholson
was opposed to the court ruling ___ appeals for inmates who might be ___ .

A. barringculpable
B. curbingexonerated
C. encouraginginnocent
D. scrutinizingeligible
E. shieldingesteemed



3. Linda Greenhouse's articles for the New York Times are an outstanding example
of ___, capsulizing prose into a necessarily limited space.

A. Callousness
B. Brevity
C. Intuition
D. Propriety
E. Fortitude



III
VERBAL
Section
No. Of
Questions
20
Choose the correct answer for each
question and shade the
corresponding CIRCLE in the
answer sheet
A B C D E
A B C D E
A B C D E
Building Standards in Educational and Professional Testing
National Testing Service Pakistan Overseas Scholarship Scheme for PhD Studies

4. Roberto Clement was seen as ___ during his life because of both his selflessness
on the baseball field and his humanitarian work in his native Nicaragua.

A. An individualist
B. a grandstander
C. a sybarite
D. an altruist
E. an opportunist




5. His habit of spending more than he earned left him in a state of perpetual------
but he------------hoping to see a more affluent day.

A. indigence: persevered in
B. confusion: compromised by
C. enervation: retaliated by
D. motion: responded



6. Known for his commitment to numerous worthy causes, the philanthropist
deserved------ for his-----------.
A. recognition: folly
B. blame: hypocrisy
C. reward: modesty
D. credit: altruism



7. You should --------this paragraph in order to make your essay more-----.

A. Delete, succinct
B. Enlarge, redundant
C. Remove, discursive
D. Revise, abstruse



8. A------glance pays-----attention to details.

A. Furtive: meticulous
B. Cursory: little
C. Cryptic: close
D. Keen: scanty




Each question below consists of a related pair of words or phrases, followed by
five lettered pairs of words or phrases. Select the lettered pair that best
expresses a relationship similar to that expressed in the original pair.

A B C D E
A B C D E
A B C D E
A B C D E
A B C D E
Building Standards in Educational and Professional Testing
National Testing Service Pakistan Overseas Scholarship Scheme for PhD Studies
9. TIRADE: ABUSIVE

A. Diatribe: familial
B. Satire: pungent
C. Panegyric: laudatory
D. Eulogy: regretful



10. SOLDIER: REGIMENT

A. Colonel: martinet
B. Dancer: balletomane
C. Singer: chorus
D. Trooper: rifle



11. ASYLUM: SHELTER

A. Harbor: concealment
B. Palisade: display
C. Stronghold: defense
D. Cloister: storage



12. STATIC: MOVEMENT

A. Humdrum: excitement
B. Chronic: timeliness
C. Ecstatic: decay
D. Diligent: industry




13. INTEREST: FASCINATE

A. Vex: enrage
B. Vindicate: condemn
C. Regret: rue
D. Appall: bother


A B C D E
A B C D E
A B C D E
A B C D E
A B C D E
Building Standards in Educational and Professional Testing
National Testing Service Pakistan Overseas Scholarship Scheme for PhD Studies

14. LAUREL: VICTOR

A. Chevrons: army
B. Oscar: movie star
C. Power: glory
D. blue ribbon: cooking



15. PRECEDENT: JUSTIFICATION

A. Kindness: obedience
B. Authority: sanction
C. Usage: submission
D. Tradition: novelty


16. IMPLICATE: COMPLICATE

A. Vitality: inevitable
B. Empathy: sympathy
C. Importune: construct
D. Imply: simplify



17. PERMEATE: RUEFUL

A. Truculent: merciful
B. Sadden: pitiful
C. Evaporate: mournful
D. Penetrate: sorrowful


18. ENERVATE: STRENGTHEN

A. Aver: attribute
B. Divert: turn
C. Apprise: appraise
D. Stultify: enliven




Read the passages and answer the questions given at the end:

Recent technological advances in manned undersea vehicles have overcome some of the
limitations of divers and diving equipment. Without vehicles, divers often become
sluggish and their mental concentration was limited. Because of undersea pressure that
affected their speech organs, communication among divers was difficult or impossible.
But today, most oceanographers make observations by the means of instruments that
are lowered into the ocean or from samples taken from the water direct observations of
the ocean floor are made not only by divers of more than seven miles and cruise at the
depth of fifteen thousand feet. Radio equipment buoys can be operated by remote
A B C D E
A B C D E
A B C D E
A B C D E
A B C D E
Building Standards in Educational and Professional Testing
National Testing Service Pakistan Overseas Scholarship Scheme for PhD Studies
control in order to transmit information back to land based laboratories, including data
about water temperature, current and weather.
Some of mankinds most serous problems, especially those concerning energy and food,
may be solved with the help of observations made by these undersea vehicles.

19. With what topic is the passage primarily concerned?

A. Recent technological advances.
B. Communication among divers.
C. Direct observation of the ocean floor
D. Undersea vehicles




20. Divers have problems in communicating underwater because?

A. The pressure affected their speech organs
B. The vehicles they used have not been perfected.
C. They did not pronounce clearly
D. The water destroyed their speech organs.



A B C D E
A B C D E
Building Standards in Educational and Professional Testing
National Testing Service Pakistan Overseas Scholarship Scheme for PhD Studies









Drill Test II
Building Standards in Educational and Professional Testing
National Testing Service Pakistan Overseas Scholarship Scheme for PhD Studies


1. A piece of wood 35 feet, 6 inches long was used to make 4 shelves of equal length. The
length of each shelf was
A. 9 feet, 1 1/2 inches
B. 8 feet, 10 1/2 inches
C. 7 feet, 10 1/2 inches
D. 7 feet, 1 1/2 inches
E. 6 feet, 8 1/2 inches



2. The tiles in the floor of a bathroom are 15/16 inch squares. The cement between the tiles
is 1/16 inch. There are 3240 individual tiles in this floor. The area of the floor is

A. 225 sq. yds.
B. 2.5 sq. yds.
C. 250 sq. ft.
D. 22.5 sq. yds
E. 225 sq. ft.



3. A man bought a TV set that was listed at $160. He was given successive discounts of
20% and 10%. The price he paid was

A. $129.60
B. $119.60
C. $118.20
D. $115.20
E. $112.00



4. Mr. Jones' income for a year is $15,000. He pays 15% of this in federal taxes and 10%
of the remainder in state taxes. How much is left?

A. $12,750
B. $9,750
C. $14,125
D. $13,500
E. $11,475



5. The radius of a circle which has a circumference equal to the perimeter of a hexagon
whose sides are each 22 inches long is closest in length to which one of the following?
A. 7
B. 21
C. 14
D. 28
E. 24



I
Quantitative
Section
No of
Questions
10
Choose the correct answer for
each question and shade the
corresponding CIRCLE in the
answer sheet
A B C D E
A B C D E
A B C D E
A B C D E
A B C D E
Building Standards in Educational and Professional Testing
National Testing Service Pakistan Overseas Scholarship Scheme for PhD Studies
6. If a, is a multiple of 5 and b = 5a, which of the following could be the value of a + b?

I. 60 II. 100 III. 150
A. I only
B. III only
C. I and III only
D. II and III only
E. None of these


7. Which of the following expressions has the greatest value?

A. 4 4 4 + 4
B. 4 4 4 + 4
C. 4 4 4 4
D. 4 4 + 4 4
E. 2 2 + 2 2



8. If (a + 3) / 5 is an integer, what is remainder when a is divided by 5?
A. 1
B. 2
C. 3
D. 4
E. 5


9. The integral part of logarithm is called
A. Characteristic
B. Mantissa
C. Solution
D. Root
E. None of these



10. On the y-axis, the x-coordinate is
A. 1
B.
C. zero
D. -
E. 1

A B C D E
A B C D E
A B C D E
A B C D E
A B C D E
Building Standards in Educational and Professional Testing
National Testing Service Pakistan Overseas Scholarship Scheme for PhD Studies



For question 1 to 3

A volunteer uses a truck to pick up donations of unsold food and clothing from stores
and to deliver them to locations where they can be distributed. He drives only along
a certain network of roads. In the network there are two-way roads connecting each
of the following pairs of points: 1 with 2, 1 with 3, 1 with 5, 2 with 6, 3 with 7, 5
with 6, and 6 with 7. There are also one-way roads going from 2 to 4, from 3 to 2,
and from 4 to 3. There are no other roads in the network, and the roads in the
network do not intersect. To make a trip involving pickups and deliveries, the
volunteer always takes a route that for the whole trip passes through the fewest of
the points 1 through 7, counting a point twice if the volunteer passes through it
twice. The volunteer's home is at point 3. Donations can be picked up at a
supermarket at point 1, a clothing store at point 5, and a bakery at point 4.
Deliveries can be made as needed to a tutoring center at point 2, a distribution
center at point'6, and a shelter at point 7.

1. If the volunteer starts at the supermarket and next goes to the shelter, the first
intermediate point his route passes through must be

A: 2
B: 3
C: 5
D: 6
E: 7



2. If, starting from home, the volunteer is then to make pickups for the shelter at
the supermarket and the bakery (in either order), the first two intermediate
points on his route, beginning with the first, must be

A: 1 and 2
B: 1 and 3
C: 2 and 1
D: 2 and 4
E: 4 and 2



II
ANALYTICAL
Section
No. Of
Questions
20
Choose the correct answer for
each question and shade the
corresponding CIRCLE in the
answer sheet
A B C D E
A B C D E
Building Standards in Educational and Professional Testing
National Testing Service Pakistan Overseas Scholarship Scheme for PhD Studies

3. If, starting from the clothing store, the volunteer next is to pick up bread at
either the supermarket or the bakery (whichever stop makes his route go
through the fewest of the points) and then is to go to the shelter, the first two
points he reaches after the clothing store, beginning with the first, must be

A: 1 and 2
B: 1 and 3
C: 4 and 2
D: 6 and 2
E: 6 and 4



For question 4 to 5

There are seven cages next to each other in a zoo. The following is known about the
cages. Each cage has only one animal, which is either a monkey or a bear. There is
a monkey in each of the first and last cages. The cage in the middle has a bear. No
two adjacent cages have bears in them. The bears cage in the middle has two
monkey cages on either side. Each of the two other bear cages are between and
next to two monkey cages

4. How many cages have monkeys in them?

A: 2
B: 3
C: 4
D: 5
E: 6



5. The bear cage in the middle must have

A: No other bear cage to its left
B: No monkey cage on its right.
C: A bear cage to its left and to its right
D: Other bear cages next to it.
E: No monkey cage to its left.



For question 6 to 8

A nursery class in a school has a circular table with eleven seats around it. Five girls
(Kiran, Lado, Maryam, Omera and Parveen) and five boys (Farhan, Ghaus, Haris,
Imdad and Jahangir) are seated around the table. None of the girls are seated in a
seat adjacent to another girl. Kiran sits between Farhan and Ghaus, and next to
each of them. Jahangir does not sit next to Imdad.

6. Which of the following is a possible seating order around the table?

A: Empty seat, Farhan, Kiran, Ghaus, Lado, Omera, Haris, Imdad, Parveen, Jahangir,
and Maryam.
A B C D E
A B C D E
A B C D E
Building Standards in Educational and Professional Testing
National Testing Service Pakistan Overseas Scholarship Scheme for PhD Studies
B: Empty seat, Farhan, Kiran, Ghaus, Lado, Jahangir, Parveen, Omera, Imdad,
Maryam, Haris.
C: Empty seat, Farhan, kiran, Ghaus, Omera, Jahangir, Parveen, Imdad, Maryam,
Haris, Lado.
D: Empty seat, Omera, Farhan, Kiran, Ghaus, Lado, Jahangir, Imdad, Parveen,
Haris, Maryam.
E: Empty seat, Maryam, Farhan, Kiran, Ghaus, Lado, Jahangir, Perveen, Imdad,
Omera, Haris.



7. If Lado, Haris, Maryam, Jahangir, and Ghaus are seated in that order, which of
the following is a correct completion of the seating order after Ghaus?

A : Kiran, Farhan, Omera, Imdad, Parveen, empty seat.
B : Kiran, Farhan, Imdad, Omera, empty seat, Parveen
C : Farhan, Parveen, Kiran, Imdad, Omera, empty seat.
D : Kiran, Farhan, Parveen, Imdad, empty seat, Omera.
E : Kiran, Farhan, Omera, empty seats, Parveen, Imdad.



8. If Jahangir leaves his seat and occupies the empty seat, his new seating position
would be between:

A : Farhan and Kiran
B : Maryam and Ghaus
C : Kiran and Ghaus
D : Imdad and Lado
E : Parveen and Lado



For question 9 to 11
Four telephone operators (Abid, Baqir, Chauhan, and Daud) each have to perform
duties at the telephone exchange on four different days, Thursday through Sunday.
The following is known about their duty schedule: Chauhan has his duty day before
Abid. Daud has his duty day later than Baqir.

9. Which of the following is a possible order of duty days for the four operators?

A: Chauhan, Daud, Abid and Baqir.
B: Daud, Chauhan, Abid, and Baqir.
C: Baqir, Chauhan, Daud and Abid.
D: Abid, Chauhan, Daud and Baqir.
E: Abid, Baqir, Daud and Chauhan.





10. If Chauhan has his duty day on Saturday, who must have his duty day on
Thursday?

A: Either Abid or Daud.
A B C D E
A B C D E
A B C D E
A B C D E
Building Standards in Educational and Professional Testing
National Testing Service Pakistan Overseas Scholarship Scheme for PhD Studies
B: Daud
C: Abid
D: Either Baqir or Daud.
E: Baqir.



11. Each of the following is possible EXCEPT:

A: Chauhan has his duty on Thursday.
B: Baqir has his duty on Thursday.
C: Daud has his duty on Saturday.
D: Baqir has his duty on Sunday
E: Abid has his duty on Sunday.



For question 12 to 13
There are 12 seats facing the blackboard in a classroom, four seats
(A1, A2, A3 & A4) in that order are in row A, the first row from the blackboard.
Immediately behind row A is row B with four seats (B1, B2, B3 & B4) in that order.
Immediately behind row B, is the last row C with four seats (C1, C2, C3 & C4) in that
order. Six students attend the class the following is known about there seating
arrangement: Ejaz sits exactly in front of Comil, Seat A2 is always unoccupied Daud
does not sit next to Farhat, Gharuy sits in seat A4 Hamid does not sit in seat B4 All
the seats in row C always remain empty

12. If Daud sits in seat B3, then Farhat must sit in seat:

A: A3
B: A1
C: B4
D: B2
E: C2



13. Suppose that Hamid and Ejaz are sitting in seats A1 and A3 respectively, then it
CANNOT be true that seat:

A: B1 is occupied by Daud.
B: B2 is empty
C: B1 is empty
D: B3 is OCCUPIED BY Comil
E: B4 is empty




For question 14 to 17

The principal of a school is forming a committee. There are to be five members:
three teachers, chosen from Mr. J, Ms. K, Ms. L, Mr. M, and Mr. N; and two students,
chosen from O, P, Q, and R. The composition of the committee must conform to the
following conditions: Ms. J will serve only if R is also on the committee. Ms. L will
A B C D E
A B C D E
A B C D E
A B C D E
Building Standards in Educational and Professional Testing
National Testing Service Pakistan Overseas Scholarship Scheme for PhD Studies
not serve unless Ms. K and O also serve. Neither Mr. M nor Mr. N will serve without
the other. If P serves, neither Q nor R can serve.

14. Which of the following is an acceptable committee?

A: J, L, M, N, O
B: K, L, N, O, P
C: K, M, N, O, R
D: L, M, N, O, R
E: M, N, O, P, Q



15. How many different committees could include Mr. J and Q?
A: 1
B: 2
C: 3
D: 4
E: 5




16. If Q and R are both on the committee, who else must be on the committee?
A: J
B: K
C: L
D: M
E: O




17. In how many different ways can the principal select an acceptable committee?
A: Fewer than 3
B: 3
C: 5
D: 7
E: More than 7


A B C D E
A B C D E
A B C D E
A B C D E
Building Standards in Educational and Professional Testing
National Testing Service Pakistan Overseas Scholarship Scheme for PhD Studies

For question 18 to 20

Three adultsR, S, and Vwill be traveling in a van with five childrenF, H, J, L,
and M. The van has a driver's seat and one passenger seat in the front, and two
benches behind the front seats, one bench behind the other. Each bench has room
for exactly three people. Everyone must at in a seat or on a bench, and seating is
subject to the following restrictions: An adult must sit on each bench. Either R or S
must sit in the driver's seat. J must sit immediately beside M.

18. Which of the following can sit in the front passenger seat?
A: J
B: L
C: R
D: S
E: V



19. Which of the following groups of three can sit together on a bench?
A: F, J, and M
B: F, J, and V
C: F, S, and V
D: H, L, and S
E: L, M, and R



20. If F sits immediately beside V, which of the following CANNOT be true?
A: J sits immediately beside S.
B: L sits immediately beside V.
C: H sits in the front passenger seat.
D: F sits on the same bench as H.
E: H sits on the same bench as R.


A B C D E
A B C D E
A B C D E
Building Standards in Educational and Professional Testing
National Testing Service Pakistan Overseas Scholarship Scheme for PhD Studies


Each sentence below has one or two blanks; each blank indicates that
something has been omitted. Beneath the sentence are five lettered words or
sets of words. Choose the word or set of words that, when inserted in the
sentence, best fits the meaning of the sentence as a whole.

1. Surprisingly enough, it is more difficult to write about the--------than about the--
-and strange.

A. specific, foreign
B. abstract, prosaic
C. commonplace, exotic
D. simple, routine



2. A-----response is one that is made with----------.

A. stupid, fear
B. speedy, alacrity
C. sure, slowness
D. harmful, grimaces



3. A----is a-------.


A. norm, standard
B. criterion, mistake
C. discipline, school
D. doctrine, follower



4. It is widely believed that a nuclear war could ____ enough smoke and dust to
block out the sun and freeze the earth.

A. billow
B. extinguish
C. generate
D. duplicate
E. decimate



III
VERBAL
Section
No. Of
Questions
20
Choose the correct answer for each
question and shade the corresponding
CIRCLE in the answer sheet
A B C D E
A B C D E
A B C D E
A B C D E
Building Standards in Educational and Professional Testing
National Testing Service Pakistan Overseas Scholarship Scheme for PhD Studies

5. Consumption of red meat has ____ because its fat content has become a
worrisome and ____ matter.

A. abated ... dubious
B. skyrocketed ... stressful
C. abounded ... divisive
D. stabilized ... newsworthy
E. declined ... controversial



6. It takes ____ character to ____ the extremities of the arctic region.

A. an unflappable ... sustain
B. a dictatorial ... brook
C. a Spartan ... negotiate
D. an inimitable ... resist
E. a nomadic ... espouse



7. Consumers refused to buy meat products from the company because of rumors
that the water supply at the meat processing plant was ______; the rumors,
however, were quite ______, with no hard evidence to back them up.

A. uninspected .. reckless
B. contaminated .. unsubstantiated
C. impure .. damaging
D. misdirected .. scandalous
E. unscrupulous .. vicious



8. Many kinds of harmful viruses are unhindered when passing through different
parts of the host organism; indeed, there are few organic substances which such
viruses cannot______.

A. undermine
B. disseminate
C. aerate
D. exterminate
E. perforate


9. Their conversation was unsettling, for the gravity of their topic contrasted so
oddly with the ______ of their tone

A. uniqueness
B. rapidity
C. lightness
D. precision
E. reverence


A B C D E
A B C D E
A B C D E
A B C D E
A B C D E
Building Standards in Educational and Professional Testing
National Testing Service Pakistan Overseas Scholarship Scheme for PhD Studies

10. Throughout the animal kingdom, ____ bigger than the elephant.

A. whale is only the
B. only the whale is
C. is the whale only
D. only whale is the
E. whale is only



Each question below consists of a related pair of words or phrases, followed
by five lettered pairs of words or phrases. Select the lettered pair that best
expresses a relationship similar to that expressed in the original pair.

11. YAWN: BOREDOM ::

A. dream : sleep
B. anger : madness
C. smile : amusement
D. face : expression
E. impatience : rebellion



12. OBSTRUCTION : BUOY ::

A. construction : building
B. boy : girl
C. danger : red light
D. iceberg : titanic
E. arise : lay down



13. CONCERT : MUSIC ::

A. performance : artist
B. exhibition : art
C. play : actor
D. operetta : singer
E. rock : role



14. TEAMMATE : ADVERSARY ::

A. felon : criminal
B. enemy : associate
C. pacifier : agitator
D. winner : loser
E. friend : foe



A B C D E
A B C D E
A B C D E
A B C D E
A B C D E
Building Standards in Educational and Professional Testing
National Testing Service Pakistan Overseas Scholarship Scheme for PhD Studies
For Question 15-20 read the following passage:

A popular theory explaining the evolution of the universe is known as the Big Bang
Model. According to the model at some time between twenty billion years ago, all
present matter and energy were compressed into a small ball only a few kilometers
in diameter. It was, in effect, an atom that contained in the form of pure energy all
of the components of the entire universe. Then, at a moment in time that
astronomers refer to as T = 0, the ball exploded, hurling the energy into space.
Expansion occurred. As the energy cooled most of it became matter in the form of
protons, neutrons and electrons. These original particles combined to form
hydrogen and helium and continued to expand. Matter formed into galaxies with
stars and planets.


15. Which sentence best summarizes this passage?

A. The big band theory does not account for the evolution of the universe
B. According to the Big Bang Model, an explosion caused the formation of the
universe
C. The universe is made of hydrogen and helium
D. The universe is more than ten billion years old


16. According to this passage when were the galaxies formed?

A. Ten Billion Years ago
B. Fifteen billion Years ago
C. At T = 0
D. Twenty billion years ago


17. The word compressed in the passage could best be replaced by

A. Excited
B. Balanced
C. Reduced
D. Controlled


18. It may be inferred that

A. Energy and matter are the same
B. Protons, neutrons, and electrons are not matter
C. Energy may be converted into matter
D. The galaxies stopped expanding as energy cooled



19. The word it in the passage refers to

A. Energy
B. Space
C. Expansion
D. Matter

A B C D E
A B C D E
A B C D E
A B C D E
A B C D E
Building Standards in Educational and Professional Testing
National Testing Service Pakistan Overseas Scholarship Scheme for PhD Studies


20. The environment before the Big Bang is described as all the following EXCEPT

A. Compressed matter
B. Energy
C. All the components of the universe
D. Protons, electrons and neutrons



A B C D E
Building Standards in Educational and Professional Testing
National Testing Service Pakistan Overseas Scholarship Scheme for PhD Studies









Drill Test III
Building Standards in Educational and Professional Testing
National Testing Service Pakistan Overseas Scholarship Scheme for PhD Studies


I
Quantitative
Section
No. Of
Questions
20
Choose the correct answer for each
question and shade the
corresponding OVAL in the answer
sheet




1. If the pattern of dots shown above is continued so that each row after Row One
contains 1 dot more than the row immediately above it, which row will contain 12
dots?

A. Seven
B. Eight
C. Nine
D. Ten
E. Twelve



2. Each of Steve's buckets has a capacity of 11 gallons, while each of Mark's
buckets can hold 8 gallons. How much more water in gallons can 7 of Steve's
bucket's hold compared to 7 of Mark's buckets?

A. 3
B. 7
C. 21
D. 24
E. 56



3. Two integers have a sum of 42 and a difference of 22. The greater of the two
integers is

A. 22
B. 25
C. 28
D. 31
E. 32




4. The average of five numbers is 34. If three of the numbers are 28, 30 and 32,
what is the average of the other two?

A. 40
B. 50
C. 60
D. 70
A B C D E
A B C D E
A B C D E
Building Standards in Educational and Professional Testing
National Testing Service Pakistan Overseas Scholarship Scheme for PhD Studies
E. 80









5. In the figure above, what is the value of x?

A. 30
B. 40
C. 50
D. 80
E. 100



6. In a certain cake, two straight cuts (made along two different radii) succeed in
removing 4/15 of the total cake. What is the central angle in degrees of the
piece cut?

A. 26
B. 60
C. 85
D. 92
E. 96



7. If an equilateral triangle and a square have the same perimeter, what is the ratio
of the length of the sides of the equilateral triangle to the lengths of the sides of
the square?

A. 3:4
B. 4:3
C. 1:4
D. 1:3
E. 3:1



8. If 2 and 4 each divide q without remainder, which of the following must q divide
without remainder.
A B C D E
A B C D E
A B C D E
A B C D E
Building Standards in Educational and Professional Testing
National Testing Service Pakistan Overseas Scholarship Scheme for PhD Studies

A. 1
B. 2
C. 4
D. 8
E. It cannot be determined from the information given.




9. The ratio of boys to girls in a certain classroom was 2 : 3. If boys represented
five more than one third of the class, how many people were there in the
classroom?

A. 15
B. 25
C. 30
D. 45
E. 75




10. Let xy = z, where x,y,z and nonzero numbers. If x is multiplied by 3 and z is
divided by 3, this is equivalent to multiplying y by

A. 1/9
B. 1/3
C. 1
D. 3
E. 9



11. If 5x = 3, then (5x + 3) =

A. 0
B. 9
C. 18
D. 36
E. 81


A B C D E
A B C D E
A B C D E
A B C D E
Building Standards in Educational and Professional Testing
National Testing Service Pakistan Overseas Scholarship Scheme for PhD Studies

12. If the postal charges for a package are 62 cents for the first five ounces and 8
cents for each additional ounce, what is the weight of a package for which the
charges are $1.66? (Assume there are 16 ounces in one pound)

A. 1.05 pounds
B. 1.1 pounds
C. 1.125 pounds
D. 1.25 pounds
E. 1.5 pounds



13. If m/n = .75, then what is 3m + 2n?

A. 0
B. 8
C. 14
D. 17
E. 24



14. Which is greater?





A. if the quantity in Column A is greater
B. if the quantity in Column B is greater
C. if the two quantities are equal
D. if there is no relationship between these two quantities
E. if the relationship cannot be determined from the information given



15. Which of the following has a graph that is symmetric to the x-axis

A. y = x
B. y = x
2
+ 3
C. y
2
= x
D. y = x
3
x
E. y != x


Column A Column B
(10/4) / (3/2) * (3/7) (3/4) * (10/7) / (3/2)
A B C D E
A B C D E
A B C D E
A B C D E
Building Standards in Educational and Professional Testing
National Testing Service Pakistan Overseas Scholarship Scheme for PhD Studies

16. The prime factors of 96 are:

A. 2 and 3
B. 6 and 8
C. 2, 3 and 4
D. 8 and 12
E. 3 and 9



17. 8 are what percent of the 6?

A. 1.25
B. 75
C. 125
D. 133.333
E. 150



18. 2- 3 =

A. -8
B. 8
C. -6
D. -1/8
E. 1/8




19. If x + 1 < 3x + 5, then

A. X > -2
B. x < -2
C. x = 0
D. x < 2
E. x > 2



20. Which of the numbers cannot be represented by a repeating decimal?

A. 11/9
B. 23/7
C. 3
D. 4 1/3
E. 2



A B C D E
A B C D E
A B C D E
A B C D E
A B C D E
Building Standards in Educational and Professional Testing
National Testing Service Pakistan Overseas Scholarship Scheme for PhD Studies

II
Analytical
Reasoning
No. of
Questions
15
Choose the correct answer for each
question and shade the
corresponding CIRCLE in the answer
sheet

The office staff of the XYZ Corporation presently consists of three bookkeepers (L, M and
N ) and five secretaries (O, P, Q, R and S). Management is planning to open a new office
in another city sending three secretaries and two bookkeepers from the present staff. To
do so they plan to separate certain individuals who do not function well together. The
following guidelines were established to set up the new office:

(a) Bookkeepers L and N are constantly finding faults with one another therefore should
not be sent together to the new office.
(b) N and P function well alone but not as a team. They should be separated.
(c) O and R have not been on speaking terms for many months. They should not go
together.
(d) Since O and Q have been competing for a promotion, they should not be in one team.
Based on the information given above find the correct answers to the following Questions:
1. If M insists on staying back then how many combinations are possible?

A. 1
B. 2
C. 3
D. None



2. If L is to be moved as one of the bookkeepers, which of the following CANNOT be a
possible working unit?

A. LMOPS
B. LMPQS
C. LMORS
D. LMPRS



3. If N is sent to the new Office which member of the staff CANNOT be sent?

A. O
B. M
C. Q
D. R



4. If O is sent to the new office then which of the following is a possible team?

A. LMOPR
B. MNOQS
C. MNOPS
D. LMOPS



5. If both N and Q are moved to the new office, how many combinations are possible?

A. 2
B. 3
C. 4
A B C D E
A B C D E
A B C D E
A B C D E
Building Standards in Educational and Professional Testing
National Testing Service Pakistan Overseas Scholarship Scheme for PhD Studies
D. 1




6. A map representing countries R, S, W, X, Y and Z is to be drawn. Adjacent countries
cannot have the same color in the map.

The countries adjacent to each other are as follows:

Each of R, S, X and Y is adjacent to W.
X is adjacent to Y.
Each of R and S is adjacent to Z.

If X is the same color as Z then it must be true that

A. W is a different color from any other country.
B. S is a different color from any other country.
C. X is the same color as Y.
D. S is the same color as X.



Two statements, labeled I. & II, follow each of the following questions. The
statements contain certain information. In the questions you do not actually
have to compute an answer, rather you have to decide whether the information
given in the statements I. and II. is sufficient to find a correct answer by using
basic mathematics and every day facts?

7. A long distance runner has just completed running 28 miles. How long did it take him to
finish the journey?

I. His record speed is 8.25 miles per hour.
II. His average speed through the journey was 8 miles per hour.

A. Statement I. ALONE is sufficient but II. ALONE is not sufficient to answer this
question.
B. Statement II. ALONE is sufficient but I. ALONE is not sufficient to answer this
question.
C. Statements I. and II. TOGETHER are sufficient to answer the question but
NEITHER of them is sufficient ALONE.
D. Statements I. and II. COMBINED are NOT sufficient to answer the question and
additional information is needed to find the correct answer.



8. Captain of the national cricket team has to be the most popular member of the team.
Who is the captain of Pakistans national cricket team?
I. Waqar is the best player on the team.
II. Waseem is the senior-most member.

A. Statement I. ALONE is sufficient but II. ALONE is not sufficient to answer this
question.
B. Statement II. ALONE is sufficient but I. ALONE is not sufficient to answer this
question.
C. Statements I. and II. TOGETHER are sufficient to answer the question but
NEITHER of them is sufficient ALONE.
D. Statements I. and II. COMBINED are NOT sufficient to answer the question and
additional information is needed to find the correct
answer.

A B C D E
A B C D E
A B C D E
Building Standards in Educational and Professional Testing
National Testing Service Pakistan Overseas Scholarship Scheme for PhD Studies



9. In a BCE class at CIIT, 30 boys and 10 girls registered Calculus II. How many boys
passed the course?
I. 5 students could not pass.
II. There were 2 girls who obtained A grade.

A. Statement I. ALONE is sufficient but II. ALONE is not sufficient to answer this
question.
B. Statement II. ALONE is sufficient but I. ALONE is not sufficient to answer this
question.
C. Statements I. and II. TOGETHER are sufficient to answer the question but
NEITHER of them is sufficient ALONE.
D. Statements I. and II. COMBINED are NOT sufficient to answer the question and
additional information is needed to find the correct answer.




10. A horse ran 100 miles without stopping. What was its average speed in miles per hour?
I. The journey started at 8 PM and ended at 4 AM the following day.
II. The horse ran 20 miles per hour for the first 50 miles.

A. Statement I. ALONE is sufficient but II. ALONE is not sufficient to answer this
question.
B. Statement II. ALONE is sufficient but I. ALONE is not sufficient to answer this
question.
C. Statements I. and II. TOGETHER are sufficient to answer the question but
NEITHER of them is sufficient ALONE.
D. Statements I. and II. COMBINED are NOT sufficient to answer the question and
additional information is needed to find the correct answer.


A B C D E
A B C D E
A B C D E
Building Standards in Educational and Professional Testing
National Testing Service Pakistan Overseas Scholarship Scheme for PhD Studies

11. How much time will computer a need to solve 150 problems?
I. The computer needs 50 seconds to solve one problem.
II. Computer never takes more than 60 seconds to solve a problem.

A. Statement I. ALONE is sufficient but II. ALONE is not sufficient to answer this
question.
B. Statement II. ALONE is sufficient but I. ALONE is not sufficient to answer this
question.
C. Statements I. and II. TOGETHER are sufficient to answer the question but
NEITHER of them is sufficient ALONE.
D. Statements I. and II. COMBINED are NOT sufficient to answer the question and
additional information is needed to find the correct answer.



12. How many pencils does Raheel have?

I. He bought two boxes each containing 10 pencils.
II. He lent two pencils to Khaleel

A. Statement I. ALONE is sufficient but II. ALONE is not sufficient to answer this
question.
B. Statement II. ALONE is sufficient but I. ALONE is not sufficient to answer this
question.
C. Statements I. and II. TOGETHER are sufficient to answer the question but
NEITHER of them is sufficient ALONE.
D. Statements I. and II. COMBINED are NOT sufficient to answer the question and
additional information is needed to find the correct answer.



13. In a certain farm there are 47 goats. How many large brown goats are there?

I. 16 of the goats are large.
II. There are 18 brown goats in the farm.

A. Statement I. ALONE is sufficient but II. ALONE is not sufficient to answer this
question.
B. Statement II. ALONE is sufficient but I. ALONE is not sufficient to answer this
question.
C. Statements I. and II. TOGETHER are sufficient to answer the question but
NEITHER of them is sufficient ALONE.
D. Statements I. and II. COMBINED are NOT sufficient to answer the question and
additional information is needed to find the correct answer.


14. Can there be more than 200 pictures in a 60-page book?

I. There is at least one picture in each page.
II. There are no more than 3 pictures in any page.

A. Statement I. ALONE is sufficient but II. ALONE is not sufficient to answer this
question.
B. Statement II. ALONE is sufficient but I. ALONE is not sufficient to answer this
question.
C. Statements I. and II. TOGETHER are sufficient to answer the question but
NEITHER of them is sufficient ALONE.
D. Statements I. and II. COMBINED are NOT sufficient to answer the question and
additional information is needed to find the correct answer.


A B C D E
A B C D E
A B C D E
A B C D E
Building Standards in Educational and Professional Testing
National Testing Service Pakistan Overseas Scholarship Scheme for PhD Studies
15. If P > Q and R > S, then, P + R > Q + S. Is X > Y?
I. X + A > Y + B
II. A > B

A. Statement I. ALONE is sufficient but II. ALONE is not sufficient to answer this
question.
B. Statement II. ALONE is sufficient but I. ALONE is not sufficient to answer this
question.
C. Statements I. and II. TOGETHER are sufficient to answer the question but
NEITHER of them is sufficient ALONE.
D. Statements I. and II. COMBINED are NOT sufficient to answer the question and
additional information is needed to find the correct answer.

A D E B C
Building Standards in Educational and Professional Testing
National Testing Service Pakistan Overseas Scholarship Scheme for PhD Studies



Each sentence below has one or two blanks, each blank indicates
that something has been omitted. Beneath the sentence are five
lettered words or sets of words. Choose the word or set of words
that, when inserted in the sentence, best fits the meaning of the
sentence as a whole.


1. Although its publicity has been ___, the film itself is intelligent, well-acted,
handsomely produced and altogether ___

A. tasteless respectable
B. extensive moderate
C. sophisticated moderate
D. risqu crude
E. perfect spectacular



2. The Inuit natives of Alaska's North Slope worry that ___ oil exploration might
___their sensitive natural environment.

A. additionalassist
B. currentbolster
C. curtailedshatter
D. unregulateddamage
E. controlledreassess



3. Ants live in colonies based on ___; each member contributes to the good of all by
actively working with others in performing necessary tasks.

A. Heredity
B. Individualism
C. Cooperation
D. Reasoning
E. Instinct



Each question below consists of a related pair of words or phrases, followed
by five lettered pairs of words or phrases. Select the lettered pair that best
expresses a relationship similar to that expressed in the original pair.

4. STUDYING: LEARNING::
A. running : jumping
B. investigating : discovering
C. reading : writing


Choose the correct answer for each
question and shade the corresponding
CIRCLE in the answer sheet
III
VERBAL
Section No of
Questions
10
A B C D E
A B C D E
A B C D E
Building Standards in Educational and Professional Testing
National Testing Service Pakistan Overseas Scholarship Scheme for PhD Studies
D. dancing : swimming
E. talking : listening



5. AFTERNOON : DUSK ::

A. breakfast : dinner
B. yesterday : tomorrow
C. Sunday : Saturday
D. night : dawn
E. arise : lay down



6. VIBRATION: SOUND :

A. gravity : pull
B. watercolor : paint
C. accident : death
D. worm : reptile
E. arrive : home



7. RUN : RACE ::

A. walk : pogo stick
B. swim : boat
C. fly : kite
D. sink : bottle
E. repair : automobile



Read the passages and answer the questions asked at its end.

Almost a century ago Alfred Binet, a gifted psychologist, was asked by the French
Ministry of Education to help determine who would experience difficulty in school. Given
the influx of provincials to the capital, along with immigrants of uncertain stock, Parisian
officials believed they needed to know who might not advance smoothly through the
system. Proceeding in an empirical manner, Binet posed many questions to youngsters
of different ages. He ascertained which questions when answered correctly predicted
success in school, and which questions when answered incorrectly foretold school
difficulties. The items that discriminated most clearly between the two groups became,
in effect, the first test of intelligence.

Binet is a hero to many psychologists. He was a keen observer, a careful scholar, an
inventive technologist. Perhaps even more important for his followers, he devised the
instrument that is often considered psychology's greatest success story. Millions of
people who have never heard Binet's name have had aspects of their fate influenced by
instrumentation that the French psychologist inspired. And thousands of
psychometricians specialists in the measurement of psychological variables earn
their living courtesy of Binet's invention.
A B C D E
A B C D E
A B C D E
A B C D E
Building Standards in Educational and Professional Testing
National Testing Service Pakistan Overseas Scholarship Scheme for PhD Studies

Although it has prevailed over the long run, the psychologist's version of intelligence is
now facing its biggest threat. Many scholars and observers and even some
iconoclastic psychologists feel that intelligence is too important to be left to the
psychometricians. Experts are extending the breadth of the concept proposing much
intelligence, including emotional intelligence and moral intelligence. They are
experimenting with new methods of ascertaining intelligence, including some that avoid
tests altogether in favor of direct measures of brain activity. They are forcing citizens
everywhere to confront a number of questions: What is intelligence? How ought it to be
assessed? And how do our notions of intelligence fit with what we value about human
beings? In short, experts are competing for the "ownership" of intelligence in the next
century.

8. According to the passage, which of the following is most similar to the "barometer"
developed by Binet?

A. The S.A.T. or other standardized college admission test.
B. The written portion of a driver's license test.
C. Open tryouts for a varsity athletic team
D. An electronic scan of brain-wave activity.
E. The trivia questions of a game show.



9. The author suggests which of the following about "citizens everywhere"?

A. They do not have a sufficiently accurate definition of intelligence to evaluate
recent scientific developments.
B. They stand to benefit from recent progress in the scientific assessment of
intelligence.
C. The experiments they are performing with new methods of intelligence
measurement are valuable and interesting.
D. They are at odds with the experts over who should have the right to define
"intelligence."
E. Traditionally they have not given careful consideration to some important
issues concerning intelligence.


10. As used in line # 8, "discriminated" most nearly means

A. equalized
B. predetermined
C. showed favoritism
D. displayed intolerance
E. distinguished


A B C D E
A B C D E
A B C D E
Building Standards in Educational and Professional Testing
National Testing Service Pakistan Overseas Scholarship Scheme for PhD Studies





Drill Test IV
Building Standards in Educational and Professional Testing
National Testing Service Pakistan Overseas Scholarship Scheme for PhD Studies


I
Quantitative
Section
No. Of
Questions
20
Choose the correct answer for each
question and shade the
corresponding CIRCLE in the
answer sheet


1. If the length of BC is twice the length of AC, what are the coordinates of B where
A=(x,y)?

A. (x,2y)
B. (-x,2y)
C. (2x,y)
D. (-2x,y)
E. (-2x,2y)


2. The average of five numbers is 34. If three of the numbers are 28, 30 and 32, what
is the average of the other two?
A. 40
B. 50
C. 60
D. 70
E. 80

A B C D E
A B C D E
1
Building Standards in Educational and Professional Testing
National Testing Service Pakistan Overseas Scholarship Scheme for PhD Studies



3. In the figure above, rectangle AEJL has been divided into 8 congruent squares with
each of the 8 squares having an area of 16. What is the length of
AE + MF + LG+ AL + BK + CJ + DH + EG?

A. 32
B. 44
C. 88
D. 128
E. 176



4. For any positive integer x, #x = x/3 and &x = 9 /x. which of the following is an
expression for the product of #x and &x?

A. 3x
B. x
C. 1
D. x
3
/64
E. 27 x
3




5. In a certain town, p gallons of gasoline are needed per month for each car. How
long will q gallons last at this rate given that there are r cars in town?

A. pr/q
B. qr/p
C. r/pq
D. q/pr
E. pqr


G
A B C D E
A B C D E
A B C D E
Building Standards in Educational and Professional Testing
National Testing Service Pakistan Overseas Scholarship Scheme for PhD Studies

6. Let xy = z, where x,y,z are nonzero numbers. If x is multiplied by 3 and z is divided
by 3, this is equivalent to multiplying y by

A. 1/9
B. 1/3
C. 1
D. 3
E. 9




7. If x, y, and z are different positive odd integers and x + y + z = 11, what is the
greatest possible value of z?

A. 10
B. 9
C. 8
D. 7
E. 6



8. If the postal charges for a package are 62 cents for the first five ounces and 8 cents
for each additional ounce, what is the weight of a package for which the charges are
$1.66? (Assume there are 16 ounces in one pound)

A. 1.05 pounds
B. 1.1 pounds
C. 1.125 pounds
D. 1.25 pounds
E. 1.5 pounds



9. What fraction of two weeks is 24 minutes?

A. 1/120
B. 1/336
C. 1/840
D. 1/2880
E. 1/20160


A B C D E
A B C D E
A B C D E
A B C D E
Building Standards in Educational and Professional Testing
National Testing Service Pakistan Overseas Scholarship Scheme for PhD Studies

10. If the vertices of a triangle are at (0,0), (-3, 4) and (3, 4), what is the area of the
triangle?

A. 4
B. 6
C. 12
D. 14
E. 18




11. A water-tank has a base with dimensions 2 feet by 6 feet. If a cube with each side 1
foot is totally immersed in the water, how many inches will the water rise? (12inches
= 1 foot)

A. 1
B. 2
C. 4
D. 8
E. It cannot be determined from the information given





12. In the figure above, the quadrilateral ABCD is a trapezoid with x = 2. The diameter
of each semicircle is a side of the trapezoid. What is the sum of the lengths of the
four drawn semicircles? (Round to the nearest whole number.)

A. 13
B. 16
C. 19
D. 22
E. 31


A B C D E
A B C D E
A B C D E
Building Standards in Educational and Professional Testing
National Testing Service Pakistan Overseas Scholarship Scheme for PhD Studies

13. If n + 3 = n x 3, then n =

A. 0.5
B. 1.5
C. 2
D. 2.5
E. 3



14. If an equilateral triangle and a square have the same perimeter, what is the ratio of
the length of the sides of the equilateral triangle to the lengths of the sides of the
square?

A. 3:4
B. 4:3
C. 1:4
D. 1:3
E. 3:1




15. A restaurant has a special whereby both parents can eat for $20 and each child can
eat for $5. Assuming a family group consists of both parents and at least one child,
what is the maximum number of family groups that could have attended given that
the restaurant took $115?

A. 6
B. 5
C. 4
D. 3
E. 2




16. Which of the following points lays in the interior of the circle whose radius is 10 and
whose center is at the origin?

A. (-9, 4)
B. (5, -19)
C. (0, -10)
D. (10, -1)
E. (0,15)



A B C D E
A B C D E
A B C D E
A B C D E
Building Standards in Educational and Professional Testing
National Testing Service Pakistan Overseas Scholarship Scheme for PhD Studies

17. If the perimeter of the rectangle ABCD is 14, what is the perimeter of BCD?




18. The roots of ax
2
+ bx + c = 0 are real only if

A. b
2
4ac 0
B. b
2
4ac = 0
C. b
2
+ 4ac = 0
D. b
2
4ac < 0
E. b
2
4bc < 0



19. The two numbers, whose sum is -13 and product -30, are

A. 2, 15
B. 2, -15
C. -3, 10
D. 3, 10
E. -3, -13



20. Let A = total area of five circles of radius r and let B = total area of three circles of
radius s. If A = B, then r / s =

A. 3/5
B. 3 / 5
C. 3 / 5
D. (3) / 5
E. 3





Two statements labeled I & II, follow each of the following questions. The statements contain
certain information. In the questions you do not actually have to compute an answer, rather you

II
Analytical
Reasoning
No. Of
Questions
15
Choose the correct answer for each
question and shade the
corresponding CIRCLE in the answer
sheet
A. 7
B. 12
C. 7 + 29
D. 86
E. It cannot be determined from
the information given.
C
B
D
A
A B C D E
A B C D E
A B C D E
A B C D E
Building Standards in Educational and Professional Testing
National Testing Service Pakistan Overseas Scholarship Scheme for PhD Studies
have to decide whether the information given in the statements I. and II. is sufficient to find a
correct answer by using basic mathematics and every day facts?

1. What day of the week is today?

I. Today is March 25.
II. Akram left Pakistan on Wednesday.

A. Statement I. ALONE is sufficient but II. ALONE is not sufficient to answer this
question.
B. Statement II. ALONE is sufficient but I. ALONE is not sufficient to answer this
question.
C. Statements I. and II. TOGETHER are sufficient to answer the question but
NEITHER of them is sufficient ALONE.
D. Statements I. and II. COMBINED are NOT sufficient to answer the question and
additional information is needed to find the correct answer.



2. Can any of the four rivers be more than 300 meters wide?

I. The narrowest of the four rivers is 240 meters wide.
II. Average width of the four rivers is 300 meters.

A. Statement I. ALONE is sufficient but II. ALONE is not sufficient to answer this
question.
B. Statement II. ALONE is sufficient but I. ALONE is not sufficient to answer this
question.
C. Statements I. and II. TOGETHER are sufficient to answer the question but
NEITHER of them is sufficient ALONE.
D. Statements I. and II. COMBINED are NOT sufficient to answer the question and
additional information is needed to find the correct answer




3. If it is raining then there must be clouds. Are there clouds?

I. It is not raining.
II. It rained yesterday.

A. Statement I. ALONE is sufficient but II. ALONE is not sufficient to answer this
question.
B. Statement II. ALONE is sufficient but I. ALONE is not sufficient to answer this
question.
C. Statements I. and II. TOGETHER are sufficient to answer the question but
NEITHER of them is sufficient ALONE.
D. Statements I. and II. COMBINED are NOT sufficient to answer the question and
additional information is needed to find the correct answer.



Read the passage to answer the question 4-5
A map representing countries R, S, W, X, Y and Z is to be drawn. Adjacent countries
cannot have the same color in the map. The countries adjacent to each other are as
follows:
Each of R, S, X and Y is adjacent to W.
X is adjacent to Y.
A B C D E
A B C D E
A B C D E
Building Standards in Educational and Professional Testing
National Testing Service Pakistan Overseas Scholarship Scheme for PhD Studies
Each of R and S is adjacent to Z.

4. Which of the following countries can be the same color as W?

A. S
B. X
C. Y
D. Z




5. Which of the following is a pair of countries that can be the same color?

A. R and S
B. S and W
C. W and X
D. X and Y


Questions 6 to 11 depends on the following passage

A college president wishes to select four members of a faculty-student committee as special
representatives to meet with the college's board of trustees.
The faculty-student committee consists of eight members four of which (F, G, H and I) are faculty
members whereas the other four (R, S, T and U) are students.
The president can select any four of the eight committee members as long as the following rules
are observed:
The four representatives must consist of exactly two faculty members and two students.
Either F or G must be one of the representatives but F and G both cannot be the representatives.
If R is a representative then H must also be a representative.
If T is a representative then G cannot be a representative.
6. If T is a representative but H is not a representative then the whole group can be
determined if it were also true that:

A. F is a representative.
B. I is a representative.
C. R is not a representative.
D. U is not a representative.


7. If R is a representative then which of the following CANNOT be a representative?

A. H
B. I
C. S
D. T


8. If G is a representative then which of the following can be the other three representatives?

A. F, S and U
B. H, I and R
C. H, R and S
D. I, R and U
A B C D E
A B C D E
A B C D E
A B C D E
Building Standards in Educational and Professional Testing
National Testing Service Pakistan Overseas Scholarship Scheme for PhD Studies

9. If neither S nor U is a representative then which of the following is the pair of faculty-
member representatives?

A. F and G
B. F and H
C. F and I
D. G and H



10. If G, I and S are representatives then which of the following must also be a
representative?

A. H
B. R
C. T
D. U



11. If F and I are representatives then which of the following is not a representative?

A. I
B. S
C. U
D. R



Questions 12 to 14 depends on the following passage

At a congress of the Ruling Party, the seven top party leaders, who are all cabinet
ministers, are seated on a platform in order of rank the Prime Minister being in the
center. The closer a person is to the Prime Minister; the higher is his/her rank.
Moreover, a person sitting on the right of the PM outranks the one sitting equidistant on
the left of the PM. The seven leaders are T, U, V, W, X, Y, and Z.
Y is four places to the left of the Minister of Agriculture, who is two places to the right of
V.
Us neighbors are T and the Minister of Agriculture.
Z is two places to the left of W.
The Ministers of Education, Mining and Culture are seated together, in order, from left to
right.
The remaining Ministers are those of Social Welfare and Defense.

12. The fifth ranking person in the party hierarchy is:
A. Z, the Minister of Mining
B. Y, the Minister of Culture
C. W, the Prime Minister.
D. X, the minister of Defense.




13. How many of the seven party leaders outrank the Minister of Education?
A B C D E
A B C D E
A B C D E
A B C D E
A B C D E
Building Standards in Educational and Professional Testing
National Testing Service Pakistan Overseas Scholarship Scheme for PhD Studies
A. 3
B. 4
C. 5
D. 6




14. The lowest ranking Minister is
A. Minister of Social Welfare.
B. Minister of Defense.
C. Minister of Education.
D. Minister of Mining.




15. A meadow in springtime is beautiful, even if no one is there to appreciate it.
This statement would be a logical opposite to which of the following claims?
A. People will see only what they want to see.
B. Beauty exits only in the eyes of the beholder.
C. Beauty does not depend on seasons.
D. The greatest pleasure available to mankind is the contemplation of
beauty.


A B C D E
A B C D E
A B C D E
Building Standards in Educational and Professional Testing
National Testing Service Pakistan Overseas Scholarship Scheme for PhD Studies


Each sentence below has one or two blanks, each blank indicates that
something has been omitted. Beneath the sentence are five lettered words or
sets of words. Choose the word or set of words that, when inserted in the
sentence, best fits the meaning of the sentence as a whole.
1. Some illnesses such as smallpox, which have been almost eliminated in the United
States are still ____ in many places abroad.
A. discussed
B. prevalent
C. scarce
D. unknown
E. hospitalized



2. A recent study indicates that the crime rate in the United States remains ____ and
that one in three households ____ some form of major crime in any year
A. incredible ... witnesses
B. astronomical ... experiences
C. simultaneous ... perpetrates
D. unsuccessful ... initiates
E. defeated ... prosecutes



Each question below consists of a related pair of words or phrases, followed by
five lettered pairs of words or phrases. Select the lettered pair that best
expresses a relationship similar to that expressed in the original pair.

3. SALVAGE : TREASURE
A. settle : argument
B. incorporate : company
C. send : correspondence
D. rescue : victim
E. recycle : newspaper



4. CONTROVERSY : ARBITRATOR
A. peacemaker : conflict
B. artifact : anthropologist
C. game : referee
D. dispute : mediator
E. disease : pathologist




Read the passages and answer the questions given at its end:

III
VERBAL
Section
No. Of
Questions
10
Choose the correct answer for each
question and shade the
corresponding CIRCLE in the answer
sheet
A B C D E
A B C D E
A B C D E
A B C D E
Building Standards in Educational and Professional Testing
National Testing Service Pakistan Overseas Scholarship Scheme for PhD Studies

We are profoundly ignorant about the origins of language and have to content ourselves
with more or less plausible speculations. We do not even know for certain when
language arose, but it seems likely that it goes back to the earliest history of man,
perhaps half a million years. We have no direct evidence, but it seems probable that
speech arose at the same time as tool making and the earliest forms of specifically
human cooperation. In the great Ice Ages of the Pleistocene period, our earliest human
ancestors established the Old Stone Age culture; they made flint tools and later tools of
bone, ivory, and antler; they made fire and cooked their food; they hunted big game,
often by methods that called for considerable cooperation and coordination. As their
material culture gradually improved, they became artists and made carvings and
engravings on bones and pebbles, and wonderful paintings of animals on the walls of
caves. It is difficult to believe that the makers of these Paleolithic cultures lacked the
power of speech. It is a long step Admittedly, from the earliest flint weapons to the
splendid art of the late Old Stone Age: the first crude flints date back perhaps to
500,000 B.C., while the finest achievements of Old Stone Age man are later than
100,000 B.C.; and, in this period, we can envisage a corresponding development of
language, from the most primitive and limited language of the earliest human groups to
a fully developed language in the flowering time of Old Stone Age culture.

How did language arise in the first place? There are many theories about this, based on
various types of indirect evidence, such as the language of children, the language of
primitive societies, the kinds of changes that have taken place in languages in the
course of recorded history, the behavior of higher animals like chimpanzees, and the
behavior of people suffering from speech defects. These types of evidence may provide
us with useful pointers, but they all suffer from limitations, and must be treated with
caution. When we consider the language of children, we have to remember that their
situations are quite different from that of our earliest human ancestors, because the
child is growing up in an environment where there is already a fully developed language,
and is surrounded by adults who use that language and are teaching it to him. For
example, it has been shown that the earliest words used by children are mainly the
names of things and people (Doll, Spoon, Mummy): but, this does not prove that
the earliest words of primitive man were also the names of things and people. When
the child learns the name of an object, he may then use it to express his wishes or
demands: Doll!: often means Give me my doll! Or Ive dropped my doll: pick it up
for me!; the child is using language to get things done, and it is almost an accident of
adult teaching that the words used to formulate the childs demands are mainly nouns,
instead of words like Bring! Pick up!; and so on.

5. The main idea of this excerpt is
(A) to provide evidence of the origin of language.
(B) to present the need for language.
(C) to discuss how early man communicated.
(D) to present the culture of early man.
(E) to narrate the story of English.



6. Theories of the origin of language include all of the following EXCEPT
(A) changes occurring through the years.
(B) the need to communicate.
(C) language of children.
(D) the first mans extensive vocabulary.
(E) communication among primitive men.

A B C D E
Building Standards in Educational and Professional Testing
National Testing Service Pakistan Overseas Scholarship Scheme for PhD Studies


7. The purpose of the discussion of the word, Doll, is intended to
(A) Trace the evolution of a noun.
(B) Support the fact that naming things is most important.
(C) Indicate how adults teach language to children.
(D) Show the evolution of many meanings for one word.
(E) Evince mans multiple uses of single words



8. The implication of the author regarding the early elements of language is that
(A) There were specific real steps followed to develop our language.
(B) Care must be exercised when exhuming what we consider the roots of
language.
(C) We owe a debt of gratitude to the chimpanzee contribution.
(D) Adults created language in order to instruct their children.
(E) Language was fully developed by primitive man.



9. If we accept that primitive man existed for a very long period of time without
language, then we may assume that
(A) language is not necessary to mans existence.
(B) language developed with the developing culture of primitives.
(C) primitives existed in total isolation from one another.
(D) children brought about a need for language.
(E) mankind was not intended to communicate.



10. After a reading of this article, one might infer that
(A) society creates problems with language.
(B) language is for adults to instruct children.
(C) society uses language to improve itself.
(D) with the evolution of language came wisdom.
(E) language brings power.






A B C D E
A B C D E
A B C D E
A B C D E
A B C D E
Building Standards in Educational and Professional Testing
National Testing Service Pakistan Overseas Scholarship Scheme for PhD Studies






Answer Keys to
Drill Tests
Building Standards in Educational and Professional Testing
National Testing Service Pakistan Overseas Scholarship Scheme for PhD Studies
DRILL TEST I - ANSWER KEY
Section-I Quantitative

1.
2.
3.
4.
5.
6.
7.
8.
9.
10.

Section-II Analytical Reasoning

1.
2.
3.
4.
5.
6.
7.
8.
9.
10.
11.
12.
13.
14.




15.
16.
17.
18.
19.
20.

Section-III Verbal
1.
2.
3.
4.
5.
6.
7.
8.
9.
10.
11.
12.
13.
14.
15.
16.
17.
18.
19.
20.

B C E
A
D E B C
A D C
D B C
A D E
A D B

A C
A D C
D E B C
D E B

A E C
A E B
D B
C

A B

A E B


C
C D E

B

A D
C A E B

C A

B
D
B
D B
A
D B
A
C
C
C A D
E
E
E
E
A E B C
B
B
B
A

A
A
A
A
E
E
E
E
D
C
C
D
D
E
D
D
A
D E C
A
E B C
A
D E B
A
D E C
A
D B C
A E B
D
A
D E B
A
D E C
A
D E C
D E B C
A
E B C
E B C
D
A
D E C
A
E B C
A
D E B
A
D E B
D E B C
E B C
D
D E C
D E C
D E C
A
E B C
C A E
B
C A E B
C D E B
A
A
A
Building Standards in Educational and Professional Testing
National Testing Service Pakistan Overseas Scholarship Scheme for PhD Studies
DRILL TEST II - ANSWER KEY

Section-I Quantitative

1.
2.
3.
4.
5.
6.
7.
8.
9.
10.

Section-II Analytical Reasoning

1.
2.
3.
4.
5.
6.
7.
8.
9.
10.
11.
12.
13.
14.




15.
16.
17.
18.
19.
20.

Section-III Verbal
1.
2.
3.
4.
5.
6.
7.
8.
9.
10.
11.
12.
13.
14.
15.
16.
17.
18.
19.
20.







C E
A
D E C
A C
D B C
A D E
A D B
A C
A D C
D E B

E B
A E C
D E B
D E
A E B
A E B


C
C D

B
D
B
C A B

A

B
D
B
C B

D B
A

B
C
C
A D
E
E
E
E
E B C
B
B
A

A
A
A
A
A
E
E
E
E
D
C
C
D
D
E
D
D
C
D

D
D
A
D E B
A
E B C
A
D E C
A
E B C
A
D B C
A E B
D
A
D E
D E C
A
D E
D B C
A
E B
E C
D
A
E C
A
E B
A
D E
A
D E B
D E B
E C
D
D B C
D E C
D E B
A
E B
A E
B
C E B
C E B
A
A
A
A
C

A
C
B
C
C
C
B
B
B
A
A
A
A
D
D
D
D
D
A

Building Standards in Educational and Professional Testing
National Testing Service Pakistan Overseas Scholarship Scheme for PhD Studies
DRILL TEST III - ANSWER KEY

Section-I Quantitative

1.
2.
3.
4.
5.
6.
7.
8.
9.
10.
11.
12.
13.
14.
15.
16.
17.
18.
19.
20.
Section-II Analytical Reasoning
1.
2.
3.
4.
5.
6.
7.
8.
9.
10.


11.
12.
13.
14.
15.

Section-III Verbal
1.
2.
3.
4.
5.
6.
7.
8.
9.
10.
A B C D E
A B C D E
A B C D E
A B C D E
A B C D E
A B C D E
A B C D E
A B C D E
A B C D E
A B C D E
A B C D E
A B C D E
A B C D E
A B C D E
A B C D E
A B C D E
A B C D E
A B C D E
A B C D E
A B C D E
A B C D E
A B C D E
A B C D E
A B C D E
A B C D E
A B C D E
A B C D E
A B C D E
A B C D E
A B C D E
A B C D E
A B C D E
A B C D E
A B C D E
A B C D E
A B C D E
A B C D E
A B C D E
A B C D E
A B C D E
A B C D E
A B C D E
A B C D E
A B C D E
A B C D E
Building Standards in Educational and Professional Testing
National Testing Service Pakistan Overseas Scholarship Scheme for PhD Studies

DRILL TEST IV - ANSWER KEY
Section-I Quantitative

1.
2.
3.
4.
5.
6.
7.
8.
9.
10.
11.
12.
13.
14.
15.
16.
17.
18.
19.
20.
Section-II Analytical Reasoning
1.
2.
3.
4.
5.
6.
7.
8.
9.
10.


11.
12.
13.
14.
15.

Section-III Verbal
1.
2.
3.
4.
5.
6.
7.
8.
9.
10.

A B C D E
A B C D E
A B C D E
A B C D E
A B C D E
A B C D E
A B C D E
A B C D E
A B C D E
A B C D E
A B C D E
A B C D E
A B C D E
A B C D E
A B C D E
A B C D E
A B C D E
A B C D E
A B C D E
A B C D E
A B C D E
A B C D E
A B C D E
A B C D E
A B C D E
A B C D E
A B C D E
A B C D E
A B C D E
A B C D E
A B C D E
A B C D E
A B C D E
A B C D E
A B C D E
A B C D E
A B C D E
A B C D E
A B C D E
A B C D E
A B C D E
A B C D E
A B C D E
A B C D E
A B C D E
Building Standards in Educational and Professional Testing
National Testing Service Pakistan Overseas Scholarship Scheme for PhD Studies









SAMPLE TEST
General
















Note: The Sample Test does not include quantitatively the same number
of questions as there would be in the actual papers. They are merely
meant to provide conceptual guidance to the users or prospective
candidates.
Building Standards in Educational and Professional Testing
National Testing Service Pakistan Overseas Scholarship Scheme for PhD Studies



I
VERBAL
Section
No. Of
Questions
15
Choose the correct answer for each
question and shade the
corresponding CIRCLE in the answer
sheet

Each sentence below has one or two blanks, each blank indicates
that something has been omitted. Beneath the sentence are five
lettered words or sets of words. Choose the word or set of words
that, when inserted in the sentence, best fits the meaning of the
sentence as a whole.
1. Despite the millions of dollars spent on improvements, the telephone system in India
remains ________ and continues to ___________ the citizens who depend upon it.

A. Primitiveinconvenience
B. Bombastic...upset
C. Suspicious...connect
D. Outdated...elate
E. Impartial...vex

2. Unlike the images in symbolist poetry which are often vague and _______ , the
images of surrealist poetry are startlingly ________ and bold.

A. extraneous...furtive
B. trivial...inadvertent
C. obscure...concrete
D. spectacular...pallid
E. symmetricalvirulent

3. A good trial lawyer will argue only what is central to an issue, eliminating
___________ information or anything else that might __________ the client.

A. Seminal...amuse
B. Extraneous...jeopardize
C. Erratic...enhance
D. Prodigious...extol
E. Reprehensibleinitiate

4. Pollen grains and spores that are 200 millions old are now being extracted from
shale and are ____________ the theory that the breakup of the continents occurred
in stages; in fact, it seems that the breakups occurred almost __________ .

A. refining...blatantly
B. reshaping...simultaneously
C. countermanding...imperceptibly
D. forging...vicariously
E. supporting...haphazardly

Building Standards in Educational and Professional Testing
National Testing Service Pakistan Overseas Scholarship Scheme for PhD Studies
Each question below consists of a related pair of words or phrases, followed by
five lettered pairs of words or phrases. Select the lettered pair that best
expresses a relationship similar to that expressed in the original pair.

5. DETENTION : RELEASE ::

A. viciousness : attack
B. calamity : repair
C. qualification : employ
D. induction : discharge
E. therapy : confuse

6. PONDEROUS : WEIGHT ::

A. eternal : temporality
B. convincing : decision
C. gargantuan : size
D. ancient : value
E. prototypical : affection

7. FEBRILE : ILLNESS ::

A. tenacious : astonishment
B. juvenile : maturity
C. classic : cultivation
D. eccentric : discrimination
E. delusional : insanity

8. EQUIVOCATION : MEANING ::

A. feint : intention
B. secrecy : stealth
C. geniality : amiability
D. travesty : insight
E. refinement : innovation

Choose the lettered word or phrase that is most nearly opposite in
meaning to the word in capital letters.

9. WHIMSICAL :

A. chivalrous
B. perfect
C. predictable
D. hidden
E. backward


10. REVERE :

A. collide
B. succumb
C. threaten
D. divide
E. despise
Building Standards in Educational and Professional Testing
National Testing Service Pakistan Overseas Scholarship Scheme for PhD Studies

11. INURED :

A. authoritative
B. dissolute
C. bereft
D. sensitive
E. taxing

12. ALACRITY :

A. skullduggery
B. reluctance
C. interment
D. bellicosity
E. specificity

Read the passages and answer the questions asked at its end.

Art, like words, is a form of communication. Words, spoken and written, render
accessible to humans of the latest generations all the knowledge discovered by the
experience and reflection, both of preceding generations and of the best and foremost
minds of their own times. Art renders accessible to people of the latest generations all
the feelings experienced by their predecessors, and those already felt by their best and
foremost contemporaries. Just as the evolution of knowledge proceeds by dislodging and
replacing that which is mistaken, so too the evolution of feeling proceeds through art.
Feelings less kind and less necessary for the well-being of humankind are replaced by
others kinder and more essential to that end. This is the purpose of art, and the more
art fulfills that purpose the better the art; the less it fulfills it, the worse the art.

13. The author develops the passage primarily by

A. theory and refutation
B. example and generalization
C. comparison and contrast
D. question and answer
E. inference and deduction

14. According to the author, knowledge is

A. evolutionary and emotional
B. cumulative and progressive
C. static and unmoving
D. dynamic and cyclical
E. practical and directionless

15. According to the passage, all of the following are true EXCEPT:

A. Art is a form of communication.
B. Art helps to refine sensibilities.
C. Art is a repository of experience.
D. Real art can never be bad.
E. Art is a progressive human endeavor.
Building Standards in Educational and Professional Testing
Building Standards in Education and Professional Testing 133
II
Analytical
Reasoning
Section
No. Of
Questions
20
Choose the correct answer for
each question and shade the
corresponding CIRCLE in the
answer sheet

Questions 16-19 are based on the following.

The Western Derby is a race held annually at Bayshore Racetrack. There are eight
gates at the racetrack, but only seven horses are entered in this raceJulius
Caesar, King's Bounty, Longshot, Man Among Boys, Nocturnal, Odyssey, and
Phantom. One of the gates is left empty. The horses are at the gate, waiting for the
race to begin.

Gate 1, on the inside of the racetrack, is occupied.
Phantom is at a gate inside of Nocturnal.
The number of gates separating Julius Caesar and King's Bounty equals the number
of gates separating Longshot and Man among Boys.
Nocturnal and Odyssey are next to each other.


16. If Odyssey is at Gate 2, which of the following must be true?

A. Nocturnal is at the innermost gate.
B. King's Bounty is at the outermost gate.
C. A horse occupies the outermost gate.
D. Phantom is at the innermost gate.
E. The outermost gate is not empty.

17. Which of the following is a possible assignment for the horses, from the inside to
the outside?

A. Phantom, King's Bounty, Julius Caesar, Odyssey, Nocturnal, Man Among
Boys, Longshot, vacant
B. vacant, Phantom, Julius Caesar, Longshot, King's Bounty, Man Among Boys,
Nocturnal, Odyssey
C. Longshot, Man Among Boys, Nocturnal, vacant, Phantom, Odyssey, King's
Bounty, Julius Caesar
D. Julius Caesar, King's Bounty, Longshot, Phantom, vacant, Man Among Boys,
Nocturnal, Odyssey
E. Phantom, Julius Caesar, Nocturnal, vacant, Odyssey, King's Bounty,
Longshot, Man Among Boys


18. If Julius Caesar is at Gate 6, King's Bounty is at Gate 7, and Odyssey is at Gate
4, which of the following must be true?
I. Longshot is at Gate 1.
II. Nocturnal is at Gate 5.
III. Man Among Boys is at Gate 2.
IV. Gate 8 is vacant.
Building Standards in Educational and Professional Testing
Building Standards in Education and Professional Testing 134


A. I and II only
B. II and III only
C. II and IV only
D. I, II, and III only
E. I, II, III, and IV

19. If Julius Caesar and King's Bounty are at the second and fourth gates,
respectively, all of the following can be true EXCEPT

A. Phantom is at Gate 1
B. Man Among Boys is at Gate 3
C. Longshot is at Gate 6
D. Odyssey is at Gate 7
E. Nocturnal is at Gate 7

20. Studies have shown that families who install smoke detectors and own fire
extinguishers have a reduced risk of losing a child in a house fire. Therefore, no
family who installs smoke detectors and owns a fire extinguisher will lose a child
in a house fire.

Of the following, the best criticism of the argument above is that the argument
does not

A. take into account the possibility of losing a child in a house fire despite all
precautionary measures
B. indicate that fire extinguishers are effective during early stages of a fire
C. cite the fact that smoke detectors have proven to be effective in waking
sleeping children during a house fire
D. differentiate between the two major causes of house fires: cooking and
heating
E. take into account that families who buy smoke detectors are also more likely
to purchase fire insurance

21. LSD is a drug known to cause synesthesia, a phenomenon in which sensory input
somehow becomes interchanged in the brain: a person with synesthesia might
smell a symphony, hear sun light, or taste a pinprick. While most cases are drug
induced, some people suffer from synesthesia in various forms since birth.
Which of the following can be most safely inferred from the information above?

A. Synesthesia is not always a drug-induced phenomenon.
B. Some great artists of this century have been known for their synesthetic
proclivities.
C. LSD is an addictive drug.
D. Synesthesia is rarely bothersome to those who experience it.
E. Synesthesia at birth is a result of mothers who have tried LSD.

22. Palindromes are easier to solve than acrostics, but acrostics are more difficult to
create than palindromes. Rebuses are more difficult to solve than acrostics, yet
rebuses are easier to create than palindromes.
Building Standards in Educational and Professional Testing
Building Standards in Education and Professional Testing 135
If the above information is true, then it must also be true that

A. acrostics are more difficult to create than rebuses
B. palindromes are more difficult to solve than rebuses
C. rebuses are easier to solve than acrostics
D. acrostics are easier to create than rebuses
E. rebuses are easier to solve than palindromes
Questions 23-25 are based on the following.

A university has a procedure for registering and recording complaints. Due to
strict bureaucratic regulations, the following system of passing complaints must be
observed:

A is the first registrar to receive all incoming complaints.
F is the recorder and final administrator to handle a complaint.
Personnel B, C, D, and E may pass complaints only as follows:
A to B
B to either C or D
C to either B or E
D to C
E to either D or F

23. Which is an acceptable path for a complaint to follow, passing from A?

A. B to C to D to F
B. B to D to C to F
C. B to C to E to F
D. B to E to F
E. D to C to F

24. If a complaint is received and is handled by each personnel member only one
time, which of the following could be one of the passes?

A. A to C
B. C to B
C. C to F
D. D to C
E. E to D

25. Between which two personnel may a complaint pass by means of two different
paths without any duplication of passes?

A. B to E
B. C to D
C. C to E
D. D to B
E. E to B

Building Standards in Educational and Professional Testing
Building Standards in Education and Professional Testing 136
Questions 26-31 are based on the following.

In a baseball field, one team can practice at a time. There are seven teamsthe
Aces, the Bears, the Cubs, the Ducks, the Eagles, the Falcons, and the Giants.
The baseball field is open seven evenings a week from Monday to Sunday
(Sunday being considered the last day of the week), and the allocation of
practice times is governed by the following rules:

On any evening, only one team can play.
The Aces must practice on Monday.
The Ducks practice exactly one day before the Falcons practice.
The Falcons practice exactly one day before the Giants practice.
The Cubs and the Bears must practice earlier in the week than the Eagles.

26. The latest day in the week that the Bears can practice is

A. Tuesday
B. Wednesday
C. Thursday
D. Friday
E. Saturday

27. If a person went to the baseball field on three consecutive evenings, he or she
could see which of the following teams in the order listed?

A. the Falcons, the Giants, the Cubs
B. the Falcons, the Giants, the Ducks
C. the Aces, the Ducks, the Cubs
D. the Bears, the Cubs, the Falcons
E. the Ducks, the Eagles, the Falcons

28. One week, the Cubs practiced on Wednesday and the Ducks practiced the next
day. That week, the Bears must have practiced on

A. Monday
B. Tuesday
C. Friday
D. Saturday
E. Sunday

29. If the Giants practice on Thursday, the Eagles and the Ducks must practice on
which days, respectively?

A. Sunday and Tuesday
B. Saturday and Tuesday
C. Friday and Wednesday
D. Wednesday and Thursday
E. Tuesday and Monday

30. If the Falcons practice on Saturday, the Eagles must practice on what day?

A. Tuesday
Building Standards in Educational and Professional Testing
Building Standards in Education and Professional Testing 137
B. Wednesday
C. Thursday
D. Friday
E. Sunday

31. The practice schedule has to adhere to which of the following?

A. The Ducks practice earlier in the week than the Eagles.
B. The Falcons practice on a later day than the Eagles.
C. The Falcons practice earlier in the week than the Giants.
D. The Cubs practice earlier in the week than the Ducks.
E. The Bears practice earlier in the week than the Cubs.

32. Wine, cheese, butter, and raisins are all examples of early techniques to preserve
food. In modern times, food scientists have developed other techniques such as
dehydration, hermetic sealing, and radiation. Of these, radiation is the most con-
troversial because preliminary studies have shown that radiation alters the
natural chemical bonds in fruits and vegetables. Instead of providing salutary
effects, eating radiated produce may well introduce irritating chemicals into the
body, creating a possible health hazard.
Which of the following, if true, supports the conclusion that eating radiated
produce poses a possible health hazard?
A. Radiation affects only those chemical bonds associated with water, that is,
hydrogen and oxygen.
B. Radiation kills microorganisms that hasten food decay.
C. The radiation-induced bonds are unlike any of those found in non-radiated
produce.
D. Certain microorganisms, namely those found in yogurt cultures, are essential
for proper digestion.
E. Radiation has no effect on foods preserved by drying.

33. Blue Blood, Inc., is a private blood products company that buys blood only from
qualified donors. To qualify, a person must weigh at least 105 pounds, must not
have taken malaria medication in the last three years, must never have had
hepatitis, and must never have used intravenous drugs. Blue Blood nurses know
that traveling has an effect on the possibilities for blood donation: Everyone who
travels to Malaysia is required to take malaria medication; no one who enters
Singapore can have ever used intravenous drugs; everyone traveling to Gorisimi
gets hepatitis.
Which of the following situations would not automatically disqualify a person
from selling blood to Blue Blood?

A. traveling to Malaysia two years ago
B. having once weighed 110 pounds and now weighing 95 pounds
C. being denied admission to Singapore
D. traveling to Gorisimi five years ago
E. using intravenous drugs that were legal at the time

Building Standards in Educational and Professional Testing
Building Standards in Education and Professional Testing 138
34. Before marriage, couples should be tested for AIDS and any other sexually
communicable diseases. Negative results will guarantee the health and safe-ness
of their marriage.
Which of the following is an assumption of the argument in the passage
above?

A. Current state laws require couples who are planning to get married to be
tested for infectious disease in order to prevent possible health problems in
the future.
B. There are many infectious diseases that can be sexually transmitted from
one individual to another.
C. Fortunately even if a test proves positive for a communicable disease,
couples can still lead healthy marriages by taking the proper precautions.
D. Due to advances in medical research over the years, infectious diseases that
used to be fatal can now be effectively treated.
E. All the diseases detectable through testing have no incubation period and the
results of these tests can immediately indicate whether or not the individual
has the disease.

Question 35 is based on the following.

Nine athletes attend a sports banquet. Three of the athletes}, K, and Lare varsity
football players; two of the athletesM and Nare varsity basketball players. The
other four athletes O, P, Q, and Rbelong to the hockey club. All nine athletes will
be seated at three small tables, each seating three athletes. The athletes must be
seated according to the following rules: O and J do not sit at the same table.
P sits together with at least one of K or M.
There can be at most only one football player at a table.
There can be at most only one basketball player at a table.

35. Suppose just one varsity athlete sits at a certain table, and that athlete happens
to be J. If so, who else sits with J?

A. P, Q
B. P, R
C. Q, R
D. O, Q
E. O, P






Building Standards in Educational and Professional Testing
Building Standards in Education and Professional Testing 139




36. If y = 9, then y
2
- y =

A. 3-9
B. 0
C. 9-3
D. 6552
E. 6561

37. If (x+3)/6 = 12/(x+4), what is the positive value of x?

A. 2
B. 3
C. 5
D. 60
E. 12

38. Cindy wants to paint her office. She can buy three cans of the same-priced paint
and three identical brushes for $21, or she can buy four cans of the same paint
and one brush for $22. How much does a can of paint cost?

A. $2
B. $3
C. $4
D. $5
E. $6

39. Which of the following must be true?

I. (25 - 81) = (5 - 9) (5 + 9)
II. 7(9 + 6) = 7(9) + 7(6)
III. 6 (3 - 1) = (6 3) - (61)

A. I only
B. II only
C. III only
D. I and II only
E. I, II, and III


40. The sum of a and 9 - 2a is less than 8. Which of the following is (are) the
value(s) of a?

I. a<-1
II. a< 1
III
Quantitative
Section
No of
Questions
15
Choose the correct answer for
each question and shade the
corresponding CIRCLE in the
answer sheet
Building Standards in Educational and Professional Testing
Building Standards in Education and Professional Testing 140
III. a>1

A. I only
B. II only
C. III only
D. I and II only
E. I and III only

41. Susan is having a party. At 7:00 P.M., guests begin arriving at a uniform rate of
8 people every 15 minutes. If this pattern continues, how many guests will have
arrived by 9:30 P.M.?

F. 10
G. 20
H. 40
I. 64
J. 80

42. For positive integers p and q, if p
2
+ 2q
2
= 41, and 2p
2
+q
2
= 34, then p
2
=

A. 2.5
B. 7
C. 3
D. 9
E. 16

43. If a:b is 7:6 and 3b:2c is 2:3, what is c/a ?

A. 14/27
B. 7/9
C. 6/7
D. 9/7
E. 27/14



Building Standards in Educational and Professional Testing
Building Standards in Education and Professional Testing 141
44. In the figure above, if the radius of the circle is 8, and triangle TRS is inscribed
in the circle, then the length of arc TRS is

A. 16/3
B. (32 )/3
C. 16
D. (128 )/3
E. 64

45. For developing pictures, XYZ Photo Lab charges a service fee of $3 for every
order it receives in addition to a printing fee. If the order consists of 12 pictures
or less, the printing fee per picture is $0.36. If the order consists of more than
12 pictures, the printing fee per picture is $0.24. What is the total cost per
picture for an order consisting of 30 pictures?

A. $0.11
B. $0.24
C. $0.34
D. $0.46
E. $3.24

46. Lisa found an easy way to add up a sequence of positive even integers with an
even number of terms. She formed pairs of equal sums by adding the first
integer to the last, the second integer to the next-to-last, and so on. She then
computed the total by adding these equal sums. If the total Lisa obtained was
930, how many terms were there in the sequence of positive even integers if
the sequence started with the number 2?

A. 30
B. 39
C. 40
D. 60
E. 465

47. December is the busiest month at Lamont's Gift Shoppe, where sales in
December are 40 percent higher than average. If sales in February are typically
20 percent lower than average, what is the ratio of February sales to December
sales?

A. 1:2
B. 4:2
C. 4:5
D. 4:7
E. 6:7

48. How many 4-digit numbers are there that consist of only odd digits?

A. 20
B. 625
C. 1,024
D. 4,500
E. 5,000
Building Standards in Educational and Professional Testing
Building Standards in Education and Professional Testing 142

49. For some integer m, let [m] be defined by the equation {m} = m (1- m). If n +
1 = {n + 1}, then n =

A. -2
B. -1
C. 0
D. 1
E. 2


50. Box A and box B have 6 cards each. Each card is marked with one integer, 1
through 6. Both boxes can have more than one card with the same integer, but
the sum of all the integers in each box must be 18. Two of the cards in box/1
are 6's and two of the cards in box B are 5's. If one card is drawn from box A
and one from box B, but neither a 6 nor a 5 is drawn, what is the largest pos-
sible sum of the integers on the cards drawn from the two boxes?

A. 3
B. 4
C. 7
D. 8
E. 12

You might also like